Evoluution loppu?

onko.loppu

Missä vaiheessa ihmisen evoluutio loppui? Kertokaapa käsityksenne ensin ja jatkan sitten. Toivon asiallista keskustelua.

191

1744

    Vastaukset

    Anonyymi (Kirjaudu / Rekisteröidy)
    5000
    • Tuota... Kukas sen on sanonut, että ihmisen evoluutio olisi loppunut? Sitä ei ole tapahtunut. Kehitymme edelleen. Se vain ei ole nopeaa, joten muutokset näkyvät hitaasti.

      • Fyi

        Mitkä muutokset?


      • Msmslsl
        tutkimusta kirjoitti:

        https://www.tiede.fi/artikkeli/uutiset/ihmisen-evoluutio-jatkuu-yha

        Jatkuu toki.

        Geneetikko John Sanford: Ihmiskunta rappeutuu pikavauhtia:

        ”Meidän sukupolvellamme on keskimäärin 100-200 mutaatiota enemmän kuin vanhemmillamme, ja heillä on niitä saman verran enemmän kuin heidän vanhemmillaan. Tämä johtuu siitä, että joka kerta, kun DNA:ta välittyy sukupolvelta toiselle, siihen ilmestyy lisää mutaatioita.

        Mutaatiot ovat enimmäkseen vahingollisia. Ne ovat geneettisiä kirjoitusvirheitä, mutta darvinismin mukaan ne ovat yksi evoluution kivijaloista, ja niiden pitäisi miljoonien vuosien saatossa parantaa lajin elinkelpoisuutta.

        Tämä kivijalka rapistuu kuitenkin melko nopeasti. Kehitys kulkee väärään suuntaan.

        Cornellin yliopiston professori John Sanford luennoi aiheesta hiljattain Darwin Was Wrong -konferenssissa. Geneetikko Sanford kertoi, että darvinistit ovat aliarvioineet vahingollisten mutaatioiden määrää, koska he eivät ole laskeneet ”roska-DNA:n” muutoksia. Näin ollen heiltä on jäänyt huomioimatta 90 prosenttia mutaatioista.

        Professori Sanford ei ole ainoa, joka sanoo ihmisten rappeutuvan. Myös Nature News vahvisti hiljattain hänen havaintonsa. Tulos kävi ilmi, kun tutkijat sekvensoivat Y-kromosomia.

        Tästä voitaneen päätellä, että mitä enemmän aikaa kuluu, sitä pahemmin ihmiskunta rappeutuu. Evoluutio kulkee siis täysin väärään suuntaan. Vuosimiljoonat merkitsisivät sukupuuttoa.

        Myös lääkäri Pekka Reinikainen ja biokemisti Pauli Ojala ovat ehdottaneet, että näemme evoluution sijasta devoluutiota eli kehityksen asemesta rappeutumista.

        Lähteet:

        Dolgin, Elie. 2009. Human mutation rate revealed. Nature News (27.8.). http://www.nature.com/news/2009/090827/full/news.2...

        Sanford, John. 2009. Darwin Was Wrong about Natural Selection. Luento Darwin Was Wrong konferenssissa (14.11.)

        http://bwanajoe.blogspot.com/2009/11/geneetikko-jo...

        Uusi tutkimus näyttää, etta rappeutuminen keräytyy geenibooliin:

        "Take, to begin with, the Swedish chickens. Three years ago, researchers led by a professor at the university of Linköping in Sweden created a henhouse that was specially designed to make its chicken occupants feel stressed. The lighting was manipulated to make the rhythms of night and day unpredictable, so the chickens lost track of when to eat or roost. Unsurprisingly, perhaps, they showed a significant decrease in their ability to learn how to find food hidden in a maze.

        The surprising part is what happened next: the chickens were moved back to a non-stressful environment, where they conceived and hatched chicks who were raised without stress - and yet these chicks, too, demonstrated unexpectedly poor skills at finding food in a maze. They appeared to have inherited a problem that had been induced in their mothers through the environment.

        Further research established that the inherited change had altered the
        chicks' "gene expression" - the way certain genes are turned "on" or "off", bestowing any given animal with specific traits. The stress had affected the mother hens on a genetic level, and they had passed it on to their offspring."

        Eli ongelma oli periytynyt äidiltä lapselle päinvastoin kuin evoteoria ennustaa.

        Eli kanat eivät kehittyneet ympäristön mukaan, vaan päinvastoin heidän geeninsä muuttuivat huonommaksi uuden ympäristön mukaan! Eli kävi päinvastoin kuin mitä evoluutioteoria ennustaa...

        http://www.guardian.co.uk/science/2010/mar/19/evol...

        Biologisen informaation mitattavuus:

        ”Nykyään elää monia lentokyvyttömiä lintulajeja, joille siivillä ei ole sama merkitys kuin niiden lentäville sukulaisille. Evolutionistit pitävät siipiä surkastumina tai keskeneräisinä rakenteina.


      • Msmsmsmsm

        HS tiedepalsta tarinoi taas kerran (17.7): ”IHMISEN ALKUPERÄ: Tutkijat löysivät Kiinasta maailman vanhimmat todisteet varhaisista ihmisistä Afrikan ulkopuolella”.

        Tällä kertaa löydettiin ”2,1 miljoonaa vuotta” vanhoja kivityökaluja Aasiasta ja niiden käyttäjän kuvakin komeilee artikkelin yhteydessä! ”Työkalujen ikä on ajoitettu suurella varmuudella, ja löytö mullistaa käsityksiä varhaisten ihmisten liikkeistä”.

        Ajoitettu suurella varmuudella!?

        ”TYÖKALUT ajoitettiin hyödyntämällä niin kutsuttua paleomagnetismia. Se perustuu siihen, että maapallon magneettikentän navat vaihtavat paikkaa aika ajoin, ja jälki näistä muinaisista magneettikentän käännöksistä jää kivikerrostumien magneettisiin mineraaleihin. Ulkopuoliset tutkijat pitävät ryhmän tekemää ajoitusta hyvin vankkana. Vanhimmat työkaluista ovat 2,12 miljoonan ja tuoreimmat 1,26 miljoonan vuoden ikäisiä. Jokin varhainen ihmislaji on siis asunut alueella hyvin pitkään”.

        Mikä tässä on ongelma?
        Paleomagnetismin tulkinta on kiistanalaista ja magneettisuuden muutokset ovat voineet tapahtua nopeasti. Väite, jonka mukaan saman alueen työkalujen ikä vaihtelisi lähes miljoonalla (!) vuodella ei ota huomioon, että eroosio olisi ohentanut alueen maakerrostumia tässä ajassa noin puolen kilometrin verran. Artikkelin lopussa todetaan, että Homo Sapiens olisi ollut olemassa 0,3 miljoonaa vuotta. Tämä ei ole mahdollista perimän rapeutumisen takia, sillä tuossa ajassa se olisi kuollut sukupuuttoon jo kymmeniä kertoja, kuten käy ilmi JC Sanfordin kirjasta Eliömaailma rappeutuu.

        https://luominen.fi/pekan-blogi/ihmisen-sukupuu-muuttui-taas


      • lslsl

        "Uusi tutkimus näyttää, etta rappeutuminen keräytyy geenibooliin:"

        Kahdeksan vuotta vanha tutkimus ei ole enää uusi. Tarjoamasi linkki ei edes toimi.


      • Lslsls
        lslsl kirjoitti:

        "Uusi tutkimus näyttää, etta rappeutuminen keräytyy geenibooliin:"

        Kahdeksan vuotta vanha tutkimus ei ole enää uusi. Tarjoamasi linkki ei edes toimi.

        Kahdeksan vuotta on miljardeissa vuosisaa lyhyt aika.


      • Msmslsl kirjoitti:

        Geneetikko John Sanford: Ihmiskunta rappeutuu pikavauhtia:

        ”Meidän sukupolvellamme on keskimäärin 100-200 mutaatiota enemmän kuin vanhemmillamme, ja heillä on niitä saman verran enemmän kuin heidän vanhemmillaan. Tämä johtuu siitä, että joka kerta, kun DNA:ta välittyy sukupolvelta toiselle, siihen ilmestyy lisää mutaatioita.

        Mutaatiot ovat enimmäkseen vahingollisia. Ne ovat geneettisiä kirjoitusvirheitä, mutta darvinismin mukaan ne ovat yksi evoluution kivijaloista, ja niiden pitäisi miljoonien vuosien saatossa parantaa lajin elinkelpoisuutta.

        Tämä kivijalka rapistuu kuitenkin melko nopeasti. Kehitys kulkee väärään suuntaan.

        Cornellin yliopiston professori John Sanford luennoi aiheesta hiljattain Darwin Was Wrong -konferenssissa. Geneetikko Sanford kertoi, että darvinistit ovat aliarvioineet vahingollisten mutaatioiden määrää, koska he eivät ole laskeneet ”roska-DNA:n” muutoksia. Näin ollen heiltä on jäänyt huomioimatta 90 prosenttia mutaatioista.

        Professori Sanford ei ole ainoa, joka sanoo ihmisten rappeutuvan. Myös Nature News vahvisti hiljattain hänen havaintonsa. Tulos kävi ilmi, kun tutkijat sekvensoivat Y-kromosomia.

        Tästä voitaneen päätellä, että mitä enemmän aikaa kuluu, sitä pahemmin ihmiskunta rappeutuu. Evoluutio kulkee siis täysin väärään suuntaan. Vuosimiljoonat merkitsisivät sukupuuttoa.

        Myös lääkäri Pekka Reinikainen ja biokemisti Pauli Ojala ovat ehdottaneet, että näemme evoluution sijasta devoluutiota eli kehityksen asemesta rappeutumista.

        Lähteet:

        Dolgin, Elie. 2009. Human mutation rate revealed. Nature News (27.8.). http://www.nature.com/news/2009/090827/full/news.2...

        Sanford, John. 2009. Darwin Was Wrong about Natural Selection. Luento Darwin Was Wrong konferenssissa (14.11.)

        http://bwanajoe.blogspot.com/2009/11/geneetikko-jo...

        Uusi tutkimus näyttää, etta rappeutuminen keräytyy geenibooliin:

        "Take, to begin with, the Swedish chickens. Three years ago, researchers led by a professor at the university of Linköping in Sweden created a henhouse that was specially designed to make its chicken occupants feel stressed. The lighting was manipulated to make the rhythms of night and day unpredictable, so the chickens lost track of when to eat or roost. Unsurprisingly, perhaps, they showed a significant decrease in their ability to learn how to find food hidden in a maze.

        The surprising part is what happened next: the chickens were moved back to a non-stressful environment, where they conceived and hatched chicks who were raised without stress - and yet these chicks, too, demonstrated unexpectedly poor skills at finding food in a maze. They appeared to have inherited a problem that had been induced in their mothers through the environment.

        Further research established that the inherited change had altered the
        chicks' "gene expression" - the way certain genes are turned "on" or "off", bestowing any given animal with specific traits. The stress had affected the mother hens on a genetic level, and they had passed it on to their offspring."

        Eli ongelma oli periytynyt äidiltä lapselle päinvastoin kuin evoteoria ennustaa.

        Eli kanat eivät kehittyneet ympäristön mukaan, vaan päinvastoin heidän geeninsä muuttuivat huonommaksi uuden ympäristön mukaan! Eli kävi päinvastoin kuin mitä evoluutioteoria ennustaa...

        http://www.guardian.co.uk/science/2010/mar/19/evol...

        Biologisen informaation mitattavuus:

        ”Nykyään elää monia lentokyvyttömiä lintulajeja, joille siivillä ei ole sama merkitys kuin niiden lentäville sukulaisille. Evolutionistit pitävät siipiä surkastumina tai keskeneräisinä rakenteina.

        "Tästä voitaneen päätellä, että mitä enemmän aikaa kuluu, sitä pahemmin ihmiskunta rappeutuu. "

        Kun tästä "rappeutumisesta" on taidettu "kantaa huolta" jo vuosisatoja, niin vähän hämäräksi jää, mitä tällä rappeutumisella siteen oikeasti tarkoitetaan?


      • KreationistinenRealismi
        Msmslsl kirjoitti:

        Geneetikko John Sanford: Ihmiskunta rappeutuu pikavauhtia:

        ”Meidän sukupolvellamme on keskimäärin 100-200 mutaatiota enemmän kuin vanhemmillamme, ja heillä on niitä saman verran enemmän kuin heidän vanhemmillaan. Tämä johtuu siitä, että joka kerta, kun DNA:ta välittyy sukupolvelta toiselle, siihen ilmestyy lisää mutaatioita.

        Mutaatiot ovat enimmäkseen vahingollisia. Ne ovat geneettisiä kirjoitusvirheitä, mutta darvinismin mukaan ne ovat yksi evoluution kivijaloista, ja niiden pitäisi miljoonien vuosien saatossa parantaa lajin elinkelpoisuutta.

        Tämä kivijalka rapistuu kuitenkin melko nopeasti. Kehitys kulkee väärään suuntaan.

        Cornellin yliopiston professori John Sanford luennoi aiheesta hiljattain Darwin Was Wrong -konferenssissa. Geneetikko Sanford kertoi, että darvinistit ovat aliarvioineet vahingollisten mutaatioiden määrää, koska he eivät ole laskeneet ”roska-DNA:n” muutoksia. Näin ollen heiltä on jäänyt huomioimatta 90 prosenttia mutaatioista.

        Professori Sanford ei ole ainoa, joka sanoo ihmisten rappeutuvan. Myös Nature News vahvisti hiljattain hänen havaintonsa. Tulos kävi ilmi, kun tutkijat sekvensoivat Y-kromosomia.

        Tästä voitaneen päätellä, että mitä enemmän aikaa kuluu, sitä pahemmin ihmiskunta rappeutuu. Evoluutio kulkee siis täysin väärään suuntaan. Vuosimiljoonat merkitsisivät sukupuuttoa.

        Myös lääkäri Pekka Reinikainen ja biokemisti Pauli Ojala ovat ehdottaneet, että näemme evoluution sijasta devoluutiota eli kehityksen asemesta rappeutumista.

        Lähteet:

        Dolgin, Elie. 2009. Human mutation rate revealed. Nature News (27.8.). http://www.nature.com/news/2009/090827/full/news.2...

        Sanford, John. 2009. Darwin Was Wrong about Natural Selection. Luento Darwin Was Wrong konferenssissa (14.11.)

        http://bwanajoe.blogspot.com/2009/11/geneetikko-jo...

        Uusi tutkimus näyttää, etta rappeutuminen keräytyy geenibooliin:

        "Take, to begin with, the Swedish chickens. Three years ago, researchers led by a professor at the university of Linköping in Sweden created a henhouse that was specially designed to make its chicken occupants feel stressed. The lighting was manipulated to make the rhythms of night and day unpredictable, so the chickens lost track of when to eat or roost. Unsurprisingly, perhaps, they showed a significant decrease in their ability to learn how to find food hidden in a maze.

        The surprising part is what happened next: the chickens were moved back to a non-stressful environment, where they conceived and hatched chicks who were raised without stress - and yet these chicks, too, demonstrated unexpectedly poor skills at finding food in a maze. They appeared to have inherited a problem that had been induced in their mothers through the environment.

        Further research established that the inherited change had altered the
        chicks' "gene expression" - the way certain genes are turned "on" or "off", bestowing any given animal with specific traits. The stress had affected the mother hens on a genetic level, and they had passed it on to their offspring."

        Eli ongelma oli periytynyt äidiltä lapselle päinvastoin kuin evoteoria ennustaa.

        Eli kanat eivät kehittyneet ympäristön mukaan, vaan päinvastoin heidän geeninsä muuttuivat huonommaksi uuden ympäristön mukaan! Eli kävi päinvastoin kuin mitä evoluutioteoria ennustaa...

        http://www.guardian.co.uk/science/2010/mar/19/evol...

        Biologisen informaation mitattavuus:

        ”Nykyään elää monia lentokyvyttömiä lintulajeja, joille siivillä ei ole sama merkitys kuin niiden lentäville sukulaisille. Evolutionistit pitävät siipiä surkastumina tai keskeneräisinä rakenteina.

        John Sanford on nuoren Maan kreationisti. Mitä muuta häneltä voi odottaa kuin tuollaista julistusta. Muut geneetikot eivät vaan suostu allekirjoittamaan Sanfordin väitteitä, vaan pitävät hänen "tutkimuksiaan" päämäärähakuisina ja siten vääristeltyinä.
        Entisen NL:n aikoihin kaiken yhteiskuntatieteellisen tutkimuksen piti tukea kommunismia. Tulos oli jo enneen tutkimusta tiedossa.


      • KreationistinenRealismi
        Msmsmsmsm kirjoitti:

        HS tiedepalsta tarinoi taas kerran (17.7): ”IHMISEN ALKUPERÄ: Tutkijat löysivät Kiinasta maailman vanhimmat todisteet varhaisista ihmisistä Afrikan ulkopuolella”.

        Tällä kertaa löydettiin ”2,1 miljoonaa vuotta” vanhoja kivityökaluja Aasiasta ja niiden käyttäjän kuvakin komeilee artikkelin yhteydessä! ”Työkalujen ikä on ajoitettu suurella varmuudella, ja löytö mullistaa käsityksiä varhaisten ihmisten liikkeistä”.

        Ajoitettu suurella varmuudella!?

        ”TYÖKALUT ajoitettiin hyödyntämällä niin kutsuttua paleomagnetismia. Se perustuu siihen, että maapallon magneettikentän navat vaihtavat paikkaa aika ajoin, ja jälki näistä muinaisista magneettikentän käännöksistä jää kivikerrostumien magneettisiin mineraaleihin. Ulkopuoliset tutkijat pitävät ryhmän tekemää ajoitusta hyvin vankkana. Vanhimmat työkaluista ovat 2,12 miljoonan ja tuoreimmat 1,26 miljoonan vuoden ikäisiä. Jokin varhainen ihmislaji on siis asunut alueella hyvin pitkään”.

        Mikä tässä on ongelma?
        Paleomagnetismin tulkinta on kiistanalaista ja magneettisuuden muutokset ovat voineet tapahtua nopeasti. Väite, jonka mukaan saman alueen työkalujen ikä vaihtelisi lähes miljoonalla (!) vuodella ei ota huomioon, että eroosio olisi ohentanut alueen maakerrostumia tässä ajassa noin puolen kilometrin verran. Artikkelin lopussa todetaan, että Homo Sapiens olisi ollut olemassa 0,3 miljoonaa vuotta. Tämä ei ole mahdollista perimän rapeutumisen takia, sillä tuossa ajassa se olisi kuollut sukupuuttoon jo kymmeniä kertoja, kuten käy ilmi JC Sanfordin kirjasta Eliömaailma rappeutuu.

        https://luominen.fi/pekan-blogi/ihmisen-sukupuu-muuttui-taas

        Nisäkäsljien keskimääräinen elinaika on neljä miljoonaa vuotta. Eivät nekään 300 000 vuodessa rappeudu.
        Aikamäärittelyt kalibroidaan aina muilla menetelmillä. Jos tulos on yli miljoona vuotta, ei sitä alle kuudeksi tuhanneksi väännä millään. Lapsellisia nuo kreationistit.


      • MissäSeNäkyy
        mummomuori kirjoitti:

        "Tästä voitaneen päätellä, että mitä enemmän aikaa kuluu, sitä pahemmin ihmiskunta rappeutuu. "

        Kun tästä "rappeutumisesta" on taidettu "kantaa huolta" jo vuosisatoja, niin vähän hämäräksi jää, mitä tällä rappeutumisella siteen oikeasti tarkoitetaan?

        Ihmiskunta on lukuisampaa, kookkaampaa ja terveempää kuin ikinä. Kadehdittavaa rappeutumista.


      • KreationistinenRealismi kirjoitti:

        John Sanford on nuoren Maan kreationisti. Mitä muuta häneltä voi odottaa kuin tuollaista julistusta. Muut geneetikot eivät vaan suostu allekirjoittamaan Sanfordin väitteitä, vaan pitävät hänen "tutkimuksiaan" päämäärähakuisina ja siten vääristeltyinä.
        Entisen NL:n aikoihin kaiken yhteiskuntatieteellisen tutkimuksen piti tukea kommunismia. Tulos oli jo enneen tutkimusta tiedossa.

        "Geenipyssy" Sanford on jatkuvasti kretujen tapetilla, koska paljon muitakaan heikäläisiä ei läntisen maailman sadoista tuhansista luonnontieteilijöistä löydy. Toni Mikael Torpan jutuissa hän seikkailee lähes yhtä usein kuin muuan bioprosessitekniikan emeritusprofessori.


      • KreationistinenRealismi kirjoitti:

        John Sanford on nuoren Maan kreationisti. Mitä muuta häneltä voi odottaa kuin tuollaista julistusta. Muut geneetikot eivät vaan suostu allekirjoittamaan Sanfordin väitteitä, vaan pitävät hänen "tutkimuksiaan" päämäärähakuisina ja siten vääristeltyinä.
        Entisen NL:n aikoihin kaiken yhteiskuntatieteellisen tutkimuksen piti tukea kommunismia. Tulos oli jo enneen tutkimusta tiedossa.

        >Entisen NL:n aikoihin kaiken yhteiskuntatieteellisen tutkimuksen piti tukea kommunismia. Tulos oli jo ennen tutkimusta tiedossa.

        Ei ihan kaiken, mutta kaiken mikä julkaistiin ulos piti olla jos ei myönteistä niin ainakin rakentavaa. NKP:n korkein johto oli valtakunnan viimeisinä vuosikymmeninä hyvin informoitu yhteiskunnan todellisuudesta, mutta kaikki se tieto oli lähtökohtaisesti erittäin salaista.


      • MissäSeNäkyy kirjoitti:

        Ihmiskunta on lukuisampaa, kookkaampaa ja terveempää kuin ikinä. Kadehdittavaa rappeutumista.

        Jos liikkuisimme enemmän, ero entiseen olisi suurempikin. Halu elää mahdollisimman mukavasti ei kuitenkaan johdu geneettisestä rappeutumisesta, sillä se on ollut olemassa ihmiskunnan alkuhämäristä saakka. Toteutus ei vain ennen onnistunut kuin harvoilta.


      • Nykyluonnontietieteissä po. Nykyluonnontieteissä


      • lisäävirheitä
        mummomuori kirjoitti:

        "Tästä voitaneen päätellä, että mitä enemmän aikaa kuluu, sitä pahemmin ihmiskunta rappeutuu. "

        Kun tästä "rappeutumisesta" on taidettu "kantaa huolta" jo vuosisatoja, niin vähän hämäräksi jää, mitä tällä rappeutumisella siteen oikeasti tarkoitetaan?

        Sillä tarkoitataan sitä, että jatkuvasti syntyy uusia sairauksia ja löydetään uusia virhegeenejä!!!
        Vaikka hoitomenetelmät ovat parantuneet, niin sairastuvuus ei vähene.


      • evlut101

        Jokainen joka on tehnyt ohjelmointityötä tietää, että ohjelmaa ei voi kehittää evoluutioprosessilla eli että
        a) luodaan satunnaisia virheitä koodiin
        b) karsitaan huonoiten toimivat koodit pois
        Toistamalla vaiheita a ja b N kertaa ei tule mitään järkeviä uusia ohjelmia (rakenteita) vaan pelkkää virhekoodia, vaikka N lähestyisi ääretöntä.

        Samasta syystä jos eliön genomiin tehdään satunnaisia virheitä, ne aiheuttaa pelkästään sairauksia tai jäävät piileviksi (eli eivät aiheuta mitään näkyvää).

        Biologit eivät tätä ymmärrä, koska eivät osaa koodata. Siksi he päätyvät taikauskoiseen ajatteluun että tuo a b toistettuna N kertaa johtaa uusien rakenteiden syntyyn jotenkin taikaiskusta.


      • evlut101 kirjoitti:

        Jokainen joka on tehnyt ohjelmointityötä tietää, että ohjelmaa ei voi kehittää evoluutioprosessilla eli että
        a) luodaan satunnaisia virheitä koodiin
        b) karsitaan huonoiten toimivat koodit pois
        Toistamalla vaiheita a ja b N kertaa ei tule mitään järkeviä uusia ohjelmia (rakenteita) vaan pelkkää virhekoodia, vaikka N lähestyisi ääretöntä.

        Samasta syystä jos eliön genomiin tehdään satunnaisia virheitä, ne aiheuttaa pelkästään sairauksia tai jäävät piileviksi (eli eivät aiheuta mitään näkyvää).

        Biologit eivät tätä ymmärrä, koska eivät osaa koodata. Siksi he päätyvät taikauskoiseen ajatteluun että tuo a b toistettuna N kertaa johtaa uusien rakenteiden syntyyn jotenkin taikaiskusta.

        Jos rinnakkain on asetettava Sinun uskonnollinen julistuksesi ja biologian ammattilaisten tutkimukset, valinta on helppo.

        Ettet vain satu olemaan eräs datanomi, joka täällä vuosia sitten julisti kaikenmoista?


      • Ei.luotu.eikirj
        evlut101 kirjoitti:

        Jokainen joka on tehnyt ohjelmointityötä tietää, että ohjelmaa ei voi kehittää evoluutioprosessilla eli että
        a) luodaan satunnaisia virheitä koodiin
        b) karsitaan huonoiten toimivat koodit pois
        Toistamalla vaiheita a ja b N kertaa ei tule mitään järkeviä uusia ohjelmia (rakenteita) vaan pelkkää virhekoodia, vaikka N lähestyisi ääretöntä.

        Samasta syystä jos eliön genomiin tehdään satunnaisia virheitä, ne aiheuttaa pelkästään sairauksia tai jäävät piileviksi (eli eivät aiheuta mitään näkyvää).

        Biologit eivät tätä ymmärrä, koska eivät osaa koodata. Siksi he päätyvät taikauskoiseen ajatteluun että tuo a b toistettuna N kertaa johtaa uusien rakenteiden syntyyn jotenkin taikaiskusta.

        Jokainen joka tietää jotain ohjelmoinnista ei edes yritä verrata sitä evoluutioon. Täysin eri asiat.


      • lisäävirheitä kirjoitti:

        Sillä tarkoitataan sitä, että jatkuvasti syntyy uusia sairauksia ja löydetään uusia virhegeenejä!!!
        Vaikka hoitomenetelmät ovat parantuneet, niin sairastuvuus ei vähene.

        Syntyykö?

        Linkkiä tälle "tiedolle"?


      • evlut101
        Ei.luotu.eikirj kirjoitti:

        Jokainen joka tietää jotain ohjelmoinnista ei edes yritä verrata sitä evoluutioon. Täysin eri asiat.

        “DNA is like a computer program but far, far more advanced than any software ever created.”
        Bill Gates

        Hänkö ei tiedä mitä on ohjelmointi. Itselläkin muuten ohjelmointikursseja suoritettuna java ja c


      • evlut101
        InhottavaRealisti kirjoitti:

        Jos rinnakkain on asetettava Sinun uskonnollinen julistuksesi ja biologian ammattilaisten tutkimukset, valinta on helppo.

        Ettet vain satu olemaan eräs datanomi, joka täällä vuosia sitten julisti kaikenmoista?

        En ole datanomi. Tohtorin tutkinto löytyy Aalto Yliopistosta.


      • evlut101 kirjoitti:

        Jokainen joka on tehnyt ohjelmointityötä tietää, että ohjelmaa ei voi kehittää evoluutioprosessilla eli että
        a) luodaan satunnaisia virheitä koodiin
        b) karsitaan huonoiten toimivat koodit pois
        Toistamalla vaiheita a ja b N kertaa ei tule mitään järkeviä uusia ohjelmia (rakenteita) vaan pelkkää virhekoodia, vaikka N lähestyisi ääretöntä.

        Samasta syystä jos eliön genomiin tehdään satunnaisia virheitä, ne aiheuttaa pelkästään sairauksia tai jäävät piileviksi (eli eivät aiheuta mitään näkyvää).

        Biologit eivät tätä ymmärrä, koska eivät osaa koodata. Siksi he päätyvät taikauskoiseen ajatteluun että tuo a b toistettuna N kertaa johtaa uusien rakenteiden syntyyn jotenkin taikaiskusta.

        Itseasiassa geneettisiä algoritmejä ja "evoluutiota" on käytetty tiettyjä ongelmia ratkaistaessa jo pitemmän aikaa.

        http://www.talkorigins.org/faqs/genalg/genalg.html


      • TotuusSattuuQC
        evlut101 kirjoitti:

        “DNA is like a computer program but far, far more advanced than any software ever created.”
        Bill Gates

        Hänkö ei tiedä mitä on ohjelmointi. Itselläkin muuten ohjelmointikursseja suoritettuna java ja c

        "“DNA is like a computer program but far, far more advanced than any software ever created.”
        Bill Gates
        Hänkö ei tiedä mitä on ohjelmointi. Itselläkin muuten ohjelmointikursseja suoritettuna java ja c "

        DNA:ta voi sinällään koodina olla kuin tietokoneohjelma mutta se ei kopioidu ollenkaan siten kuten tietokoneohjelma. DNA:n muutos joka sukupolvessa on kuin jos jokaisen uuden tietokoneen tekeminen edellyttäisi että laitetaan kaksi hieman erilaista versiota käyttöjärjestelmästä aina yhteen ja niiden yhdistelmästä syntyisi tätä uutta tietokonetta varten uusi aivan uniikki versio käyttöjärjestelmästä.
        Suvullisessa lisääntymisessä kun syntyy joka ikinen kerta uusi uniikki perimä jälkeläiselle vanhempien perimän ydistelmänä. Mikään tietokoneohjelma taas ei muutu tai synny tällä tavalla. Eikä mikään tietokoneohjelma edes toimisi niin että se yhdistettäisiin satunnaisella tavalla toiseen hieman poikkeavaan versioon samasta ohjelmasta.


      • evlut101
        TotuusSattuuQC kirjoitti:

        "“DNA is like a computer program but far, far more advanced than any software ever created.”
        Bill Gates
        Hänkö ei tiedä mitä on ohjelmointi. Itselläkin muuten ohjelmointikursseja suoritettuna java ja c "

        DNA:ta voi sinällään koodina olla kuin tietokoneohjelma mutta se ei kopioidu ollenkaan siten kuten tietokoneohjelma. DNA:n muutos joka sukupolvessa on kuin jos jokaisen uuden tietokoneen tekeminen edellyttäisi että laitetaan kaksi hieman erilaista versiota käyttöjärjestelmästä aina yhteen ja niiden yhdistelmästä syntyisi tätä uutta tietokonetta varten uusi aivan uniikki versio käyttöjärjestelmästä.
        Suvullisessa lisääntymisessä kun syntyy joka ikinen kerta uusi uniikki perimä jälkeläiselle vanhempien perimän ydistelmänä. Mikään tietokoneohjelma taas ei muutu tai synny tällä tavalla. Eikä mikään tietokoneohjelma edes toimisi niin että se yhdistettäisiin satunnaisella tavalla toiseen hieman poikkeavaan versioon samasta ohjelmasta.

        "uuden tietokoneen tekeminen edellyttäisi että laitetaan kaksi hieman erilaista versiota käyttöjärjestelmästä aina yhteen ja niiden yhdistelmästä syntyisi tätä uutta tietokonetta varten uusi aivan uniikki versio käyttöjärjestelmästä.
        Suvullisessa lisääntymisessä kun syntyy joka ikinen kerta uusi uniikki perimä jälkeläiselle vanhempien perimän ydistelmänä. Mikään tietokoneohjelma taas ei muutu tai synny tällä tavalla. "

        Ihan hyvä pointti, mutta mietipä vähän että miten se uusi informaatio tässä syntyy? Sinähän vain sekoitat tuossa kahta olemassaolevaa. Ja jos prosessissa on jotain random-efektejä, niin randomaalisten virheiden lisääminen koodiin simuloisi samaa asiaa.

        Toisekseen luonnossa on paljon suvuttomasti lisääntyviä (esim. bakteerit). Eikö niille sitten tapahdu evoluutiota lainkaan?


      • evlut101
        Oudoksuva kirjoitti:

        Itseasiassa geneettisiä algoritmejä ja "evoluutiota" on käytetty tiettyjä ongelmia ratkaistaessa jo pitemmän aikaa.

        http://www.talkorigins.org/faqs/genalg/genalg.html

        "Itseasiassa geneettisiä algoritmejä ja "evoluutiota" on käytetty tiettyjä ongelmia ratkaistaessa jo pitemmän aikaa. "

        Hyvä pointti... Geneettisiä algoritmejä käytetään eniten matemaattisten inversio-ongelmien ratkaisussa esimerkiksi datan sovituksessa.

        Jokainen joka on käyttänyt niitä, tietää että GA päätyy hyvin herkästi paikalliseen minimiin eikä koskaan pääse sieltä pois. Näinollen jos GA simuloisi evoluutiota, se olisi hakenut yhden paikallisen minimin (optimaalisen eliön) ja jämähtänyt siihen.

        GA on muutenkin raaka matemaattinen yksinkertaistus, joka eroaa aivan merkittävästi evoluutiosta. Esimerkiksi genomissa on tyypillisesti yli miljardi muuttujaa. GA ongelmissa tyypillisesti alle kymmenen. Yksi GA sykli voidaan tehdä mikrosekunneissa (vertaa luonnossa vähintään kymmeniä minuutteja tai vuosia). Lisäksi GA pystyy asettamaan oliot matemaattisen kaavan kautta hyvyysjärjestykseen kun taas luonnonvalinta ei ole läheskään yhtä tarkka. Esimerkkinä puun siemenet. Osa siemenistä putoaa kiven päälle, toinen kuivaan hiekkaan jne... PIeni osa siemenistä putoaa hedelmälliseen maahan. Onko se siemen muka geneettisesti muita kyvykkäämpi vai kävikö sillä vain säkä.


      • evlut101 kirjoitti:

        "Itseasiassa geneettisiä algoritmejä ja "evoluutiota" on käytetty tiettyjä ongelmia ratkaistaessa jo pitemmän aikaa. "

        Hyvä pointti... Geneettisiä algoritmejä käytetään eniten matemaattisten inversio-ongelmien ratkaisussa esimerkiksi datan sovituksessa.

        Jokainen joka on käyttänyt niitä, tietää että GA päätyy hyvin herkästi paikalliseen minimiin eikä koskaan pääse sieltä pois. Näinollen jos GA simuloisi evoluutiota, se olisi hakenut yhden paikallisen minimin (optimaalisen eliön) ja jämähtänyt siihen.

        GA on muutenkin raaka matemaattinen yksinkertaistus, joka eroaa aivan merkittävästi evoluutiosta. Esimerkiksi genomissa on tyypillisesti yli miljardi muuttujaa. GA ongelmissa tyypillisesti alle kymmenen. Yksi GA sykli voidaan tehdä mikrosekunneissa (vertaa luonnossa vähintään kymmeniä minuutteja tai vuosia). Lisäksi GA pystyy asettamaan oliot matemaattisen kaavan kautta hyvyysjärjestykseen kun taas luonnonvalinta ei ole läheskään yhtä tarkka. Esimerkkinä puun siemenet. Osa siemenistä putoaa kiven päälle, toinen kuivaan hiekkaan jne... PIeni osa siemenistä putoaa hedelmälliseen maahan. Onko se siemen muka geneettisesti muita kyvykkäämpi vai kävikö sillä vain säkä.

        "Jokainen joka on käyttänyt niitä, tietää että GA päätyy hyvin herkästi paikalliseen minimiin eikä koskaan pääse sieltä pois. Näinollen jos GA simuloisi evoluutiota, se olisi hakenut yhden paikallisen minimin (optimaalisen eliön) ja jämähtänyt siihen."

        Se ero tässä ja oikean elämän evoluutiossa on lähinnä monimutkaisuus koska olosuhteet ja muut kilpailevat lajit aiheuttavat liian kovan selviytymispaineen että voisi syntyä pysyvä minimi. Eli siinä vaiheessa kun laji on kehittynyt "täydellisesti sopivaksi" (esimerkkisi minimi) ovat olosuhteet ja muut lajit myös kehittynyt tarpeeksi ettei se enää riitä ja tarvitsee muuttua kilpailua varten :)


      • evlut101 kirjoitti:

        "Itseasiassa geneettisiä algoritmejä ja "evoluutiota" on käytetty tiettyjä ongelmia ratkaistaessa jo pitemmän aikaa. "

        Hyvä pointti... Geneettisiä algoritmejä käytetään eniten matemaattisten inversio-ongelmien ratkaisussa esimerkiksi datan sovituksessa.

        Jokainen joka on käyttänyt niitä, tietää että GA päätyy hyvin herkästi paikalliseen minimiin eikä koskaan pääse sieltä pois. Näinollen jos GA simuloisi evoluutiota, se olisi hakenut yhden paikallisen minimin (optimaalisen eliön) ja jämähtänyt siihen.

        GA on muutenkin raaka matemaattinen yksinkertaistus, joka eroaa aivan merkittävästi evoluutiosta. Esimerkiksi genomissa on tyypillisesti yli miljardi muuttujaa. GA ongelmissa tyypillisesti alle kymmenen. Yksi GA sykli voidaan tehdä mikrosekunneissa (vertaa luonnossa vähintään kymmeniä minuutteja tai vuosia). Lisäksi GA pystyy asettamaan oliot matemaattisen kaavan kautta hyvyysjärjestykseen kun taas luonnonvalinta ei ole läheskään yhtä tarkka. Esimerkkinä puun siemenet. Osa siemenistä putoaa kiven päälle, toinen kuivaan hiekkaan jne... PIeni osa siemenistä putoaa hedelmälliseen maahan. Onko se siemen muka geneettisesti muita kyvykkäämpi vai kävikö sillä vain säkä.

        "Lisäksi GA pystyy asettamaan oliot matemaattisen kaavan kautta hyvyysjärjestykseen kun taas luonnonvalinta ei ole läheskään yhtä tarkka. Esimerkkinä puun siemenet. Osa siemenistä putoaa kiven päälle, toinen kuivaan hiekkaan jne... PIeni osa siemenistä putoaa hedelmälliseen maahan. Onko se siemen muka geneettisesti muita kyvykkäämpi vai kävikö sillä vain säkä."

        Vastasit tähän juuri omassa viestissäsi muuten :)

        Se syy miksi sillä ei ole väliä jos osa siemenistä putoaa "väärään paikkaan" on juuri sen vuoksi koska yhden sukupolven aikana ei ole niin paljon muutoksia (suuremmissa lajeissa. Bakteerit ovat asia erikseen koska niillä ei ole genomin virheenkorjausta samalla tavalla kuin kasveilla ja nisäkkäillä joten ne luottavat mutaatioihin niin että aina vähintään yksi selviytyy)

        Hyvänä esimerkkinä oheinen video Harvardista jossa otettiin videolle bakteerien evoluutiota :)
        https://www.youtube.com/watch?v=plVk4NVIUh8


      • evlut101
        Oudoksuva kirjoitti:

        "Lisäksi GA pystyy asettamaan oliot matemaattisen kaavan kautta hyvyysjärjestykseen kun taas luonnonvalinta ei ole läheskään yhtä tarkka. Esimerkkinä puun siemenet. Osa siemenistä putoaa kiven päälle, toinen kuivaan hiekkaan jne... PIeni osa siemenistä putoaa hedelmälliseen maahan. Onko se siemen muka geneettisesti muita kyvykkäämpi vai kävikö sillä vain säkä."

        Vastasit tähän juuri omassa viestissäsi muuten :)

        Se syy miksi sillä ei ole väliä jos osa siemenistä putoaa "väärään paikkaan" on juuri sen vuoksi koska yhden sukupolven aikana ei ole niin paljon muutoksia (suuremmissa lajeissa. Bakteerit ovat asia erikseen koska niillä ei ole genomin virheenkorjausta samalla tavalla kuin kasveilla ja nisäkkäillä joten ne luottavat mutaatioihin niin että aina vähintään yksi selviytyy)

        Hyvänä esimerkkinä oheinen video Harvardista jossa otettiin videolle bakteerien evoluutiota :)
        https://www.youtube.com/watch?v=plVk4NVIUh8

        "Se syy miksi sillä ei ole väliä jos osa siemenistä putoaa "väärään paikkaan" on juuri sen vuoksi koska yhden sukupolven aikana ei ole niin paljon muutoksia (suuremmissa lajeissa. Bakteerit ovat asia erikseen koska niillä ei ole genomin virheenkorjausta samalla tavalla kuin kasveilla ja nisäkkäillä joten ne luottavat mutaatioihin niin että aina vähintään yksi selviytyy)"

        Kyllä mutta eikö tämä ole osoitus siitä, että luonnonvalinta ei toimi tehokkaasti vaan haaskaa 99 % alkioista randomilla. Samanlaisia esimerkkejä löytyy läjäpäin. Sammakonkudusta syödään reunimmaiset ensin (onko reunimmaiset geneettisesti huonompia kuin keskimmäiset). Suuremmaksi selviytynyt sammakko sattuu kävelemään linnun kohdalle ja joutuu syödyksi.... Toisessa paikassa ollut sammakko selviytyy, vaikkei se ollut geneettisesti syötyä elinvoimaisempi. Sillä vaan kävi parempi mäihä.

        "Hyvänä esimerkkinä oheinen video Harvardista jossa otettiin videolle bakteerien evoluutiota :)
        https://www.youtube.com/watch?v=plVk4NVIUh8"

        Jes nää on parhaita samoin kuin koirarotuesimerkki. Evoluutioprosessin katsotaan kestävän kymmeniä tuhansia sukupolvia ennen kuin merkittäviä muutoksia voidaan havaita. Tämä johtuu siitä että evoluutioprosessi on a) satunnaisia mutaatioita ja b) luonnonvalintaa. Nyt näissä kokeissa kohdalle a ei löydy läheskään tarpeeksi aikaa ja tehdään pelkästään kohta b. Tämä itse asiassa todistaa vain että bakteerikasvustolla oli jo vastustuskykyisiä yksilöitä, jotka sitten nousivat kukoistamaan kun ei-vastustuskykyinen massa ei selvinnyt.


      • lisäävirheitä kirjoitti:

        Sillä tarkoitataan sitä, että jatkuvasti syntyy uusia sairauksia ja löydetään uusia virhegeenejä!!!
        Vaikka hoitomenetelmät ovat parantuneet, niin sairastuvuus ei vähene.

        Eli ihminen joutaa kuolla taas sukupuuttoon.


      • lisäävirheitä kirjoitti:

        Sillä tarkoitataan sitä, että jatkuvasti syntyy uusia sairauksia ja löydetään uusia virhegeenejä!!!
        Vaikka hoitomenetelmät ovat parantuneet, niin sairastuvuus ei vähene.

        ”Sillä tarkoitataan sitä, että jatkuvasti syntyy uusia sairauksia ja löydetään uusia virhegeenejä!!!”

        Tuota, se että tiede on niin kehittynyt, että tosiaan löydetään entistä paremmin eri taudinaiheuttajia, ei oikein minusta kerro mitenkään rappiosta?!!


      • evlut101 kirjoitti:

        "Se syy miksi sillä ei ole väliä jos osa siemenistä putoaa "väärään paikkaan" on juuri sen vuoksi koska yhden sukupolven aikana ei ole niin paljon muutoksia (suuremmissa lajeissa. Bakteerit ovat asia erikseen koska niillä ei ole genomin virheenkorjausta samalla tavalla kuin kasveilla ja nisäkkäillä joten ne luottavat mutaatioihin niin että aina vähintään yksi selviytyy)"

        Kyllä mutta eikö tämä ole osoitus siitä, että luonnonvalinta ei toimi tehokkaasti vaan haaskaa 99 % alkioista randomilla. Samanlaisia esimerkkejä löytyy läjäpäin. Sammakonkudusta syödään reunimmaiset ensin (onko reunimmaiset geneettisesti huonompia kuin keskimmäiset). Suuremmaksi selviytynyt sammakko sattuu kävelemään linnun kohdalle ja joutuu syödyksi.... Toisessa paikassa ollut sammakko selviytyy, vaikkei se ollut geneettisesti syötyä elinvoimaisempi. Sillä vaan kävi parempi mäihä.

        "Hyvänä esimerkkinä oheinen video Harvardista jossa otettiin videolle bakteerien evoluutiota :)
        https://www.youtube.com/watch?v=plVk4NVIUh8"

        Jes nää on parhaita samoin kuin koirarotuesimerkki. Evoluutioprosessin katsotaan kestävän kymmeniä tuhansia sukupolvia ennen kuin merkittäviä muutoksia voidaan havaita. Tämä johtuu siitä että evoluutioprosessi on a) satunnaisia mutaatioita ja b) luonnonvalintaa. Nyt näissä kokeissa kohdalle a ei löydy läheskään tarpeeksi aikaa ja tehdään pelkästään kohta b. Tämä itse asiassa todistaa vain että bakteerikasvustolla oli jo vastustuskykyisiä yksilöitä, jotka sitten nousivat kukoistamaan kun ei-vastustuskykyinen massa ei selvinnyt.

        "Kyllä mutta eikö tämä ole osoitus siitä, että luonnonvalinta ei toimi tehokkaasti vaan haaskaa 99 % alkioista randomilla. Samanlaisia esimerkkejä löytyy läjäpäin. Sammakonkudusta syödään reunimmaiset ensin (onko reunimmaiset geneettisesti huonompia kuin keskimmäiset). Suuremmaksi selviytynyt sammakko sattuu kävelemään linnun kohdalle ja joutuu syödyksi.... Toisessa paikassa ollut sammakko selviytyy, vaikkei se ollut geneettisesti syötyä elinvoimaisempi. Sillä vaan kävi parempi mäihä."

        Mitä tarkoitat tässä "tehokkaalla"? Luonnonvalinta ei suoranaisesti välitä siitä miten suuri osa alkioista selviytyy vaan enemmän lajin selviytymiseen vaikuttavista tekijöistä. Eli se jos ainakin yksi lukemattomista alkioista selviytyy on parempi kuin riski että on vain yksi alkio joka saattaa selviytyä tai sitten ei. Sen vuoksi nisäkkäät on eläimissä poikkeuksellisia koska niillä on verrattain vähän poikasia mutta tämän suojelemiseksi esimerkiksi alussa poikanen kasvaa naaraan sisällä turvassa.

        Kyseessä on lähinnä vain kaksi eri strategiaa jotka molemmat auttavat paineeseen että lajin jäseniä selviytyy ja jatkaa.

        "Jes nää on parhaita samoin kuin koirarotuesimerkki. Evoluutioprosessin katsotaan kestävän kymmeniä tuhansia sukupolvia ennen kuin merkittäviä muutoksia voidaan havaita. Tämä johtuu siitä että evoluutioprosessi on a) satunnaisia mutaatioita ja b) luonnonvalintaa. Nyt näissä kokeissa kohdalle a ei löydy läheskään tarpeeksi aikaa ja tehdään pelkästään kohta b. Tämä itse asiassa todistaa vain että bakteerikasvustolla oli jo vastustuskykyisiä yksilöitä, jotka sitten nousivat kukoistamaan kun ei-vastustuskykyinen massa ei selvinnyt."

        Evoluutio on prosessi joka on koko ajan toiminnassa, ei sellainen joka tapahtuu vain tuhansissa sukupolvissa. Myös se aika ennen kuin muutokset tulee ilmiselviksi riippuu ihan lajista, kuten tässä esimerkissäni bakteereilla on tarpeeksi nopea mutaatio ja lisääntymisnopeus että se vaatii verrattain vähän aikaa että evoluution vaikutus ja luonnonvalinta tiettyä standardisoitua parametria vastaan voidaan mitata ja nähdä. :)

        Videossa myös näkyy että bakteerikasvustolla ei ollut aluksi vastustuskykyisiä yksilöitä koska kesti aikaa alueiden rajoilla ennen kuin alkoi syntymään mutaatioita jotka kykenivät selviytymään ensimmäisessä antibioottia sisältävässä alueessa. Mielenkiintoisena pointtina, jotkin mutaatiot (mutteivät kaikki) kykenivät myös selviytymään melkein yhtä hyvin korkeammassa antibioottipitoisuudessa (10x) ja siten sai aluksi etumatkaa mutta sitten kun pitoisuus meni tietyn rajan yläpuolella, se oli toinen mutaatio joka paljastui paremmin tilanteeseen soveltuvaksi ja ohitti ensimmäisen.

        Tämä on täydellinen esimerkki siitä miten evoluutio toimii luonnossakin, käyttäen esimerkkinä standardisoituja olosuhteita jotta voidaan mitata evoluution vaikutus suhteessa tähän parhaiten :)


      • mummomuori kirjoitti:

        ”Sillä tarkoitataan sitä, että jatkuvasti syntyy uusia sairauksia ja löydetään uusia virhegeenejä!!!”

        Tuota, se että tiede on niin kehittynyt, että tosiaan löydetään entistä paremmin eri taudinaiheuttajia, ei oikein minusta kerro mitenkään rappiosta?!!

        Huvittavaa kyllä, jo yksittäiset esimerkit esim. mutaatio joka antoi suurimmalle osalle ihmisistä kyvyn juoda maitoa täysikasvuisenakin kykenevät todistamaan sitä vastaan että eliöstö rappeutuu mutaatioista :)


      • evlut101
        Oudoksuva kirjoitti:

        "Kyllä mutta eikö tämä ole osoitus siitä, että luonnonvalinta ei toimi tehokkaasti vaan haaskaa 99 % alkioista randomilla. Samanlaisia esimerkkejä löytyy läjäpäin. Sammakonkudusta syödään reunimmaiset ensin (onko reunimmaiset geneettisesti huonompia kuin keskimmäiset). Suuremmaksi selviytynyt sammakko sattuu kävelemään linnun kohdalle ja joutuu syödyksi.... Toisessa paikassa ollut sammakko selviytyy, vaikkei se ollut geneettisesti syötyä elinvoimaisempi. Sillä vaan kävi parempi mäihä."

        Mitä tarkoitat tässä "tehokkaalla"? Luonnonvalinta ei suoranaisesti välitä siitä miten suuri osa alkioista selviytyy vaan enemmän lajin selviytymiseen vaikuttavista tekijöistä. Eli se jos ainakin yksi lukemattomista alkioista selviytyy on parempi kuin riski että on vain yksi alkio joka saattaa selviytyä tai sitten ei. Sen vuoksi nisäkkäät on eläimissä poikkeuksellisia koska niillä on verrattain vähän poikasia mutta tämän suojelemiseksi esimerkiksi alussa poikanen kasvaa naaraan sisällä turvassa.

        Kyseessä on lähinnä vain kaksi eri strategiaa jotka molemmat auttavat paineeseen että lajin jäseniä selviytyy ja jatkaa.

        "Jes nää on parhaita samoin kuin koirarotuesimerkki. Evoluutioprosessin katsotaan kestävän kymmeniä tuhansia sukupolvia ennen kuin merkittäviä muutoksia voidaan havaita. Tämä johtuu siitä että evoluutioprosessi on a) satunnaisia mutaatioita ja b) luonnonvalintaa. Nyt näissä kokeissa kohdalle a ei löydy läheskään tarpeeksi aikaa ja tehdään pelkästään kohta b. Tämä itse asiassa todistaa vain että bakteerikasvustolla oli jo vastustuskykyisiä yksilöitä, jotka sitten nousivat kukoistamaan kun ei-vastustuskykyinen massa ei selvinnyt."

        Evoluutio on prosessi joka on koko ajan toiminnassa, ei sellainen joka tapahtuu vain tuhansissa sukupolvissa. Myös se aika ennen kuin muutokset tulee ilmiselviksi riippuu ihan lajista, kuten tässä esimerkissäni bakteereilla on tarpeeksi nopea mutaatio ja lisääntymisnopeus että se vaatii verrattain vähän aikaa että evoluution vaikutus ja luonnonvalinta tiettyä standardisoitua parametria vastaan voidaan mitata ja nähdä. :)

        Videossa myös näkyy että bakteerikasvustolla ei ollut aluksi vastustuskykyisiä yksilöitä koska kesti aikaa alueiden rajoilla ennen kuin alkoi syntymään mutaatioita jotka kykenivät selviytymään ensimmäisessä antibioottia sisältävässä alueessa. Mielenkiintoisena pointtina, jotkin mutaatiot (mutteivät kaikki) kykenivät myös selviytymään melkein yhtä hyvin korkeammassa antibioottipitoisuudessa (10x) ja siten sai aluksi etumatkaa mutta sitten kun pitoisuus meni tietyn rajan yläpuolella, se oli toinen mutaatio joka paljastui paremmin tilanteeseen soveltuvaksi ja ohitti ensimmäisen.

        Tämä on täydellinen esimerkki siitä miten evoluutio toimii luonnossakin, käyttäen esimerkkinä standardisoituja olosuhteita jotta voidaan mitata evoluution vaikutus suhteessa tähän parhaiten :)

        "Tämä on täydellinen esimerkki siitä miten evoluutio toimii luonnossakin, käyttäen esimerkkinä standardisoituja olosuhteita jotta voidaan mitata evoluution vaikutus suhteessa tähän parhaiten :) "

        Kun evoluutio muodostaa rakenteita, sen täytyy tehdä useita mutaatioita elämän koodiin. Esimerkiksi puhekyvyn rakentuminen vaatii valtavat määrät mutaatioita. Jos lähdetään siitä että genomiin tulee puhekyvyttömälle alkuihmiselle satunnaisia virheitä, joiden seurauksena hän saa puhekyvyn ja kyvyn ymmärtää puhetta, niin kuinka monta satunnaismutaatiota tarvitaan? Vastaus on vähintään miljoonia. No jos taas lasketaan todennäköisyyttä sille että miten nämä miljoonat mutaatiot syntyivät sattumalta, niin ollan täysin mahdottoman tapahtuman edellä. Siis vähän kun otettaisiin windows käyttöjärjestelmä ja heitettäisiin sinne miljoona bittivirhettä ja oletettaisiin, että siihen tulisi sattumalta uusia ominaisuuksia eikä toiminnallisuus kärsisi.

        Laskennallisesti yksi pistemutaatio on todennäköinen (tapaus antibiootit tai malarialoinen). Kahden pistemutaation polku on jo erittäin todennäköinen, mutta raportoitu malarialoisella. Se tuli vastustuskykyiseksi malarialääkkeelle. Mutta miljoonien pistemutaatioiden kulku on paitsi laskennallisesti mahdoton, sitä ei myöskään tue mikään kokeellinen data. Eli monimutkaisten rakenteiden kuten bakteerimoottori tai puhekyky, kehittyminen evoluutioprosessin kautta on teoriassa ja kokeiden kautta osoitettu mahdottomaksi. Se on pelkkää todennäköisyyslaskentaa. Pitäisi tapahtua valtava määrä mutaatioita, jotka rakentavat monimutkaista rakennetta itsestään ilman että yksilön sopeutumiskyky siinä välissä paranee lainkaan. Bakteerien antibioottiresistenssikokeet eivät anna tähän mitään apua, koska niissä kyse on pelkästään yhden tai max kahden pistemutaation polusta.


      • evlut101 kirjoitti:

        "Tämä on täydellinen esimerkki siitä miten evoluutio toimii luonnossakin, käyttäen esimerkkinä standardisoituja olosuhteita jotta voidaan mitata evoluution vaikutus suhteessa tähän parhaiten :) "

        Kun evoluutio muodostaa rakenteita, sen täytyy tehdä useita mutaatioita elämän koodiin. Esimerkiksi puhekyvyn rakentuminen vaatii valtavat määrät mutaatioita. Jos lähdetään siitä että genomiin tulee puhekyvyttömälle alkuihmiselle satunnaisia virheitä, joiden seurauksena hän saa puhekyvyn ja kyvyn ymmärtää puhetta, niin kuinka monta satunnaismutaatiota tarvitaan? Vastaus on vähintään miljoonia. No jos taas lasketaan todennäköisyyttä sille että miten nämä miljoonat mutaatiot syntyivät sattumalta, niin ollan täysin mahdottoman tapahtuman edellä. Siis vähän kun otettaisiin windows käyttöjärjestelmä ja heitettäisiin sinne miljoona bittivirhettä ja oletettaisiin, että siihen tulisi sattumalta uusia ominaisuuksia eikä toiminnallisuus kärsisi.

        Laskennallisesti yksi pistemutaatio on todennäköinen (tapaus antibiootit tai malarialoinen). Kahden pistemutaation polku on jo erittäin todennäköinen, mutta raportoitu malarialoisella. Se tuli vastustuskykyiseksi malarialääkkeelle. Mutta miljoonien pistemutaatioiden kulku on paitsi laskennallisesti mahdoton, sitä ei myöskään tue mikään kokeellinen data. Eli monimutkaisten rakenteiden kuten bakteerimoottori tai puhekyky, kehittyminen evoluutioprosessin kautta on teoriassa ja kokeiden kautta osoitettu mahdottomaksi. Se on pelkkää todennäköisyyslaskentaa. Pitäisi tapahtua valtava määrä mutaatioita, jotka rakentavat monimutkaista rakennetta itsestään ilman että yksilön sopeutumiskyky siinä välissä paranee lainkaan. Bakteerien antibioottiresistenssikokeet eivät anna tähän mitään apua, koska niissä kyse on pelkästään yhden tai max kahden pistemutaation polusta.

        Oletatko Sinä ihan vakavissasi olevasi pätevä arviomaan noita juttuja? Pätevämpi kuiin maailman biologien ja evoluutiospesialistien murskaava enemmistö?


      • capiche
        Msmslsl kirjoitti:

        Geneetikko John Sanford: Ihmiskunta rappeutuu pikavauhtia:

        ”Meidän sukupolvellamme on keskimäärin 100-200 mutaatiota enemmän kuin vanhemmillamme, ja heillä on niitä saman verran enemmän kuin heidän vanhemmillaan. Tämä johtuu siitä, että joka kerta, kun DNA:ta välittyy sukupolvelta toiselle, siihen ilmestyy lisää mutaatioita.

        Mutaatiot ovat enimmäkseen vahingollisia. Ne ovat geneettisiä kirjoitusvirheitä, mutta darvinismin mukaan ne ovat yksi evoluution kivijaloista, ja niiden pitäisi miljoonien vuosien saatossa parantaa lajin elinkelpoisuutta.

        Tämä kivijalka rapistuu kuitenkin melko nopeasti. Kehitys kulkee väärään suuntaan.

        Cornellin yliopiston professori John Sanford luennoi aiheesta hiljattain Darwin Was Wrong -konferenssissa. Geneetikko Sanford kertoi, että darvinistit ovat aliarvioineet vahingollisten mutaatioiden määrää, koska he eivät ole laskeneet ”roska-DNA:n” muutoksia. Näin ollen heiltä on jäänyt huomioimatta 90 prosenttia mutaatioista.

        Professori Sanford ei ole ainoa, joka sanoo ihmisten rappeutuvan. Myös Nature News vahvisti hiljattain hänen havaintonsa. Tulos kävi ilmi, kun tutkijat sekvensoivat Y-kromosomia.

        Tästä voitaneen päätellä, että mitä enemmän aikaa kuluu, sitä pahemmin ihmiskunta rappeutuu. Evoluutio kulkee siis täysin väärään suuntaan. Vuosimiljoonat merkitsisivät sukupuuttoa.

        Myös lääkäri Pekka Reinikainen ja biokemisti Pauli Ojala ovat ehdottaneet, että näemme evoluution sijasta devoluutiota eli kehityksen asemesta rappeutumista.

        Lähteet:

        Dolgin, Elie. 2009. Human mutation rate revealed. Nature News (27.8.). http://www.nature.com/news/2009/090827/full/news.2...

        Sanford, John. 2009. Darwin Was Wrong about Natural Selection. Luento Darwin Was Wrong konferenssissa (14.11.)

        http://bwanajoe.blogspot.com/2009/11/geneetikko-jo...

        Uusi tutkimus näyttää, etta rappeutuminen keräytyy geenibooliin:

        "Take, to begin with, the Swedish chickens. Three years ago, researchers led by a professor at the university of Linköping in Sweden created a henhouse that was specially designed to make its chicken occupants feel stressed. The lighting was manipulated to make the rhythms of night and day unpredictable, so the chickens lost track of when to eat or roost. Unsurprisingly, perhaps, they showed a significant decrease in their ability to learn how to find food hidden in a maze.

        The surprising part is what happened next: the chickens were moved back to a non-stressful environment, where they conceived and hatched chicks who were raised without stress - and yet these chicks, too, demonstrated unexpectedly poor skills at finding food in a maze. They appeared to have inherited a problem that had been induced in their mothers through the environment.

        Further research established that the inherited change had altered the
        chicks' "gene expression" - the way certain genes are turned "on" or "off", bestowing any given animal with specific traits. The stress had affected the mother hens on a genetic level, and they had passed it on to their offspring."

        Eli ongelma oli periytynyt äidiltä lapselle päinvastoin kuin evoteoria ennustaa.

        Eli kanat eivät kehittyneet ympäristön mukaan, vaan päinvastoin heidän geeninsä muuttuivat huonommaksi uuden ympäristön mukaan! Eli kävi päinvastoin kuin mitä evoluutioteoria ennustaa...

        http://www.guardian.co.uk/science/2010/mar/19/evol...

        Biologisen informaation mitattavuus:

        ”Nykyään elää monia lentokyvyttömiä lintulajeja, joille siivillä ei ole sama merkitys kuin niiden lentäville sukulaisille. Evolutionistit pitävät siipiä surkastumina tai keskeneräisinä rakenteina.

        Mutaatiothan ovat joko spontaaneja sisäsyntyisiä tapahtumia tai ulkoisen mutageenin aiheuttamia.

        Teollinen kausi lisäsi takuuvarmasti ihmiskunnan altistusta ulkoisille mutageeneille. Niitähän löytyy mm. mineraaleista tai ihmisen kehittämistä kemikaaleista. Ihmisruumiissa on nykyään kemikaalikertymiä, joita Luoja ei ole sinne tarkoittanut. Eikä elinaikana saadun kemikaalialtistuksen yhteisvaikutuksia ei edes tunneta.

        Lääketieteen kehitys on ihmiskunnan kehityksen kannalta myös kaksiteräinen miekka. Se helpottaa yksilön elämää ja lisää elinajan odotetta. Mutta samalla se myös mahdollistaa myös geneettisten sairauksien ja geenivirheiden siirtymisen jälkeläisille. Mikä osaltaan edesauttaa ihmisen geeniperimän rappeutumista.


      • capiche kirjoitti:

        Mutaatiothan ovat joko spontaaneja sisäsyntyisiä tapahtumia tai ulkoisen mutageenin aiheuttamia.

        Teollinen kausi lisäsi takuuvarmasti ihmiskunnan altistusta ulkoisille mutageeneille. Niitähän löytyy mm. mineraaleista tai ihmisen kehittämistä kemikaaleista. Ihmisruumiissa on nykyään kemikaalikertymiä, joita Luoja ei ole sinne tarkoittanut. Eikä elinaikana saadun kemikaalialtistuksen yhteisvaikutuksia ei edes tunneta.

        Lääketieteen kehitys on ihmiskunnan kehityksen kannalta myös kaksiteräinen miekka. Se helpottaa yksilön elämää ja lisää elinajan odotetta. Mutta samalla se myös mahdollistaa myös geneettisten sairauksien ja geenivirheiden siirtymisen jälkeläisille. Mikä osaltaan edesauttaa ihmisen geeniperimän rappeutumista.

        Muista nyt kuitenkin se, että tästä väitetystä rappeutumisesta ei ole tosiasiassa YHTÄÄN mitään sellaista näyttöä, joka kestäisi tarkastelun muualla kuin fundamentalistien telttakokouksissa!


      • Msmslsl kirjoitti:

        Geneetikko John Sanford: Ihmiskunta rappeutuu pikavauhtia:

        ”Meidän sukupolvellamme on keskimäärin 100-200 mutaatiota enemmän kuin vanhemmillamme, ja heillä on niitä saman verran enemmän kuin heidän vanhemmillaan. Tämä johtuu siitä, että joka kerta, kun DNA:ta välittyy sukupolvelta toiselle, siihen ilmestyy lisää mutaatioita.

        Mutaatiot ovat enimmäkseen vahingollisia. Ne ovat geneettisiä kirjoitusvirheitä, mutta darvinismin mukaan ne ovat yksi evoluution kivijaloista, ja niiden pitäisi miljoonien vuosien saatossa parantaa lajin elinkelpoisuutta.

        Tämä kivijalka rapistuu kuitenkin melko nopeasti. Kehitys kulkee väärään suuntaan.

        Cornellin yliopiston professori John Sanford luennoi aiheesta hiljattain Darwin Was Wrong -konferenssissa. Geneetikko Sanford kertoi, että darvinistit ovat aliarvioineet vahingollisten mutaatioiden määrää, koska he eivät ole laskeneet ”roska-DNA:n” muutoksia. Näin ollen heiltä on jäänyt huomioimatta 90 prosenttia mutaatioista.

        Professori Sanford ei ole ainoa, joka sanoo ihmisten rappeutuvan. Myös Nature News vahvisti hiljattain hänen havaintonsa. Tulos kävi ilmi, kun tutkijat sekvensoivat Y-kromosomia.

        Tästä voitaneen päätellä, että mitä enemmän aikaa kuluu, sitä pahemmin ihmiskunta rappeutuu. Evoluutio kulkee siis täysin väärään suuntaan. Vuosimiljoonat merkitsisivät sukupuuttoa.

        Myös lääkäri Pekka Reinikainen ja biokemisti Pauli Ojala ovat ehdottaneet, että näemme evoluution sijasta devoluutiota eli kehityksen asemesta rappeutumista.

        Lähteet:

        Dolgin, Elie. 2009. Human mutation rate revealed. Nature News (27.8.). http://www.nature.com/news/2009/090827/full/news.2...

        Sanford, John. 2009. Darwin Was Wrong about Natural Selection. Luento Darwin Was Wrong konferenssissa (14.11.)

        http://bwanajoe.blogspot.com/2009/11/geneetikko-jo...

        Uusi tutkimus näyttää, etta rappeutuminen keräytyy geenibooliin:

        "Take, to begin with, the Swedish chickens. Three years ago, researchers led by a professor at the university of Linköping in Sweden created a henhouse that was specially designed to make its chicken occupants feel stressed. The lighting was manipulated to make the rhythms of night and day unpredictable, so the chickens lost track of when to eat or roost. Unsurprisingly, perhaps, they showed a significant decrease in their ability to learn how to find food hidden in a maze.

        The surprising part is what happened next: the chickens were moved back to a non-stressful environment, where they conceived and hatched chicks who were raised without stress - and yet these chicks, too, demonstrated unexpectedly poor skills at finding food in a maze. They appeared to have inherited a problem that had been induced in their mothers through the environment.

        Further research established that the inherited change had altered the
        chicks' "gene expression" - the way certain genes are turned "on" or "off", bestowing any given animal with specific traits. The stress had affected the mother hens on a genetic level, and they had passed it on to their offspring."

        Eli ongelma oli periytynyt äidiltä lapselle päinvastoin kuin evoteoria ennustaa.

        Eli kanat eivät kehittyneet ympäristön mukaan, vaan päinvastoin heidän geeninsä muuttuivat huonommaksi uuden ympäristön mukaan! Eli kävi päinvastoin kuin mitä evoluutioteoria ennustaa...

        http://www.guardian.co.uk/science/2010/mar/19/evol...

        Biologisen informaation mitattavuus:

        ”Nykyään elää monia lentokyvyttömiä lintulajeja, joille siivillä ei ole sama merkitys kuin niiden lentäville sukulaisille. Evolutionistit pitävät siipiä surkastumina tai keskeneräisinä rakenteina.

        Sanfordin ym. tutkimukset ovat avanneet aivan uuden lehden. Koska perimä rappeutuu, niin emme ole voineet kehittyä apinasta - olisimme alkeellisempia kuin apinat - noissa miljoonissa vuosissa olisimme kuolleet sukupuuttoon jo moneen kertaan.


      • capiche
        Yksi_usko kirjoitti:

        Sanfordin ym. tutkimukset ovat avanneet aivan uuden lehden. Koska perimä rappeutuu, niin emme ole voineet kehittyä apinasta - olisimme alkeellisempia kuin apinat - noissa miljoonissa vuosissa olisimme kuolleet sukupuuttoon jo moneen kertaan.

        Perimää "rappeuttavia" tekijöitä oli todennäköisesti vähemmän kun elettiin lähempänä luontoa. Ja ne lisääntyivät exponentiaalisesti teollisen kehityksen mukana.

        Esim. tietyt syövät ovat lisääntyneet räjähdysmäisesti maailmassa sitä mukaa kuin ympäristön karsinogeenitkin ovat lisääntyneet. Ja autoimmuunisairaudet ovat tuplaantuneet noin 20 vuodessa. Elinvauriot moninkertaistuneet. Ja valtava määrä ihmisiä elää lääkkeiden avulla parantumattomien pitkäaikaissairauksien kanssa.


      • EiHelvetti
        evlut101 kirjoitti:

        Jokainen joka on tehnyt ohjelmointityötä tietää, että ohjelmaa ei voi kehittää evoluutioprosessilla eli että
        a) luodaan satunnaisia virheitä koodiin
        b) karsitaan huonoiten toimivat koodit pois
        Toistamalla vaiheita a ja b N kertaa ei tule mitään järkeviä uusia ohjelmia (rakenteita) vaan pelkkää virhekoodia, vaikka N lähestyisi ääretöntä.

        Samasta syystä jos eliön genomiin tehdään satunnaisia virheitä, ne aiheuttaa pelkästään sairauksia tai jäävät piileviksi (eli eivät aiheuta mitään näkyvää).

        Biologit eivät tätä ymmärrä, koska eivät osaa koodata. Siksi he päätyvät taikauskoiseen ajatteluun että tuo a b toistettuna N kertaa johtaa uusien rakenteiden syntyyn jotenkin taikaiskusta.

        "Jokainen joka on tehnyt ohjelmointityötä tietää, että ohjelmaa ei voi kehittää evoluutioprosessilla eli että

        a) luodaan satunnaisia virheitä koodiin
        b) karsitaan huonoiten toimivat koodit pois
        Toistamalla vaiheita a ja b N kertaa ei tule mitään järkeviä uusia ohjelmia (rakenteita) vaan pelkkää virhekoodia, vaikka N lähestyisi ääretöntä."

        No niin. Taas sotketaan puurot ja vellit ja yksinkertaistetaan asiaa, jota yksinkertaistamalla ei voi selvittää. Eli evoluutiossa on kyse monitasoisesta ja monimutkaisesta prosessista, jossa keskittyminen koodiin ei johda mihinkään. DNA muodostuu 3 miljardista emäsparista. Se on se koodi. Siinä koodissa on noin23 000 geeniä, jotka muistuttavat yksittäisiä ohjelmia, joiden tarkoituksena on tuottaa tientylainen vaste.

        a) Tohtori tietää varmasti, että somaattiset solut uudistuvat keskimäärin 200 gramman vuorokausivauhtia. Jokaisessa solunjakautumisessa tapahtuu n määrä virheitä solun sisältämässä perimässä. Solut uusiutuvat kunnes telemeerin pituus loppuu. Se kuvaa vanhenemista. Solut eivät jakaudu loputtomiin ja solut kuolevat luonnollisesti tai jonkin patogeenin murhaamina. Mutta onko sillä väliä? Ihminen muodostuu kymmenistä tai sadoista miljoonista soluista. Osa kuolee, osa muuttuu ja joistain tulee pahanlaatuisia syöpäsoluja. Nämä muutokset joihin kretut viittaavat ovat juurikin solun geneettisen aineksen muutoksia. Jokainen somaattinen solu sisältää diploidin kromosomiston eli 23 kromosomiparia, joista puolet on peritty äidiltä ja puolet isältä. Kromosomi sisältää ihmisen perimän. Muutokset yksittäisen solun sisältämään perimään eivät tarkoita juuri mitään; sitä tapahtuu koko ajan.

        b) Tohtori tietää varmasti myös, että gameetit tai sukupuolisolut sisältävät haploidin 23 yksittäisestä kromosomista muodostuvan henkilökohtaisen perimän. Siten siittiön hedelmöittäessä munasolun kaksi haploidia kromosomistoa yhdistyy yhdeksi diploidiksi kromosomistoksi. Näin äidin ja isän uniikit perimät vaikuttavat sikiön perimään, joka saa ainutkertaisen uniikin genomin. Mutta tietenkään se ei ole ihan näin yksinkertaista. Somaattiset solut jakautuvat mitoosissa ja gameetit meioosissa. Monivaiheinen meioosi luo sukupuolisolujen geneettisen aineksen moniosaisessa prosessissa, jossa henkilön geenit valikoituvat sukupuolisoluun jokseenkin sattumanvaraisesti. Siten jokainen siittiö ei sisällä täsmälleen samanlaista DNA-ketjua, eikä jokainen munasolu sisällä identtistä DNA-ketjua.

        c) Tohtori toki ymmärtää, että vaikka DNA muistuttaa koodia, jossa emäsjärjestykset vaikuttvat geenien transkriptioon jne., kyse on koodista, jota ei voi verrata mihinkään meidän rakentamaamme järjestelmään. Erilaisia DNA-sekvenssejä voi olla laskennallisesti enemmän kuin koko maailmankaikkeudessa on atomeita, eli suurin piirtein 10 potenssiin 80. Mikään rakentamamme tietokone ei vilä ole lähelläkään tätä tasoa.

        d) Mutaatioita tapahtuu ensisijaisesti solutasolla ja se on älyttömän nopeaa, mutta suhteellisen merkityksetöntä. Muutokset ihmisten perimässä tapahtuvat hitaasti kymmenien ja satojen sukupolvien aikana suvullisen lisääntymisen seurauksena. Suvullisen lisääntymisen yhteydessä epigeneettiset muutokset ovat mutaatioita kiinnostavampi ilmiö, sillä niissä ympäristö vaikuttaa suoraan geenien ilmenemiseen. Ravinto tai ravinnon puute, lämpötila, kosteus, vitamiinien tai mineraalien runsaus tai puutos muuttavat genomia tai tarkemmin yksittäisiä emäspareja siten, että ne sopeutuvat paremmin vallitseviin olosuhteisiin. Eurooppalaistaustaiset ihmiset juovat paljon maitoa ja se on tärkeä ravintoaine, mutta valtaosa maailman väestöstä ei pysty varhaislapsuuden jälkeen maitoa juomaan, koska heiltä sammuu laktoosia pilkkovaa laktaasia tuottava geeni. Laktoosi-intoleranssi tila on ihmisen normi; erooppalaistaustaiset ovat sopeutuneet juomaan maitoa myös aikuisena - kyse on siis epigeneettisestä muutoksesta, joka vaikuttaa tiettyihin populaatioihin. Tätä tapahtuu kaiken aikaa ja monella tasolla. Ihminen voi sopeutua joihinkin ympäristön ärsykkeisiin tai allergisoitua niille. Sellaisten ihmisten geenit, jotka ovat parhaiten sopeutuneet vallitseviin ympäristö-olosuhteisiin muuttuvat vähitellen populaation sisällä vallitseviksi geenimuutoksiksi. Aluksi kyse on vain yksilötason sopeutumisesta, mutta vähitellen siitä tulee populaatiota määrittävä ominaisuus. Tuhat vuotta sitten elänyt esi-isäsi ei näyttänyt sinulta - ei edes isältäsi tai äidiltäsi. Hän saattoi olla pitkä tai pätkä, tumma tai vaalea, mutta mikä tärkeintä, hänen geneettinen perimänsä - tai osa siitä on elänyt näihin päiviin asti. Kun menemme hänestä 10 000 vuotta taaksepäin, olemme kivikaudella ja esi-isät näyttävät vieläkin vieraammilta. 240 000 vuotta taaksepäin ja esi-isät ovat vasta kehittyneet nyky-ihmisiksi.


      • EiHelvetti
        EiHelvetti kirjoitti:

        "Jokainen joka on tehnyt ohjelmointityötä tietää, että ohjelmaa ei voi kehittää evoluutioprosessilla eli että

        a) luodaan satunnaisia virheitä koodiin
        b) karsitaan huonoiten toimivat koodit pois
        Toistamalla vaiheita a ja b N kertaa ei tule mitään järkeviä uusia ohjelmia (rakenteita) vaan pelkkää virhekoodia, vaikka N lähestyisi ääretöntä."

        No niin. Taas sotketaan puurot ja vellit ja yksinkertaistetaan asiaa, jota yksinkertaistamalla ei voi selvittää. Eli evoluutiossa on kyse monitasoisesta ja monimutkaisesta prosessista, jossa keskittyminen koodiin ei johda mihinkään. DNA muodostuu 3 miljardista emäsparista. Se on se koodi. Siinä koodissa on noin23 000 geeniä, jotka muistuttavat yksittäisiä ohjelmia, joiden tarkoituksena on tuottaa tientylainen vaste.

        a) Tohtori tietää varmasti, että somaattiset solut uudistuvat keskimäärin 200 gramman vuorokausivauhtia. Jokaisessa solunjakautumisessa tapahtuu n määrä virheitä solun sisältämässä perimässä. Solut uusiutuvat kunnes telemeerin pituus loppuu. Se kuvaa vanhenemista. Solut eivät jakaudu loputtomiin ja solut kuolevat luonnollisesti tai jonkin patogeenin murhaamina. Mutta onko sillä väliä? Ihminen muodostuu kymmenistä tai sadoista miljoonista soluista. Osa kuolee, osa muuttuu ja joistain tulee pahanlaatuisia syöpäsoluja. Nämä muutokset joihin kretut viittaavat ovat juurikin solun geneettisen aineksen muutoksia. Jokainen somaattinen solu sisältää diploidin kromosomiston eli 23 kromosomiparia, joista puolet on peritty äidiltä ja puolet isältä. Kromosomi sisältää ihmisen perimän. Muutokset yksittäisen solun sisältämään perimään eivät tarkoita juuri mitään; sitä tapahtuu koko ajan.

        b) Tohtori tietää varmasti myös, että gameetit tai sukupuolisolut sisältävät haploidin 23 yksittäisestä kromosomista muodostuvan henkilökohtaisen perimän. Siten siittiön hedelmöittäessä munasolun kaksi haploidia kromosomistoa yhdistyy yhdeksi diploidiksi kromosomistoksi. Näin äidin ja isän uniikit perimät vaikuttavat sikiön perimään, joka saa ainutkertaisen uniikin genomin. Mutta tietenkään se ei ole ihan näin yksinkertaista. Somaattiset solut jakautuvat mitoosissa ja gameetit meioosissa. Monivaiheinen meioosi luo sukupuolisolujen geneettisen aineksen moniosaisessa prosessissa, jossa henkilön geenit valikoituvat sukupuolisoluun jokseenkin sattumanvaraisesti. Siten jokainen siittiö ei sisällä täsmälleen samanlaista DNA-ketjua, eikä jokainen munasolu sisällä identtistä DNA-ketjua.

        c) Tohtori toki ymmärtää, että vaikka DNA muistuttaa koodia, jossa emäsjärjestykset vaikuttvat geenien transkriptioon jne., kyse on koodista, jota ei voi verrata mihinkään meidän rakentamaamme järjestelmään. Erilaisia DNA-sekvenssejä voi olla laskennallisesti enemmän kuin koko maailmankaikkeudessa on atomeita, eli suurin piirtein 10 potenssiin 80. Mikään rakentamamme tietokone ei vilä ole lähelläkään tätä tasoa.

        d) Mutaatioita tapahtuu ensisijaisesti solutasolla ja se on älyttömän nopeaa, mutta suhteellisen merkityksetöntä. Muutokset ihmisten perimässä tapahtuvat hitaasti kymmenien ja satojen sukupolvien aikana suvullisen lisääntymisen seurauksena. Suvullisen lisääntymisen yhteydessä epigeneettiset muutokset ovat mutaatioita kiinnostavampi ilmiö, sillä niissä ympäristö vaikuttaa suoraan geenien ilmenemiseen. Ravinto tai ravinnon puute, lämpötila, kosteus, vitamiinien tai mineraalien runsaus tai puutos muuttavat genomia tai tarkemmin yksittäisiä emäspareja siten, että ne sopeutuvat paremmin vallitseviin olosuhteisiin. Eurooppalaistaustaiset ihmiset juovat paljon maitoa ja se on tärkeä ravintoaine, mutta valtaosa maailman väestöstä ei pysty varhaislapsuuden jälkeen maitoa juomaan, koska heiltä sammuu laktoosia pilkkovaa laktaasia tuottava geeni. Laktoosi-intoleranssi tila on ihmisen normi; erooppalaistaustaiset ovat sopeutuneet juomaan maitoa myös aikuisena - kyse on siis epigeneettisestä muutoksesta, joka vaikuttaa tiettyihin populaatioihin. Tätä tapahtuu kaiken aikaa ja monella tasolla. Ihminen voi sopeutua joihinkin ympäristön ärsykkeisiin tai allergisoitua niille. Sellaisten ihmisten geenit, jotka ovat parhaiten sopeutuneet vallitseviin ympäristö-olosuhteisiin muuttuvat vähitellen populaation sisällä vallitseviksi geenimuutoksiksi. Aluksi kyse on vain yksilötason sopeutumisesta, mutta vähitellen siitä tulee populaatiota määrittävä ominaisuus. Tuhat vuotta sitten elänyt esi-isäsi ei näyttänyt sinulta - ei edes isältäsi tai äidiltäsi. Hän saattoi olla pitkä tai pätkä, tumma tai vaalea, mutta mikä tärkeintä, hänen geneettinen perimänsä - tai osa siitä on elänyt näihin päiviin asti. Kun menemme hänestä 10 000 vuotta taaksepäin, olemme kivikaudella ja esi-isät näyttävät vieläkin vieraammilta. 240 000 vuotta taaksepäin ja esi-isät ovat vasta kehittyneet nyky-ihmisiksi.

        Jos vielä jatketaan pikakelauksella ajassa taaksepäin, nädään kuinka esi-isämme muuttuvat vähitellen ihmisapinoiksi jne. Muutos on hidas. Jos sinulla olisi valokuva jokaisesta miespuolisesta esivanhemmastasi 300 000 vuotta ajassa taaksepäin, voisit pikakelata kehityshistoriasi ja näkisit kuinka jokainen esi-isäsi muistuttaa sekä isäänsä että poikaansa, mutta erot kasvavat mitä kauemmaksi kuljetaan. Kyse on vähän kuin harmaan sävyistä - värikartasta jossa kahden vierekkäisen pikselin sävyero on lähes mahdoton osoittaa paljaalla silmällä, mutta värikartan ääripäissä muutokset ovat silmiinpistäviä - toisella puolella valkoinen ja toisella musta.

        Tästä päästään jälleen siihen, että vaikka DNA muistuttaa tietokoneen koodia, se on jatkuvasti useiden ulkoisten ja sisäisten tekijöiden säätelemä. Ulkoiset ärsykkeet muuttavat meitä epigeneettisesti ja sisäiset transkriptiosäätelijät varmistavat, että solujen uusiutumisessa tapahtuu minimaalinen määrä virheitä.

        Kaikilla aitotumallisilla soluilla kromosomien päässä on DNA-jakso, jota kutsutaan telomeeriksi. Kun RNA kopioi solun "dataa" se aloittaa luennan telomeerin päästä ja jokaisen solunjakautumisen jälkeen kopioitu telomeeri lyhenee. Näin solut vanhenevat. Solunjakautumisen virheet ja mutaatiot eivät kuitenkaan tarkoita aivan samaa asiaa niin kuin kretupellet haluavat valehdella. Mutaatioilla tarkoitetaan sukupuolisoluihin siirtyviä geneettisiä muutoksia, jotka periytyvät sukupolvelta seuraavalle. Useimmat solunjakautumisen seurauksena syntyvät muutokset eivät periydy. Osa perityy ja etenkin jos molemmilla vanhemmilla on sama geneettinen poikkeama, jälkeläinen sairastuu. Jos tällainen mutaatio periytyy vain toiselta vanhemmalta, mahdollisuus sairastumiseen on 50 %. Kaikki geenit eivät ole aktiivisia kaiken aikaa. Ihmisillä voi olla geneettinen alttius sairastua, mutta vain jos yksi tai useampi ymäristömuuttuja laukaisee sairauden - eli herättää mutatoituneen geenin tai esim. autoimmuunireaktion.

        Kretuille on selitetty miljoona kertaa, että muutos on hidasta ja mutaatiot eivät ole ollenkaan ainoa tai niin keskeinen tekijä evoluutiossa, kuin he haluaisivat ymmärtää. Muutokseen vaikuttavat mm.

        1. suvullinen lisääntyminen (seksuaalivalinta)
        2. migraatiot (muuttoliike ja sopeutuminen uuteen ympäristöön ja/tai eristyminen muusta populaatiosta)
        3. luonnonvalinta (eli sopeutumiskykyisimmillä geeneillä on parhaat edellytykset säilyä populaatiossa, kun heikoimmmilla geeneillä on taipumus hväitä geenipoolista),
        4. geneettinen ajautuminen (Geneettinen ajautuminen voi eliminoida joitain alleeleja populaatiosta pelkästään sattuman takia. Valintaa ohjaavien voimien puutteessakin, geneettinen ajautuminen voi aiheuttaa kaksi erillistä populaatiota (jotka aloittivat samalla geneettisellä koostumuksella) ajautumaan kahdeksi poikkeavaksi populaatioksi, joilla on eri alleeliryhmät.)
        5. geneettinen liftaaminen ( luonnonvalinnan kautta tapahtuva valintapyyhkäisy (engl. selective sweep) voi johtaa haplotyypissä olevien alleelien yleistymiseen populaatiossa; tätä ilmiötä kutsutaan geneettiseksi liftaamiseksi (engl. genetic hitchhiking).)
        6. geenivirta (Geenivirta viittaa populaatioiden ja lajien väliseen geenien siirtymiseen.[99] Geenivirran läsnäololla tai puuttumisella on merkittävä vaikutus evoluution kulkuun.)
        7. Mutaatiotaipumus (Sen lisäksi, että mutaatiot ovat merkittävä lähde variaatiolle, ne voivat myös toimia evoluution mekanismina silloin kun molekyylitasolla on olemassa eri todennäköisyydet mutaatioiden tapahtumiselle – tätä ilmiötä kutsutaan mutaatiotaipumukseksi (engl. mutation bias).)
        8. Luonnonvalinta (Luonnonvalinnan keskeinen käsite on eliön kelpoisuus. Kelpoisuus kuvaa eliön kykyä selviytyä ja tuottaa jälkeläisiä, mikä määrittää kuinka suuri sen geneettinen vaikutus on seuraavaan sukupolveen. Kelpoisuus ei kuitenkaan ole sama asia kuin jälkeläisten kokonaismäärä: sen sijaan, kelpoisuudesta kertoo se osuus seuraavista sukupolvista, jotka kantavat eliön geenejä.)

        Evoluutiota tapahtuu monella tasolla samanaikaisesti. Siihen vaikuttavia muuttujia - tohtori koodaaja tuntee muuttujat - on valtavasti. Aiheen typistäminen keskusteluksi mutaatioista tai mikro- ja makroevoluutiosta on älyllisesti epärehellistä ja typerää. Kaikkia näitä tapahtuu koko ajan ja jokainen meistä on evoluution tulos. Evoluutiolla ei ole suuntaa, kyse on vain sopeutumisesta vallitseviin olosuhteisiin ja parhaiden sopeutuvien lisääntymisestä. Ei evoluutio yritä tehdä meistä parempia, viisaampia tai kauniimpia. Se on vain taustamekanismi joka säätelee eliöiden ja tarkemmin geenien leviämistä ja selviytymistä. Evoluution kannalta rappio on sitä, että yksilö tai populaatioi ei sopeudu valliteviin olosuhteisiin ja kuolee sukupuuttoon. Mitään eettisiä tai moraalisia normeja rappioon ei liity.


      • EiHelvetti
        EiHelvetti kirjoitti:

        "Jokainen joka on tehnyt ohjelmointityötä tietää, että ohjelmaa ei voi kehittää evoluutioprosessilla eli että

        a) luodaan satunnaisia virheitä koodiin
        b) karsitaan huonoiten toimivat koodit pois
        Toistamalla vaiheita a ja b N kertaa ei tule mitään järkeviä uusia ohjelmia (rakenteita) vaan pelkkää virhekoodia, vaikka N lähestyisi ääretöntä."

        No niin. Taas sotketaan puurot ja vellit ja yksinkertaistetaan asiaa, jota yksinkertaistamalla ei voi selvittää. Eli evoluutiossa on kyse monitasoisesta ja monimutkaisesta prosessista, jossa keskittyminen koodiin ei johda mihinkään. DNA muodostuu 3 miljardista emäsparista. Se on se koodi. Siinä koodissa on noin23 000 geeniä, jotka muistuttavat yksittäisiä ohjelmia, joiden tarkoituksena on tuottaa tientylainen vaste.

        a) Tohtori tietää varmasti, että somaattiset solut uudistuvat keskimäärin 200 gramman vuorokausivauhtia. Jokaisessa solunjakautumisessa tapahtuu n määrä virheitä solun sisältämässä perimässä. Solut uusiutuvat kunnes telemeerin pituus loppuu. Se kuvaa vanhenemista. Solut eivät jakaudu loputtomiin ja solut kuolevat luonnollisesti tai jonkin patogeenin murhaamina. Mutta onko sillä väliä? Ihminen muodostuu kymmenistä tai sadoista miljoonista soluista. Osa kuolee, osa muuttuu ja joistain tulee pahanlaatuisia syöpäsoluja. Nämä muutokset joihin kretut viittaavat ovat juurikin solun geneettisen aineksen muutoksia. Jokainen somaattinen solu sisältää diploidin kromosomiston eli 23 kromosomiparia, joista puolet on peritty äidiltä ja puolet isältä. Kromosomi sisältää ihmisen perimän. Muutokset yksittäisen solun sisältämään perimään eivät tarkoita juuri mitään; sitä tapahtuu koko ajan.

        b) Tohtori tietää varmasti myös, että gameetit tai sukupuolisolut sisältävät haploidin 23 yksittäisestä kromosomista muodostuvan henkilökohtaisen perimän. Siten siittiön hedelmöittäessä munasolun kaksi haploidia kromosomistoa yhdistyy yhdeksi diploidiksi kromosomistoksi. Näin äidin ja isän uniikit perimät vaikuttavat sikiön perimään, joka saa ainutkertaisen uniikin genomin. Mutta tietenkään se ei ole ihan näin yksinkertaista. Somaattiset solut jakautuvat mitoosissa ja gameetit meioosissa. Monivaiheinen meioosi luo sukupuolisolujen geneettisen aineksen moniosaisessa prosessissa, jossa henkilön geenit valikoituvat sukupuolisoluun jokseenkin sattumanvaraisesti. Siten jokainen siittiö ei sisällä täsmälleen samanlaista DNA-ketjua, eikä jokainen munasolu sisällä identtistä DNA-ketjua.

        c) Tohtori toki ymmärtää, että vaikka DNA muistuttaa koodia, jossa emäsjärjestykset vaikuttvat geenien transkriptioon jne., kyse on koodista, jota ei voi verrata mihinkään meidän rakentamaamme järjestelmään. Erilaisia DNA-sekvenssejä voi olla laskennallisesti enemmän kuin koko maailmankaikkeudessa on atomeita, eli suurin piirtein 10 potenssiin 80. Mikään rakentamamme tietokone ei vilä ole lähelläkään tätä tasoa.

        d) Mutaatioita tapahtuu ensisijaisesti solutasolla ja se on älyttömän nopeaa, mutta suhteellisen merkityksetöntä. Muutokset ihmisten perimässä tapahtuvat hitaasti kymmenien ja satojen sukupolvien aikana suvullisen lisääntymisen seurauksena. Suvullisen lisääntymisen yhteydessä epigeneettiset muutokset ovat mutaatioita kiinnostavampi ilmiö, sillä niissä ympäristö vaikuttaa suoraan geenien ilmenemiseen. Ravinto tai ravinnon puute, lämpötila, kosteus, vitamiinien tai mineraalien runsaus tai puutos muuttavat genomia tai tarkemmin yksittäisiä emäspareja siten, että ne sopeutuvat paremmin vallitseviin olosuhteisiin. Eurooppalaistaustaiset ihmiset juovat paljon maitoa ja se on tärkeä ravintoaine, mutta valtaosa maailman väestöstä ei pysty varhaislapsuuden jälkeen maitoa juomaan, koska heiltä sammuu laktoosia pilkkovaa laktaasia tuottava geeni. Laktoosi-intoleranssi tila on ihmisen normi; erooppalaistaustaiset ovat sopeutuneet juomaan maitoa myös aikuisena - kyse on siis epigeneettisestä muutoksesta, joka vaikuttaa tiettyihin populaatioihin. Tätä tapahtuu kaiken aikaa ja monella tasolla. Ihminen voi sopeutua joihinkin ympäristön ärsykkeisiin tai allergisoitua niille. Sellaisten ihmisten geenit, jotka ovat parhaiten sopeutuneet vallitseviin ympäristö-olosuhteisiin muuttuvat vähitellen populaation sisällä vallitseviksi geenimuutoksiksi. Aluksi kyse on vain yksilötason sopeutumisesta, mutta vähitellen siitä tulee populaatiota määrittävä ominaisuus. Tuhat vuotta sitten elänyt esi-isäsi ei näyttänyt sinulta - ei edes isältäsi tai äidiltäsi. Hän saattoi olla pitkä tai pätkä, tumma tai vaalea, mutta mikä tärkeintä, hänen geneettinen perimänsä - tai osa siitä on elänyt näihin päiviin asti. Kun menemme hänestä 10 000 vuotta taaksepäin, olemme kivikaudella ja esi-isät näyttävät vieläkin vieraammilta. 240 000 vuotta taaksepäin ja esi-isät ovat vasta kehittyneet nyky-ihmisiksi.

        "Mutta onko sillä väliä? Ihminen muodostuu kymmenistä tai sadoista miljoonista soluista. " Pahus. Piti kirjoittaa "biljoonista".


      • EiHelvetti
        EiHelvetti kirjoitti:

        "Mutta onko sillä väliä? Ihminen muodostuu kymmenistä tai sadoista miljoonista soluista. " Pahus. Piti kirjoittaa "biljoonista".

        Lisätään vielä karkea, mutta kuvaava esimerkki evoluution mekanismeista. Espanjantauti tappoi 30-100 miljoonaa ihmistä viime vuosisdan alussa. Tauti levisi kaikkialle. Ne, jotka selvisivät olivat vastustuskykyisempiä tai immuuneja taudille. Lähes kaikilla nykyisin elävillä ihmisillä on immuniteetti Espanjantaudille, koska olemme perineet selviytyneiden geenejä. Luonnollinen valinta karsi jopa 3 % koko tuolloisesta ihmispopulaatiosta. Se ei kysellyt uskonnollista vakaumusta, sukupuolista suuntautumista tai mitään muutakaan vaan tappoi hyviä ja pahoja, kauniita ja rumia, rikkaita ja köyhiä. Tauti tarttui lähes joka toiseen ihmiseen.

        "Viimeaikainen tutkimus on osoittanut, että taudista aikanaan selvinneet nykyiset vanhukset kantavat edelleen espanjantautia aiheuttavan influenssaviruksen vasta-aineita veressään."


      • KreationistinenRealismi kirjoitti:

        John Sanford on nuoren Maan kreationisti. Mitä muuta häneltä voi odottaa kuin tuollaista julistusta. Muut geneetikot eivät vaan suostu allekirjoittamaan Sanfordin väitteitä, vaan pitävät hänen "tutkimuksiaan" päämäärähakuisina ja siten vääristeltyinä.
        Entisen NL:n aikoihin kaiken yhteiskuntatieteellisen tutkimuksen piti tukea kommunismia. Tulos oli jo enneen tutkimusta tiedossa.

        Faktat ovat kuitenkin faktoja. Tutkimustulokset voidaan toistaa ja tulokset ovat samat. Joten evolutionistien on aika haudata harhaoppinsa.


      • KylläHelvetti
        EiHelvetti kirjoitti:

        Lisätään vielä karkea, mutta kuvaava esimerkki evoluution mekanismeista. Espanjantauti tappoi 30-100 miljoonaa ihmistä viime vuosisdan alussa. Tauti levisi kaikkialle. Ne, jotka selvisivät olivat vastustuskykyisempiä tai immuuneja taudille. Lähes kaikilla nykyisin elävillä ihmisillä on immuniteetti Espanjantaudille, koska olemme perineet selviytyneiden geenejä. Luonnollinen valinta karsi jopa 3 % koko tuolloisesta ihmispopulaatiosta. Se ei kysellyt uskonnollista vakaumusta, sukupuolista suuntautumista tai mitään muutakaan vaan tappoi hyviä ja pahoja, kauniita ja rumia, rikkaita ja köyhiä. Tauti tarttui lähes joka toiseen ihmiseen.

        "Viimeaikainen tutkimus on osoittanut, että taudista aikanaan selvinneet nykyiset vanhukset kantavat edelleen espanjantautia aiheuttavan influenssaviruksen vasta-aineita veressään."

        Kiitos kattavasta selostuksestasi tätä asiaa kohtaan yllä olevissa viesteissäsi. Valitettavasti tohtori, jota opetit, ei piittaa siitä miten häntä täällä ojennetaan. Olen nähnyt, että hän toistuvasti luettelee samoja argumentointivirheitä sen jälkeen, kun hänelle ne on osoitettu.

        Periaatteessa on ihan mahdollista, ettei hän vain ole huomannut sitä, että hänen virheensä on hänelle osoitettu, mutta koska kaikki kreationistit ovat samanlaisia tässä asiassa, että he jatkuvat jankuttamistaan vastaavissa tilanteissa, niin pidän tätä epätodennäköisenä. Jotkut ihmiset eivät vain halua olla rehellisiä ja kehittää ajatteluaan, vaan pysyttelevät mielummin jo virheellisiksi osoitetuissa käsityksissään.

        Selvennän vielä vähän tätä esittämääni kreationisteja koskevaa yleistystä, etteivät he myönnä virheitään, että tämän olen havainnut tarkasteltuani asiaa muutamien vuosien ajan. On käynyt poikkeuksetta ilmeiseksi, että kreationistit alan organisaatioiden julistajista palstojen ristinritareihin ovat aivan samanlaisia tässä suhteessa.

        Kun kumoat yhden heidän "argumenttinsa", niin he sivuuttavat sen asian ja vaihtavat toiseen; kun kumoat sen toisen, niin he siirtyvät kolmanteen jne. Sitten kun tämä "entäs sitten tämä, että..." -argumentointi heidän osaltaan päätty siihen pisteeseen, että he ovat saaneet käytyä läpi suosimansa luomissivuston listan, niin he aloittavat sen alusta ehkä viikon kahden sisällä, samalla palstalla tai vaikka samassa ketjussa.

        Jos nyt huomautat heille siitä, että nämä asiat on jo käsitelty heidän argumentatiiviseksi tappiokseen, niin he jälleen vain sivuuttavat asian. Ja sama rumba jatkuu vuosikausia putkeen. Kreationistit eivät opi, koska he eivät ole asenteeltaan rehellisiä, vaan he haluavat uskoa niin kovasti, että he uskottelevat itselleen lisäsyitä uskoa, olivatpa ne kuinka kelvottomia tahansa.

        Sivistysmaissa pitkällä aikavälillä ihmisiin vetoaa kuitenkin järkipuhe, jota sinäkin yllä esitit, ja kreationismin kaltainen pseudotieteellis-apologeettinen populismi jää alati ikääntyneemmän vanhan ajan tietotason ja arvojen väen osaksi, eli kuihtuu kuihtumistaan. Maailma voi näyttää sivistyksen kannalta synkältä paikalta, mutta ei pidä harhautua katsomaan tätä noiden muutamien, vaikkakin äänekkäiden paasaajien dominoimaksi kentäksi.


      • evlut101 kirjoitti:

        "Tämä on täydellinen esimerkki siitä miten evoluutio toimii luonnossakin, käyttäen esimerkkinä standardisoituja olosuhteita jotta voidaan mitata evoluution vaikutus suhteessa tähän parhaiten :) "

        Kun evoluutio muodostaa rakenteita, sen täytyy tehdä useita mutaatioita elämän koodiin. Esimerkiksi puhekyvyn rakentuminen vaatii valtavat määrät mutaatioita. Jos lähdetään siitä että genomiin tulee puhekyvyttömälle alkuihmiselle satunnaisia virheitä, joiden seurauksena hän saa puhekyvyn ja kyvyn ymmärtää puhetta, niin kuinka monta satunnaismutaatiota tarvitaan? Vastaus on vähintään miljoonia. No jos taas lasketaan todennäköisyyttä sille että miten nämä miljoonat mutaatiot syntyivät sattumalta, niin ollan täysin mahdottoman tapahtuman edellä. Siis vähän kun otettaisiin windows käyttöjärjestelmä ja heitettäisiin sinne miljoona bittivirhettä ja oletettaisiin, että siihen tulisi sattumalta uusia ominaisuuksia eikä toiminnallisuus kärsisi.

        Laskennallisesti yksi pistemutaatio on todennäköinen (tapaus antibiootit tai malarialoinen). Kahden pistemutaation polku on jo erittäin todennäköinen, mutta raportoitu malarialoisella. Se tuli vastustuskykyiseksi malarialääkkeelle. Mutta miljoonien pistemutaatioiden kulku on paitsi laskennallisesti mahdoton, sitä ei myöskään tue mikään kokeellinen data. Eli monimutkaisten rakenteiden kuten bakteerimoottori tai puhekyky, kehittyminen evoluutioprosessin kautta on teoriassa ja kokeiden kautta osoitettu mahdottomaksi. Se on pelkkää todennäköisyyslaskentaa. Pitäisi tapahtua valtava määrä mutaatioita, jotka rakentavat monimutkaista rakennetta itsestään ilman että yksilön sopeutumiskyky siinä välissä paranee lainkaan. Bakteerien antibioottiresistenssikokeet eivät anna tähän mitään apua, koska niissä kyse on pelkästään yhden tai max kahden pistemutaation polusta.

        "Kun evoluutio muodostaa rakenteita, sen täytyy tehdä useita mutaatioita elämän koodiin. Esimerkiksi puhekyvyn rakentuminen vaatii valtavat määrät mutaatioita. Jos lähdetään siitä että genomiin tulee puhekyvyttömälle alkuihmiselle satunnaisia virheitä, joiden seurauksena hän saa puhekyvyn ja kyvyn ymmärtää puhetta, niin kuinka monta satunnaismutaatiota tarvitaan? Vastaus on vähintään miljoonia."

        Siinä menee vähän hakuseen että puhekyvyn kehittyminen ei vaadi sitä että kaikki mutaatiot tulisivat kerralla ja jo osittaisella kyvyllä äännellä on hyötyä eläimille, joten evoluutio "ohjaa" nostamalla selviytymistodennäköisyyttä.

        Tällöin todennäköisyys että tulee positiivinen mutaatio nousee.

        Vinkkinä, jos yrität ottaa esiin "todennäköisyyksiä ja miten epätodennäköistä että jotain tapahtuu" kannattaa yrittää tuoda ilmi asioita joista ei olisi hyötyä osittain kehittyneenä. Joskin toisaalta tällä hetkellä ei tiedetä mitään vastaavaa joka auttaisi todistamaan kreationismin :)


      • Msmslsl kirjoitti:

        Geneetikko John Sanford: Ihmiskunta rappeutuu pikavauhtia:

        ”Meidän sukupolvellamme on keskimäärin 100-200 mutaatiota enemmän kuin vanhemmillamme, ja heillä on niitä saman verran enemmän kuin heidän vanhemmillaan. Tämä johtuu siitä, että joka kerta, kun DNA:ta välittyy sukupolvelta toiselle, siihen ilmestyy lisää mutaatioita.

        Mutaatiot ovat enimmäkseen vahingollisia. Ne ovat geneettisiä kirjoitusvirheitä, mutta darvinismin mukaan ne ovat yksi evoluution kivijaloista, ja niiden pitäisi miljoonien vuosien saatossa parantaa lajin elinkelpoisuutta.

        Tämä kivijalka rapistuu kuitenkin melko nopeasti. Kehitys kulkee väärään suuntaan.

        Cornellin yliopiston professori John Sanford luennoi aiheesta hiljattain Darwin Was Wrong -konferenssissa. Geneetikko Sanford kertoi, että darvinistit ovat aliarvioineet vahingollisten mutaatioiden määrää, koska he eivät ole laskeneet ”roska-DNA:n” muutoksia. Näin ollen heiltä on jäänyt huomioimatta 90 prosenttia mutaatioista.

        Professori Sanford ei ole ainoa, joka sanoo ihmisten rappeutuvan. Myös Nature News vahvisti hiljattain hänen havaintonsa. Tulos kävi ilmi, kun tutkijat sekvensoivat Y-kromosomia.

        Tästä voitaneen päätellä, että mitä enemmän aikaa kuluu, sitä pahemmin ihmiskunta rappeutuu. Evoluutio kulkee siis täysin väärään suuntaan. Vuosimiljoonat merkitsisivät sukupuuttoa.

        Myös lääkäri Pekka Reinikainen ja biokemisti Pauli Ojala ovat ehdottaneet, että näemme evoluution sijasta devoluutiota eli kehityksen asemesta rappeutumista.

        Lähteet:

        Dolgin, Elie. 2009. Human mutation rate revealed. Nature News (27.8.). http://www.nature.com/news/2009/090827/full/news.2...

        Sanford, John. 2009. Darwin Was Wrong about Natural Selection. Luento Darwin Was Wrong konferenssissa (14.11.)

        http://bwanajoe.blogspot.com/2009/11/geneetikko-jo...

        Uusi tutkimus näyttää, etta rappeutuminen keräytyy geenibooliin:

        "Take, to begin with, the Swedish chickens. Three years ago, researchers led by a professor at the university of Linköping in Sweden created a henhouse that was specially designed to make its chicken occupants feel stressed. The lighting was manipulated to make the rhythms of night and day unpredictable, so the chickens lost track of when to eat or roost. Unsurprisingly, perhaps, they showed a significant decrease in their ability to learn how to find food hidden in a maze.

        The surprising part is what happened next: the chickens were moved back to a non-stressful environment, where they conceived and hatched chicks who were raised without stress - and yet these chicks, too, demonstrated unexpectedly poor skills at finding food in a maze. They appeared to have inherited a problem that had been induced in their mothers through the environment.

        Further research established that the inherited change had altered the
        chicks' "gene expression" - the way certain genes are turned "on" or "off", bestowing any given animal with specific traits. The stress had affected the mother hens on a genetic level, and they had passed it on to their offspring."

        Eli ongelma oli periytynyt äidiltä lapselle päinvastoin kuin evoteoria ennustaa.

        Eli kanat eivät kehittyneet ympäristön mukaan, vaan päinvastoin heidän geeninsä muuttuivat huonommaksi uuden ympäristön mukaan! Eli kävi päinvastoin kuin mitä evoluutioteoria ennustaa...

        http://www.guardian.co.uk/science/2010/mar/19/evol...

        Biologisen informaation mitattavuus:

        ”Nykyään elää monia lentokyvyttömiä lintulajeja, joille siivillä ei ole sama merkitys kuin niiden lentäville sukulaisille. Evolutionistit pitävät siipiä surkastumina tai keskeneräisinä rakenteina.

        Ihminen ei ole kana jota tahallaan stressataan, sinusta en tiedä.


      • Fyi kirjoitti:

        Mitkä muutokset?

        Stagnaatiota näyttä tapahtuvan eräille, varsinkin niille jotka eivät ymmärrä eikä halua ymmärtää.


      • lisäävirheitä kirjoitti:

        Sillä tarkoitataan sitä, että jatkuvasti syntyy uusia sairauksia ja löydetään uusia virhegeenejä!!!
        Vaikka hoitomenetelmät ovat parantuneet, niin sairastuvuus ei vähene.

        Ihmiset elää siis ihan "tuurilla" vanhemmaksi sukupolvelta toiseen, ja paiserutto, polio y. m taudit on nykyään kohtalaisen harvinaisia .


    • Evoluutio ei lopu milloinkaan maapallolla. Kosminen tapahtuma ja ydinaseilla käyty sota tietenkin voi lopettaa kaiken.

      • Ei nyt yksi asteroidi tai muu pikkupamaus suuria merkitse.


      • Ihminen voi muuttaa evoluution kehityskulkua, mutta ei pysäyttää tai lopettaa sitä.


      • mummomuori kirjoitti:

        Ihminen voi muuttaa evoluution kehityskulkua, mutta ei pysäyttää tai lopettaa sitä.

        Jotenkin juu ovat tuhannet sääskilajitkin kehittyneet. Taitaa olla jatkuvaa muuttumista ja uuden syntymistä pitemmällä aikavälillä.


      • kekek-kekek kirjoitti:

        Ei nyt yksi asteroidi tai muu pikkupamaus suuria merkitse.

        Onko änkyttävällä kekek lukemisen ymmärtämisen kanssa jotain vaikeuxia.


    • eialaeiklopu

      Evoluutio on joko nopeaa tai hidasta sen mukaan miten asia pitää nähdä. Hidasta se on silloin kun ajatellaan, että tapahtunut jotain lähes käsittämätöntä. Kun ihminen muovautunut jostain alkuitiöstä, niin siihen jo pannaan miljardeja vuosia vaikkei elinkelpoista maapalloa ole ollut niin kauan. Sitten kun taasen puhutaan vaikka puhetaidon sekä sen ymmärtämisen kehittymisestä, niin siinä on tapahtunut evoluutiohyppäys muutaman tuhannen vuoden sisällä. Muutoinhan luolien seinät olisi täynnä ihmisten kirjoituksia.

      Ihmisen mikroevoluutio ei ole mitään varsinaista evoluutiota. Se on vain lajikohtaisia muutoksia, jotka ovat lajin sisällä jo ennetään. Niiden esiintyminen vain vaihtelee sen mukaan miten lisäännytään. Siinä miljardi vuotta ei muuta ihmistä mitenkään mitä se jo nyt on. Jo olevat ominaisuudet vain voivat painottua eri tavalla.


      Makroevoluutio ei ole vielä edes alkanut.

      • Biologian.perusteet

        Makroevoluutio on vain kumuloitunutta mikroevoluutiota. Kuten luonto osoittaa se, missä vaiheessa kaksi yhteisestä kantamuodosta polveutuvaa populaatiota ei enää pysty risteytymään keskenään (=lopullinen lajiutuminen =makroevoluutiota), on sattumanvaraista.


      • ElinkelpoinenPallo

        Maa on ollut elinkelpoinen yli 3,5 miljardia vuotta. Kehittyneet eliöt ovat tulleet maapallolle viimeisen 500 miljoonan vuoden aikana. Sitä ennen kehitys oli hidasta.


      • Ei nyt sentään muutaman tuhannen vuotta riitä, nolla perään niin ollaan lähempänä todellisuutta.


    • nää_on_näitä

      Onko alkanutkaan. Tietenkin on keksitty kaikenlaisia härpäkkeitä ja selityksiä omille kokemuksille, (tiede) mitä uutta tai kehitystä se edustaa?
      Kuvitellaan joku ilmiö ja sitten sille kehitetään mitari - ilmaisin. Ei se ole ihmisen kehitystä, se on selitysten kehittymistä.

    • Kysyjäx

      Täh? Tässäpä taas yksi järjetön kysymys. Ei evoluutio "lopu". Tämä käsitys pohjaa ilmeisesti sille väärälle tiedolle, että evoluutiolla olisi muka joku päätepiste. Näin Ei evoluutiolla ole mitään "uskonnollista" päätepistettä. Evoluutio vain on, ei sen enempää. Jos ja kun luonnossa tapahtuu joku suuri mullistus, niin sen seurauksena elämä etsii omat lajinsa, jotka sopeutuvat paremmin uusiin oloihin.

      • "Evoluutio vain on, ei sen enempää. "

        Täsmälleen näin.


      • dafbgtg
        mummomuori kirjoitti:

        "Evoluutio vain on, ei sen enempää. "

        Täsmälleen näin.

        Evoluutio on kasa teorioita ja oletuksia. Tieteen kanssa sillä ei ole mitään tekemistä.


      • mummomuori kirjoitti:

        "Evoluutio vain on, ei sen enempää. "

        Täsmälleen näin.

        On syytä erottaa evoluutio ja evoluutioteoria toisistaan. Myös kreationistit hyväksyvät evoluution, joka on muuntelua. Sen sijaan evolutionistit sekoittavat muuntelun ja evoluutioteorian ja puhuvat näistä ikään kuin samasta asiasta.


    • Kilpailu

      Jaa, kehitys kyllä jatkuu yhä. Laji on kuitenkin saavuttanut jo huippunsa, se on paras kaikessa eikä ole enää kilpailua. Jos kaikki menee niin kuin on nyt ennustettu, seuraavaksi ihmisen evoluutiossa alkaa lajin taantumisen vaihe.

      • Mistä muuten tuollainen harhaoppi, että paras olisi aina luonnonvalinnassa valittu? Tosiasia on kuitenkin että ei paras vaan onnekkain. Eikä tämäkään tieto edistä evoluutiota tippaakaan.


      • Yksi_usko kirjoitti:

        Mistä muuten tuollainen harhaoppi, että paras olisi aina luonnonvalinnassa valittu? Tosiasia on kuitenkin että ei paras vaan onnekkain. Eikä tämäkään tieto edistä evoluutiota tippaakaan.

        Jaa. Ja me olemme kolmas ihmislaji, joka selvisi muita paremmin.

        Mutta ei siinä mitään evoluutioa ole.


      • torre12 kirjoitti:

        Jaa. Ja me olemme kolmas ihmislaji, joka selvisi muita paremmin.

        Mutta ei siinä mitään evoluutioa ole.

        Höpsis. Neandertalilaiset ym. evulutionistien "ihmislajit" ovat täyttä huuhaata. Kyseessä on ihminen, niin kuin mekin. Nämä evolutionistien "ihmislajit" ovat alentaneet Neandertalilaiset ym. ihmiset alkeellisiksi. Hyi olkoot.

        "Evolutionistit tulkitsevat neandertalilaisten mtDNA:n ja nykyihmisten mtDNA:n välillä olevat erot seuraavasti: Neandertalilaistesn ajatellaan eronneet nykyihmisiin johtaneesta kehityslinjasta noin 550 000 - 690 000 vuotta sitten. Neandertalilaiset kuolivat sukupuuttoon ilman, että niiden mtDNA:ta olisi periytynyt meille saakka. Tämä viittaisi siihen, että neandertalilaiset eivät kehittyneet aidoiksi ihmisiksi. He olivat nykyihmisistä erillinen laji. He olivat vain yksi monista esi-ihmisen muodoista, yksi kehityksen epäonnistunut koe. Vain me kehityimme täyteen ihmisyyteen." (lainaus Lubenowin kirjasta Myytti apinaihmisestä).

        Mitokondrio-DNA (mtDNA) periytyy pelkästään naisen kautta. Voin kertoa sinulle, että sinulla on vanhat harhaopit.


      • Ei evoluutiolla ole mitään Pyhää Päämäärää .


    • viimeinen.tuomio.2

      Evoluutio menee taaksepäin kuin Suomi24:n kehitystiimissä. Kuukauden tärkeimmät asiakeskustelut ja kysymykset suunnataan kolmevuotiaille :D
      https://keskustelu.suomi24.fi/t/15451143/tulossa-kysy-mita-vain-suomi24ssa-marraskuussa

      Kyllähän sitä on huomannut keskustelevansa palstalla monen lapsen kehitystasolle jääneen "uskovaisen"kanssa, mutta että sivuston kohde- ja käyttäjäryhmä onkin 3-vuotiaat, joilla ei pitäisi vielä olla mitään asiaa nettiin... Ei siis ole ihme eräiden kirjoitusongelmat, hymiöt ja kyselykaudet, sekä lapsiavioliitot...

      Tämä onkin Suomi2-4.fi ! Eli suunnattu 2-4 -vuotiaille.

    • eijääikävä

      Evoluutio voi oikein hyvin, myös ihmisen evoluutio. Ainoastaan kreationismi näyttää tulleen kulttuurievolutiivisen tiensä päähän 1859 kun Darwin julkaisi Lajien synnyn. Siitä lähtien kreationismi on rappeutunut rappeutumistaan nykyisen kaltaiseksi denialistiseksi vääristelyksi.

      Kreationismin rippeet odottaa enää viimeistä niskalaukausta tieteeltä jonka jälkeen koko kreationismi voidaan haudata yhtenä ihmiskunnan harha-askelena ja antaa sille kevyet mullat.

      • MelkoTuoreTapaus

        Kreationismi oli alkujaan konservatiivinen vastareaktio evoluutioterialle, sen syntyvuodeksi voidaan katsoa 1856. Kreationismi eli luomisusko on oppi, jonka mukaan kaikkeus ja sen elolliset olennot ovat syntyneet luomistapahtumassa. Kreationismilla voidaan suppeimmillaan tarkoittaa nk. nuoren maan kreationismia, laajimmillaan sillä voidaan tarkoittaa uskoa jonkinlaiseen luomistapahtumaan. Monet ateistit pitävät nuorenmaankreationisteja ainoina oikeina kreationisteina, kun taas vanhanmaan kreationistit ovat luopioita ja lällyjä. Itse kreationismi on näennäistiedettä, eikä sen opilla ole tieteellistä todellisuusperää.


      • capiche
        MelkoTuoreTapaus kirjoitti:

        Kreationismi oli alkujaan konservatiivinen vastareaktio evoluutioterialle, sen syntyvuodeksi voidaan katsoa 1856. Kreationismi eli luomisusko on oppi, jonka mukaan kaikkeus ja sen elolliset olennot ovat syntyneet luomistapahtumassa. Kreationismilla voidaan suppeimmillaan tarkoittaa nk. nuoren maan kreationismia, laajimmillaan sillä voidaan tarkoittaa uskoa jonkinlaiseen luomistapahtumaan. Monet ateistit pitävät nuorenmaankreationisteja ainoina oikeina kreationisteina, kun taas vanhanmaan kreationistit ovat luopioita ja lällyjä. Itse kreationismi on näennäistiedettä, eikä sen opilla ole tieteellistä todellisuusperää.

        "Itse kreationismi on näennäistiedettä, eikä sen opilla ole tieteellistä todellisuusperää."

        Kreationismi on uskoa, sillä ei ole tieteen kanssa mitään tekemistä. Ei edes näennäistieteen. Raamattu on minulle hengellinen opas, eikä minkään tieteen alan oppikirja.


      • Väärästä_ekvivalenssista

        "Kreationismin rippeet odottaa enää viimeistä niskalaukausta tieteeltä jonka jälkeen koko kreationismi voidaan haudata yhtenä ihmiskunnan harha-askelena ja antaa sille kevyet mullat."

        Kysehän tuossa on toki vertauskuvasta, mutta itse muotoilisin sen niin, että kreationismi itse laittaa itselleen sen viimeisen laakin siinä vaiheessa kun myöntää, ettei tiede edes piittaa koko kreationismista. Sinänsä oivaa vertauskuvaasi muuttaisinkin näin lähinnä siksi, ettei kenellekään tulisi kreationismin agendan mukaisesti väärinkäsitystä ekvivalenssista.

        Kreationismihan nimittäin esittäytyy kuin sillä olisi jotain rahkeita pelata tieteen kentällä. Ikään kuin siis olisi kreationistista tiedettä vs. "evolutionistinen" tai "ateistinen tiede". Ikään kuin kreationistit tietäisivät siitä mistä he puhuvat yhtä paljon kuin kyseisten alojen tieteelliset asiantuntijat.

        Todellisuudessa kun ei ole kyse edes siitä, että tieteessä ja kreationismissa millään tavalla pelattaisiin samassa sarjassa, kun ei siinä pelata edes samaa peliä tai pelata mitään ylipäätään! Siinä missä tiede on ammattimaista työtä, kreationismi on kieroa ja valheellista uskottelua, jolla yritetään pönkittää uskontoa yleensä lähinnä vääristelemällä tiedettä.

        Mutta tästä huolimatta kreationistit yrittävät antaa vaikutelmaa, että heillä muka olisi jotain oikeaa asiaa tieteelliseen vuoropuheluun. Kun tieteilijät kieltäytyvät debateista kreationistien kanssa, kreationistit teeskentelevät olevansa liian kovia luita tieteilijöille, vaikka tosiasiassa tieteilijöillä ei vain ole mitään motiivia haaskata aikaansa antaen samalla vaikutelmaa, että kreationistit ylipäätään olisivat yhtä vakavastiotettavia kuin tieteilijät.

        Kreationistit siis saavat vain evankeliointitilaisuuden areenoilla, joille he eivät kuulu, silloin kun tieteilijät viitsivät alentua heidän kanssaan keskusteluun. Nyt muuten huomautan vielä siitä, että on tietysti kreationistisiakin tieteilijöitä, eikä tarkoitukseni ole väittää, että olla kreationisti olisi yhtä kuin olla olematta tieteilijä - kyse vain on siitä, ettei mitään kreationistista tieteenalaa ole, eikä mitän kreationistista tiedettä, ja silloin kun tieteilijä on kreationisti, johtuu hänen kreationisminsa hänen uskonnollisista uskomuksistaan eikä tieteellisistä käsityksistään saati ansioistaan.

        Älkää siis menkö vahingossakaan siihen halpaan, että antaisitte kreationismille sellaista arvoa, mitä se yrittää väärin perustein ottaa. Kun tiede on älymystön raskaan sarjan ammattilaisten kovalla työllä, yhtäältä kilpailulla, toisaalta myös parviälyllä aikaansaatu tiedonhankkimisjärjestelmä, niin kreationismi on raamattupiirien valtaa pönkittävien organisaatioiden nimittämien paasaajien älämölöä vailla järjen häivää.


    • Looooogisesti

      Kysymykseen on sisäänrakennettu oletus että evoluutiota tapahtuisi. Kannattaa olla tarkkana tuollaisten kysymysten suhteen. Oikea tapa on kysyä ensin tapahtuuko ihmisessä evoluutiota. Senkin ohessa tulisi antaa esimerkki millaista evoluutiota tarkoitetaan, koska evoluutiouskontoja on niin monenlaisia

      • Faktaa.väliin

        Evoluutioteoria on modernin biologian keskeisin tulos. Evoluutioteoria on ainoa kattava ja ristiriidattomasti havaintoihin sopiva selitys maapallon biodiversiteetin synnylle. Tästä ovat yksimielisiä kaikki vapaat biologiaa tutkivat yliopistot.

        Kreationismi on uskontoa ja uskontona niitä on tietenkin monenlaisia eri lahjoja.


      • Faktaa.väliin kirjoitti:

        Evoluutioteoria on modernin biologian keskeisin tulos. Evoluutioteoria on ainoa kattava ja ristiriidattomasti havaintoihin sopiva selitys maapallon biodiversiteetin synnylle. Tästä ovat yksimielisiä kaikki vapaat biologiaa tutkivat yliopistot.

        Kreationismi on uskontoa ja uskontona niitä on tietenkin monenlaisia eri lahjoja.

        "Evoluutioteoria on ainoa kattava ja ristiriidattomasti havaintoihin sopiva selitys maapallon biodiversiteetin synnylle. "

        Näin, tästä ollaan yksimielisiä. Sen sijaan vaihtoehtoiset selitykset ovat uskontojen - mukaan lukien alkuperäiskansojen kertomukset - selityksiä.


      • evlut101
        mummomuori kirjoitti:

        "Evoluutioteoria on ainoa kattava ja ristiriidattomasti havaintoihin sopiva selitys maapallon biodiversiteetin synnylle. "

        Näin, tästä ollaan yksimielisiä. Sen sijaan vaihtoehtoiset selitykset ovat uskontojen - mukaan lukien alkuperäiskansojen kertomukset - selityksiä.

        Se että joku selitys sopii havaintoihin ei tarkoita että se olisi oikea teoria. Tieteen historia tuntee paljon selityksiä, jotka sopivat havaintoihin, mutta mitkä osottautuivat virheellisiksi.

        Tieteellisen tiedon vaatimuksena on pidetty tyypillisesti toistettavuutta ja falsifioitavuutta. Evoluutioteoriaa ei voi yksikäsitteisesti falsifioida, eikä myöskään toistaa, joten se täytyy laskea enemmänkin ideologiseksi kuin tieteelliseksi teoriaksi.

        Mutta kun jotain väitettä toistaa tarpeeksi, niin siitä tulee ihmisten mielissä tosi. Tästä syystä tekin pidätte sitä totena. Aikalaiset kritisoivat ankarasti evoluutioteoriaa eikä monet arvostetut tieteentekijät hyväksyneet sitä. Suuri ongelma oli evoluution todisteiden puute (ongelma yhä) ja toisekseen vielä suurempana ongelmana evoluutiomekanismin täydellinen puuttuminen.


      • evlut101 kirjoitti:

        Se että joku selitys sopii havaintoihin ei tarkoita että se olisi oikea teoria. Tieteen historia tuntee paljon selityksiä, jotka sopivat havaintoihin, mutta mitkä osottautuivat virheellisiksi.

        Tieteellisen tiedon vaatimuksena on pidetty tyypillisesti toistettavuutta ja falsifioitavuutta. Evoluutioteoriaa ei voi yksikäsitteisesti falsifioida, eikä myöskään toistaa, joten se täytyy laskea enemmänkin ideologiseksi kuin tieteelliseksi teoriaksi.

        Mutta kun jotain väitettä toistaa tarpeeksi, niin siitä tulee ihmisten mielissä tosi. Tästä syystä tekin pidätte sitä totena. Aikalaiset kritisoivat ankarasti evoluutioteoriaa eikä monet arvostetut tieteentekijät hyväksyneet sitä. Suuri ongelma oli evoluution todisteiden puute (ongelma yhä) ja toisekseen vielä suurempana ongelmana evoluutiomekanismin täydellinen puuttuminen.

        Näin on. Ei kannata uskoa mihinkään.


      • evlut101 kirjoitti:

        Se että joku selitys sopii havaintoihin ei tarkoita että se olisi oikea teoria. Tieteen historia tuntee paljon selityksiä, jotka sopivat havaintoihin, mutta mitkä osottautuivat virheellisiksi.

        Tieteellisen tiedon vaatimuksena on pidetty tyypillisesti toistettavuutta ja falsifioitavuutta. Evoluutioteoriaa ei voi yksikäsitteisesti falsifioida, eikä myöskään toistaa, joten se täytyy laskea enemmänkin ideologiseksi kuin tieteelliseksi teoriaksi.

        Mutta kun jotain väitettä toistaa tarpeeksi, niin siitä tulee ihmisten mielissä tosi. Tästä syystä tekin pidätte sitä totena. Aikalaiset kritisoivat ankarasti evoluutioteoriaa eikä monet arvostetut tieteentekijät hyväksyneet sitä. Suuri ongelma oli evoluution todisteiden puute (ongelma yhä) ja toisekseen vielä suurempana ongelmana evoluutiomekanismin täydellinen puuttuminen.

        ”Evoluutioteoriaa ei voi yksikäsitteisesti falsifioida, eikä myöskään toistaa, joten se täytyy laskea enemmänkin ideologiseksi kuin tieteelliseksi teoriaksi.”

        Evoluutio on yksimielisesti tieteellisesti todettu. Mitä tulee teorioihin, niin tiede korjaa niitä sitä mukaa, kun uutta tietoa saadaan.

        Mitä sitten tulee johonkin maailman syntyteorioihin, niin niitäkin on useampia. Mutta tähtitiede ja monitieteelliset tutkimukset antavat tästä pohjaa aina uusille teorioille. Kreationismi ei ole sellainen, vaan se on uskonnollinen näkemys.


      • anna.taas.nauraa
        mummomuori kirjoitti:

        ”Evoluutioteoriaa ei voi yksikäsitteisesti falsifioida, eikä myöskään toistaa, joten se täytyy laskea enemmänkin ideologiseksi kuin tieteelliseksi teoriaksi.”

        Evoluutio on yksimielisesti tieteellisesti todettu. Mitä tulee teorioihin, niin tiede korjaa niitä sitä mukaa, kun uutta tietoa saadaan.

        Mitä sitten tulee johonkin maailman syntyteorioihin, niin niitäkin on useampia. Mutta tähtitiede ja monitieteelliset tutkimukset antavat tästä pohjaa aina uusille teorioille. Kreationismi ei ole sellainen, vaan se on uskonnollinen näkemys.

        "Evoluutio on yksimielisesti tieteellisesti todettu. Mitä tulee teorioihin, niin tiede korjaa niitä sitä mukaa, kun uutta tietoa saadaan. "
        Johan taas vitsin murjaisit!!!


      • Ei.luotu.eikirj
        anna.taas.nauraa kirjoitti:

        "Evoluutio on yksimielisesti tieteellisesti todettu. Mitä tulee teorioihin, niin tiede korjaa niitä sitä mukaa, kun uutta tietoa saadaan. "
        Johan taas vitsin murjaisit!!!

        Eli tunnustat ettet tiedä evoluutiosta tai tieteistä mitään. Tuosta se lähtee; seuraavaksi voitkin kirjastosta lainata aihetta käsitteleviä kirjoja ja alkaa sivistämään itseäsi. Niin ehkä joskus vieäl järkevään keskusteluun aiheesta.


      • evlut101
        mummomuori kirjoitti:

        ”Evoluutioteoriaa ei voi yksikäsitteisesti falsifioida, eikä myöskään toistaa, joten se täytyy laskea enemmänkin ideologiseksi kuin tieteelliseksi teoriaksi.”

        Evoluutio on yksimielisesti tieteellisesti todettu. Mitä tulee teorioihin, niin tiede korjaa niitä sitä mukaa, kun uutta tietoa saadaan.

        Mitä sitten tulee johonkin maailman syntyteorioihin, niin niitäkin on useampia. Mutta tähtitiede ja monitieteelliset tutkimukset antavat tästä pohjaa aina uusille teorioille. Kreationismi ei ole sellainen, vaan se on uskonnollinen näkemys.

        "Evoluutio on yksimielisesti tieteellisesti todettu. Mitä tulee teorioihin, niin tiede korjaa niitä sitä mukaa, kun uutta tietoa saadaan. "

        Tämä on hyvä esimerkki tuosta papukaijamaisesta toistamisesta. Kun hoetaan tarpeeksi valheellista väitettä, niin valtaosa ihmisistä uskoo, vaikkei edes ymmärrä mitä evoluutioteoria tarkottaa.

        Tieteessä aniharvoin edes esiintyy yksimielisyyttä mistään asiasta eikä se ole edes tarkoitus vaan kuten tuossa alla kirjoitat, niin teorioita korjataan. Jos evoluutioteoriasta olisi yksimielisyys, niin sitä ei enää voisi korjata. Silloin se ei olisi tieteellinen teoria laisinkaan.

        Evoluutioteoriasta ei kuitenkaan esiinny yksimielisyyttä. On lukuisia evoluutiokriittisiä tieteentekijöitä listaa esim. täällä professoreita jne...
        http://www.thethirdwayofevolution.com/people
        Täällä yli 500 tieteentekijää kiistää evoluution
        https://evolutionnews.org/2006/02/over_500_scientists_proclaim_t/
        Lisäksi on paljon ihmisiä jotka uskoo Jumalan ohjaamaan evoluutioon ja se teoriana eroaa neodarvinistisesta evoluutiotoriasta, eli yksimielisiä ei olla senkään suhteen. Täältä voi lukea erilaisista prosenteista että miten missäkin uskotaan evoluutioteoriaan
        https://en.wikipedia.org/wiki/Level_of_support_for_evolution


      • Turhaahan_tämä
        evlut101 kirjoitti:

        "Evoluutio on yksimielisesti tieteellisesti todettu. Mitä tulee teorioihin, niin tiede korjaa niitä sitä mukaa, kun uutta tietoa saadaan. "

        Tämä on hyvä esimerkki tuosta papukaijamaisesta toistamisesta. Kun hoetaan tarpeeksi valheellista väitettä, niin valtaosa ihmisistä uskoo, vaikkei edes ymmärrä mitä evoluutioteoria tarkottaa.

        Tieteessä aniharvoin edes esiintyy yksimielisyyttä mistään asiasta eikä se ole edes tarkoitus vaan kuten tuossa alla kirjoitat, niin teorioita korjataan. Jos evoluutioteoriasta olisi yksimielisyys, niin sitä ei enää voisi korjata. Silloin se ei olisi tieteellinen teoria laisinkaan.

        Evoluutioteoriasta ei kuitenkaan esiinny yksimielisyyttä. On lukuisia evoluutiokriittisiä tieteentekijöitä listaa esim. täällä professoreita jne...
        http://www.thethirdwayofevolution.com/people
        Täällä yli 500 tieteentekijää kiistää evoluution
        https://evolutionnews.org/2006/02/over_500_scientists_proclaim_t/
        Lisäksi on paljon ihmisiä jotka uskoo Jumalan ohjaamaan evoluutioon ja se teoriana eroaa neodarvinistisesta evoluutiotoriasta, eli yksimielisiä ei olla senkään suhteen. Täältä voi lukea erilaisista prosenteista että miten missäkin uskotaan evoluutioteoriaan
        https://en.wikipedia.org/wiki/Level_of_support_for_evolution

        On huvittavaa, että vieläkin jaksat toistella noita "näin monta tieteilijä julistavat" -listojasi, kun nuo merkittävästi eivät edusta edes evoluutiobiologian ymmärryksen kannalta relevantteja aloja. Lisäksi tunnetaan tapauksia, joissa asiantuntijoiden lausuntoja evoluutiota koskien on väärinkäytetty kreationistien lanseeraamissa evoluutiosta irtisanoutumislistauksissa.

        Asiantuntijain keskuudessahan evoluution tai evoluutioteorian kiistäminen on käytännössä olematonta, ja kuten tarjoamastasi linkistäkin ilmenee, niin evoluutiota ja evoluutioteoriaa vastustetaankin nimenomaan USKONNOLLISISTA, eikä tieteellisistä syistä, silloin kun sitä vastustetaan.

        Sitä paitsi mitään teoriaa Jumalan ohjaamasta evoluutiosta ei ole olemassa, vaan on vain evoluutioteoria, mihin ei sisälly mitään jumalia. On eri asia, että suurin osa evoluution hyväksyvistä ihmisistä uskovat sen olevan jonkin jumalan aikaansaama ilmiö. Teistinen evolutionismi on uskonnollinen uskomus siitä, että tieteellisesti hyväksytty evoluutioteoria selittäisi sen miten jokin jumala on luonut elämän monimuotoisuuden.

        Minusta on aika säälittävää, että vääristelet asioita noinkin räikeästi, vaikka esiinnyt korkeasti koulutettuna ihmisenä. Se menisi vielä läpi, että vääristelet asioita omine niminesi, mutta kun sotket asiasta mielipiteitä esittäiden mielipiteet agendasi mukaiseksi, on vastenmielistä. Mutta tuotahan se aina on kun uskontoon yritetään sekoittaa tiedettä uskonnon puolustamiseksi.


      • evlut101 kirjoitti:

        "Evoluutio on yksimielisesti tieteellisesti todettu. Mitä tulee teorioihin, niin tiede korjaa niitä sitä mukaa, kun uutta tietoa saadaan. "

        Tämä on hyvä esimerkki tuosta papukaijamaisesta toistamisesta. Kun hoetaan tarpeeksi valheellista väitettä, niin valtaosa ihmisistä uskoo, vaikkei edes ymmärrä mitä evoluutioteoria tarkottaa.

        Tieteessä aniharvoin edes esiintyy yksimielisyyttä mistään asiasta eikä se ole edes tarkoitus vaan kuten tuossa alla kirjoitat, niin teorioita korjataan. Jos evoluutioteoriasta olisi yksimielisyys, niin sitä ei enää voisi korjata. Silloin se ei olisi tieteellinen teoria laisinkaan.

        Evoluutioteoriasta ei kuitenkaan esiinny yksimielisyyttä. On lukuisia evoluutiokriittisiä tieteentekijöitä listaa esim. täällä professoreita jne...
        http://www.thethirdwayofevolution.com/people
        Täällä yli 500 tieteentekijää kiistää evoluution
        https://evolutionnews.org/2006/02/over_500_scientists_proclaim_t/
        Lisäksi on paljon ihmisiä jotka uskoo Jumalan ohjaamaan evoluutioon ja se teoriana eroaa neodarvinistisesta evoluutiotoriasta, eli yksimielisiä ei olla senkään suhteen. Täältä voi lukea erilaisista prosenteista että miten missäkin uskotaan evoluutioteoriaan
        https://en.wikipedia.org/wiki/Level_of_support_for_evolution

        ”Kun hoetaan tarpeeksi valheellista väitettä, niin valtaosa ihmisistä uskoo, vaikkei edes ymmärrä mitä evoluutioteoria tarkottaa.”
        ”Jos evoluutioteoriasta olisi yksimielisyys, niin sitä ei enää voisi korjata. Silloin se ei olisi tieteellinen teoria laisinkaan.”

        Onnistut tuossa itse tekemään juurin siten kuin väität muiden tekevän. Ellei sinulla ole ymmärrystä siitä, mitä on evoluutio, niin et näemmä edes ymmärrä sitä, mitä on tieteellinen tutkimus.

        ”Lisäksi on paljon ihmisiä jotka uskoo Jumalan ohjaamaan evoluutioon ja se teoriana eroaa neodarvinistisesta evoluutiotoriasta, eli yksimielisiä ei olla senkään suhteen.”

        Kyllä, melkoinen osa kristityistä uskoo juuri noin. Osa uskoo, että Jumala on alkuun saattanut evoluution ja jotkut uskovat vieläpä siihenkin, että evoluution kautta Jumala ohjaa. Kuten asia on ilmaistu, Raamattu kertoo, miksi maailman on luotu, ja tiede sen, miten elämä kehittyy.

        Perinteiset konservatiiviset kristinuskon suuntaukset ovat siis teologisen evoluution kannalla. Sen sijaa kreationismi sekä ID -uskonto ovat uudemman ajan tuotteita fundamentalismista. Kreationistit eivät ole yksimielisiä lainkaan noista omista teorioistaan, mutta tietelijöiden taholta esiintyy huomattavan paljon suurempi konsensus evoluutiosta.


    • Onko silikonirinnat oiva esimerkki kreationismista?

      • IhminenKehittyy

        Kyllä silikonirinnat ja photoksilla täytetyt huulet on selvästi evoluutiota.


      • Epäkorrekti.totuus

        Itse asiassa silikonirinnat jatkaa siitä, mihin evoluutio lopetti.

        Vaikka #metoo aikana tätä ei ole korrektia sanoa ääneen, niin evoluutio on kehittänyt rinnat katseenvangitsijoiksi. Rinnat kun ovat enimmäkseen rasvaa eikä maitorauhasia. Minkään muun kädellislajin naarailla ei ole juuri minkäänlaisia rintoja paitsi silloin kun ne imettävät, jolloin maidon täyttämät maitorauhaset tuo rinnat jotenkin näkyviksi. Naisilla rinnat ovat korostetusti esillä murrosiästä lähtien ns. sekundaarisena sukupuoliominaisuutena.


      • Epäkorrekti.totuus kirjoitti:

        Itse asiassa silikonirinnat jatkaa siitä, mihin evoluutio lopetti.

        Vaikka #metoo aikana tätä ei ole korrektia sanoa ääneen, niin evoluutio on kehittänyt rinnat katseenvangitsijoiksi. Rinnat kun ovat enimmäkseen rasvaa eikä maitorauhasia. Minkään muun kädellislajin naarailla ei ole juuri minkäänlaisia rintoja paitsi silloin kun ne imettävät, jolloin maidon täyttämät maitorauhaset tuo rinnat jotenkin näkyviksi. Naisilla rinnat ovat korostetusti esillä murrosiästä lähtien ns. sekundaarisena sukupuoliominaisuutena.

        "...evoluutio on kehittänyt rinnat katseenvangitsijoiksi. "

        Niin, miehet ovat viehättyneet näistä lapsille tarkoitetuista kehonosista siinä määrin, että ovat kai kuvitelleen suurten tissien olevan tae siitä, että niillä ruokkii tehokkaammin? Sitten niistä kehittyi seksisymboli, eli miehet kokevat seksuaalista mielihyvää niistä.


      • viimeinen.tuomio.2
        mummomuori kirjoitti:

        "...evoluutio on kehittänyt rinnat katseenvangitsijoiksi. "

        Niin, miehet ovat viehättyneet näistä lapsille tarkoitetuista kehonosista siinä määrin, että ovat kai kuvitelleen suurten tissien olevan tae siitä, että niillä ruokkii tehokkaammin? Sitten niistä kehittyi seksisymboli, eli miehet kokevat seksuaalista mielihyvää niistä.

        Miestissit ovat myös herkät katseille :D älä vaan loukkaa miestissejä, ne ovat tehokkaan ruokinnan tulos ja jokainen mies on ne itse kasvattanut luonnollisesti, ilman mitään silikoneja! Kesäisin mies voi vapauttaa tissinsä ja se on miehen parasta aikaa!

        #manboobs forever


      • kärpäslätkät
        mummomuori kirjoitti:

        "...evoluutio on kehittänyt rinnat katseenvangitsijoiksi. "

        Niin, miehet ovat viehättyneet näistä lapsille tarkoitetuista kehonosista siinä määrin, että ovat kai kuvitelleen suurten tissien olevan tae siitä, että niillä ruokkii tehokkaammin? Sitten niistä kehittyi seksisymboli, eli miehet kokevat seksuaalista mielihyvää niistä.

        Joko sinun tissisi roikkuu polvissa?


      • Evopsy
        mummomuori kirjoitti:

        "...evoluutio on kehittänyt rinnat katseenvangitsijoiksi. "

        Niin, miehet ovat viehättyneet näistä lapsille tarkoitetuista kehonosista siinä määrin, että ovat kai kuvitelleen suurten tissien olevan tae siitä, että niillä ruokkii tehokkaammin? Sitten niistä kehittyi seksisymboli, eli miehet kokevat seksuaalista mielihyvää niistä.

        Suosittelen perehtymään evoluutiopsykologiaan. Liitän sinulle tähän tallenteen luennosta, jossa alan asiantuntija maassamme Markus J. Rantala, sekä biologian että psykologian tohtori valottaa asiaa.

        Lyhyesti kyse on siitä, että kaikkialla ihmiskunnassa seksuaalivalinnan kannalta miehiin vetoavat tietyt naisellisuudesta kertovat piirteet, kuten rinnat tai vaikka huulet. Riippuukin kulttuurista mitä piirrettä pyritään korostamaan tai jopa ylikorostamaan. Meidän kulttuurissamme se on tällä hetkellä rinnat, mutta jossain afrikkalaisheimossa venytetään huulia.

        Evoluutiopsykologisesti mielenkiintoisia seikkoja on toki muitakin. Immuniteetista kertovien ominaisuuksien merkitystä seksuaalivalinnassa ei voi sivuuttaa, ja tämän luennon jälkeen jokaisen luulisi pystyvän hoksaamaan sellaisia tekijöitä ihan itsekin - ja tämä tarjoaa hyvin paljon rahkeita miettiä miksi monet asiat ovat niin kuin ne ovat.

        youtube.com/watch?v=Hu5tQxu-kDw


      • Evopsy kirjoitti:

        Suosittelen perehtymään evoluutiopsykologiaan. Liitän sinulle tähän tallenteen luennosta, jossa alan asiantuntija maassamme Markus J. Rantala, sekä biologian että psykologian tohtori valottaa asiaa.

        Lyhyesti kyse on siitä, että kaikkialla ihmiskunnassa seksuaalivalinnan kannalta miehiin vetoavat tietyt naisellisuudesta kertovat piirteet, kuten rinnat tai vaikka huulet. Riippuukin kulttuurista mitä piirrettä pyritään korostamaan tai jopa ylikorostamaan. Meidän kulttuurissamme se on tällä hetkellä rinnat, mutta jossain afrikkalaisheimossa venytetään huulia.

        Evoluutiopsykologisesti mielenkiintoisia seikkoja on toki muitakin. Immuniteetista kertovien ominaisuuksien merkitystä seksuaalivalinnassa ei voi sivuuttaa, ja tämän luennon jälkeen jokaisen luulisi pystyvän hoksaamaan sellaisia tekijöitä ihan itsekin - ja tämä tarjoaa hyvin paljon rahkeita miettiä miksi monet asiat ovat niin kuin ne ovat.

        youtube.com/watch?v=Hu5tQxu-kDw

        ”Suosittelen perehtymään evoluutiopsykologiaan.”

        Kyllä, olen seurannut sitä vuosikymmenien ajan. Näitä eri näkemyksiä on liikkunut paljon, ja niitäkin on hyvä kyseenalaistaa.

        Kun aikoinaan Elaine Morgan 70 luvulla toi keskusteluun ns. vesiapinateorian, ja siitä syntyi melkoista keskustelua, kiinnostuin asiasta. Kieltämättä se oli hauskaa seurattavaa.

        Kirjoitin, että miehet ”…ovat kai kuvitelleen suurten tissien olevan tae siitä, että niillä ruokkii tehokkaammin?”. Eli jokainen kulttuuri on luonut omia mielikuviaan näistä sen mukaan, mitä on arvostettu milloinkin.

        Kyse on seksuaalivalinnan teoriasta, jossa on ikävä kyllä mielestäni välillä melkoisia puutteita. Jos kaikkea tarkastellaan vain länsimaisten miesten mieltymysten kautta, väkisinkin siitä syntyy virhepäätelmiä.

        Kysymys siis kuuluu – miksi miehiä viehättää joku yksittäinen osa naisen kehossa? Miksi miehet pitävä suuria tissejä viehättävinä? On jopa esitetty, että miehet taantuvat imeväisikäisen asteelle, ja kaihoavat sitä tunnetta, mitä vauvoina saivat äitinsä rinnoilla olleessaan. Eräs toinen selitys on, että ellei poikalasta ole imetetty riittävästi, hän jää ikuisesti kaipaamaan tuota olotilaa. No, joo. Nämä nyt vain eräinä esimerkkeinä.

        ”Immuniteetista kertovien ominaisuuksien merkitystä seksuaalivalinnassa ei voi sivuuttaa…”

        Tuossa sen sijaan on mieltä, sillä nuo ominaisuudet ovat usein ”näkymättämiä”, eikä niiden perusteella voi siis muokata omaa kehoaan siten, kuin toivotaan.

        Mutta se vanha sanonta pitää yhä paikkansa. Mitä yhteistä on naiset tisseillä ja pienoisjunaradalla? Molemmat ovat tarkoitettu lapsille, mutta miehet niillä leikkii.


    • kehitys.kulkee.eteenpäin

      Fundamentalistien kohdalla evoluutio loppui jo noin 6000 vuotta sitten ja siksi he ovat hieman kehittymättömämpiä kuin muut.

    • Miten tällainen kysymys liittyy luterilaisuuteen?
      Pohtiko esim. Luther mahdollista apinaisuuttaan?

      • Aihevapaa_palsta

        Melkein mikään tällä palstalla ei koskaan liity juuri luterilaisuuteen!

        Yleisesti ottaen tämä palsta on kuin Maailman menoa -palsta, missä erilaisista tapahtumista avautuen ihmiset valottavat omaa arvomaailmaansa. Ainoa kytkös luterilaisuuteen tässä kaikessa on se, että luterilainen kristinusko jollain tasolla katsotaan maamme valtauskonnoksi riippumatta siitä, uskotaanko siihen aidosti vai ollaanko menossa mukana vain tavan vuoksi.

        Tämä on siis vähän sellainen yleinen kansanpalsta, missä erilaiset tahot propagoivat ideaalejaan. Klassiset kiistanaiheet määrittelvät jakolinjan fundamentalismin ja vapaamuotoisemman kristinuskon tulkinnan välillä aivan kuten konservatismin ja liberalismin välillä. Myös poliittinen vasemmisto-oikeisto-jako näkyy vahvasti.

        Perinteisiä aiheita ovat siis geopolitiikka (kuten Israel vs. palestiina), talous- ja sosiaalipolitiikka, maahanmuutto, globalisaatio ja kulttuuri, perinteiset vs. modernit ihmis- ja yhteisökäsitykset, tasa-arvo, sukupuoli- ja seksuaalisuusteemat, abortti, evoluutio ja luominen, myytit kuten vedenpaisumus jne. Mukaan heitetään vielä vähän kaste- ja sapattiriitelyä höysteeksi. Näistä sitten väännetään yötäpäivää, viikosta toiseen, vuoden jokaisena kuukautena, vuosikausia tauotta ja samaa jankutusmyllyä uutterasti pyörittäen.

        Yleensä teema on jokin päivänpolttama aihe. Jos sellaista aihetta ei oikein ole, niin aihevalikoima muodostuu satunnaisesti sitä mukaa mitä jokin yleensä sitä uskonnollista äärilaitaa edustava trolli on viimeksi suosikkiapologiamediastaan lukenut, eli tarjolla on jotain lähtien Patmoksen poliitiikasta luomissivustojen evoluutionkiistolistauksiin.

        Ja mitään väliä ei ole sillä, että nämä kaikki asiat on aivan puhkikäsiteltyjä! Uudestaan vaan, tempaukseen saakka!


    • Eikiitosjeesus

      Ihmisen evoluutio loppuu kun viimeinen lajiin kuuluva yksilö kuolee.
      Miksi kysellä itsestäänselvyyksiä.

      • Muu evoluutio silti jatkuu maapallolla meidänkin jälkeemme, tilalle voi tulla uusia lajeja joiden kehityksen tiellä olemme olleet. Meidän nisäkkkäiden kehitystähän auttoi suuresti, ellei ratkaisevasti, dinosaurusten tuho asteroidiräjähdyksessä.

        Lopulta pallomme katoaa muun aurinkokuntamme kera auringon palaessa loppuun. Se on evoluution loppu ainakin tässä aurinkokunnassa - paitsi jos jotain elävää kuitenkin jossain muodossa jää jäljelle ja siirtyy muualle. Mistä senkään tietää?


      • dikduk kirjoitti:

        Muu evoluutio silti jatkuu maapallolla meidänkin jälkeemme, tilalle voi tulla uusia lajeja joiden kehityksen tiellä olemme olleet. Meidän nisäkkkäiden kehitystähän auttoi suuresti, ellei ratkaisevasti, dinosaurusten tuho asteroidiräjähdyksessä.

        Lopulta pallomme katoaa muun aurinkokuntamme kera auringon palaessa loppuun. Se on evoluution loppu ainakin tässä aurinkokunnassa - paitsi jos jotain elävää kuitenkin jossain muodossa jää jäljelle ja siirtyy muualle. Mistä senkään tietää?

        Dinosaurukset elivät samaan aikaan ihmisten kanssa ja kuolivat lajin häviämiseen. Se olisi edessä myös meillä ihmisillä, sillä perimämme rappeutuu. Jokainen sukupolvi on heikompi kuin aiempi sukupolvi. Perimän rappeutumisen huomaa muuten selkeästi esim. koirien jalostuksessa. Voimakkaasti jalostetut suosikkikoirarodut ovat todella sairaita.


      • Yksi_usko kirjoitti:

        Dinosaurukset elivät samaan aikaan ihmisten kanssa ja kuolivat lajin häviämiseen. Se olisi edessä myös meillä ihmisillä, sillä perimämme rappeutuu. Jokainen sukupolvi on heikompi kuin aiempi sukupolvi. Perimän rappeutumisen huomaa muuten selkeästi esim. koirien jalostuksessa. Voimakkaasti jalostetut suosikkikoirarodut ovat todella sairaita.

        Lisään. On selvää, että ihminen ei ole sen vanhempi kuin mitä Raamatussa kerrotaan. Jos ihminen olisi kehittynyt kehitysopin mukaisesti apinasta, niin ihminen olisi apinaa alkeellisempi ellei olisi kuollut moneen kertaan sitä ennen sukupuuttoon. Sori vaan, mutta tämä on tämän päivän tiedettä, joka perustuu faktoihin eli havaintoihin. Se on kaukana siitä, mitä vanha evoluutioteoria väittää, jolla on vuosikymmeniä aivopesty meitä.


      • Yksi_usko kirjoitti:

        Dinosaurukset elivät samaan aikaan ihmisten kanssa ja kuolivat lajin häviämiseen. Se olisi edessä myös meillä ihmisillä, sillä perimämme rappeutuu. Jokainen sukupolvi on heikompi kuin aiempi sukupolvi. Perimän rappeutumisen huomaa muuten selkeästi esim. koirien jalostuksessa. Voimakkaasti jalostetut suosikkikoirarodut ovat todella sairaita.

        Dinosaurukset kuolivat sukupuuttoon yli 60 miljoonaa vuotta sitten joten ei nyt ihan samaan aikaan olla eletty!
        Minulla on Neanderthal-esi-isänä perimää ja tuo ihmislaji laji johon kuuluvien miesten kanssa jotkut esiäidestäni pariutuivat kuoli sukupuuttoon yli 20 000 vuotta sitten joten jo oma perimämme kertoo aivan varmasti muuta lajimme iästä


      • dikduk kirjoitti:

        Dinosaurukset kuolivat sukupuuttoon yli 60 miljoonaa vuotta sitten joten ei nyt ihan samaan aikaan olla eletty!
        Minulla on Neanderthal-esi-isänä perimää ja tuo ihmislaji laji johon kuuluvien miesten kanssa jotkut esiäidestäni pariutuivat kuoli sukupuuttoon yli 20 000 vuotta sitten joten jo oma perimämme kertoo aivan varmasti muuta lajimme iästä

        Rakas ystävä. Voin sanoa sinulle suoraan: Olet väärässä. Voit lukea Marvin L. Lubenowin kirjasta Myytti apinaihmisistä sivut 180-224. En jaksa alkaa niitä tähän kopioimaan.


      • Yksi_usko kirjoitti:

        Rakas ystävä. Voin sanoa sinulle suoraan: Olet väärässä. Voit lukea Marvin L. Lubenowin kirjasta Myytti apinaihmisistä sivut 180-224. En jaksa alkaa niitä tähän kopioimaan.

        Olen lukenut alan asiantuntijoiden teoksia, muuhun ei riitä aika eikä mielenkiinto. "Lubenow, M.S., Th.M. Marvin Lubenow is Professor of Bible/ Apologetics at Christian Heritage College in San Diego California. " Tässä on tiede vs tietyt kristilliset piirit .

        Neanderthalin ihmisillä ja omalla lajillemme oli hiukan erilainen perimä, minulla on 3% perimästäni tuon toisen ihmislajin perimää. Suurin piirtein sama määrä heidän perimäänsä on myös muillakin Afrikasta aikoinaan lähteneillä populaatioilla, Afrikkaan jääneillä ei sitä ole .
        Kun tunnetan lajimme perimä ja neanderthalilaisten perimä, voidaan kustakin DNA-näytteen antaneesta henkilöstä nähdä paljonko tällä on tätä toisen rinnakkain lajimme kanssa Euroopassa ja lähi-idässä eläneen ihmislajin perimää .


    • kahviako

      ameba.ja sil ei ollu ku tuo 1solu,mik on tälläkin eläinlajilla ku ihmiseksi,kutsuttu.aivoissaan on 1 mut mikä.mukillinen kahviako?!

    • Jeesus_tulee_pian

      Evoluutio-oppi on silkka valhe ja on johtanut lukemattomat ihmiset ikuiseen kadotukseen. Tämä maailma ei ole syntynyt itsestään tyhjästä ja sitten luonut monimuotoista täydellistä ekosysteemiä. Ei. Jumala loi maailman ja ihmiset ja kaiken olevan. Ihminen on kuudessatuhannessa vuodessa pahasti rappeutunut. Olemme paljon pienikokoisempi ja elämme hyvin paljon lyhyemmän elämän kuin ensimmäiset ihmiset. Elämme tämän maailmankauden viimeisiä hetkiä. Jeesus tulee pian noutamaan omansa pois ja seuraa seitsemän kauhun vuotta, ja sen jälkeen Jeesuksen tuhatvuotinen valtakunta maan päällä.

      • Hitaat.kiireet

        Tarkoittaako "pian" sataa, tuhatta vai kahtatuhatta vuotta?

        Jeesushan on ollut palaamassa "pian" jo 2000 vuotta, mutta hitaita ovat taivaalliset kiireet.


      • SBRSG
        Hitaat.kiireet kirjoitti:

        Tarkoittaako "pian" sataa, tuhatta vai kahtatuhatta vuotta?

        Jeesushan on ollut palaamassa "pian" jo 2000 vuotta, mutta hitaita ovat taivaalliset kiireet.

        2000 vuotta on ikuisuuten verrattuna mitätön aika.


      • "Ihminen on kuudessatuhannessa vuodessa pahasti rappeutunut. "

        Kiitos päivän huumoripläjäyksestä. Miten muuten rappeutuminen on sitten näkyvissä?


      • naurattaa.taas
        mummomuori kirjoitti:

        "Ihminen on kuudessatuhannessa vuodessa pahasti rappeutunut. "

        Kiitos päivän huumoripläjäyksestä. Miten muuten rappeutuminen on sitten näkyvissä?

        Eikö mummomouri ole lukenut Raamattua?


      • naurattaa.taas kirjoitti:

        Eikö mummomouri ole lukenut Raamattua?

        Voistiko kertoa, mitä sinä "rappeutumisella" tarkoitat?


      • mummomuori kirjoitti:

        Voistiko kertoa, mitä sinä "rappeutumisella" tarkoitat?

        Rappeutumisella tarkoitetaan sitä, että perimä heikkenee. Tulee enemmän ja enemmän uusia sairauksia ym. Totuus on, että ihminen ei ole voinut nykytieteen faktojen perusteella kehittyä apinasta.


      • Katsopa tilastoja: elinikä on pidentynyt , pituuskin on kasvanut sekä naisilla että miehillä , sairauksia voidaan hoitaa aivan toisin kuin ennen. Perimäkään ei ole heikentynyt.

        Ihmisellä ja ja apinoilla on yhteinen esi-isä sen tiedämme juuri nykytieteen faktoista.


      • dikduk kirjoitti:

        Katsopa tilastoja: elinikä on pidentynyt , pituuskin on kasvanut sekä naisilla että miehillä , sairauksia voidaan hoitaa aivan toisin kuin ennen. Perimäkään ei ole heikentynyt.

        Ihmisellä ja ja apinoilla on yhteinen esi-isä sen tiedämme juuri nykytieteen faktoista.

        Lääketiede on kehittynyt, syömme terveellisemmin ja elämme fyysisesti helpompaa elämää kuin ennen. Eiköhän siinä ole lääkkeet eliniän pidentymiseen ja hienoiseen pituuden kasvuunkin. Sen sijaan valitettavasti perimä heikkenee, eikä sille mahda mitään. Mitä taas tulee yhteiseen esi-isään, niin se on täysin fiktiota.


      • Yksi_usko kirjoitti:

        Lääketiede on kehittynyt, syömme terveellisemmin ja elämme fyysisesti helpompaa elämää kuin ennen. Eiköhän siinä ole lääkkeet eliniän pidentymiseen ja hienoiseen pituuden kasvuunkin. Sen sijaan valitettavasti perimä heikkenee, eikä sille mahda mitään. Mitä taas tulee yhteiseen esi-isään, niin se on täysin fiktiota.

        Mikä ihmeen perimä se heikkenee...


    • viimeinen.tuomio.2

      Ihmisen evoluutio loppui Jeesukseen,

      Vai onko joku toinen syntynyt neitseellisesti, kuollut, ja ylösnoussut Jumalan Poikana ja on jatkuva puheenaihe vielä parintuhannen vuoden päästäkin? Se on aika korkea rima enää kehittyä siitä eteenpäin...

      • Evoluutio tuossa tosin on ollut teologinen , ei biologinen. Jos joku uskotaan alkuperältään sellaiseksi että äiti on ollut ihmisnainen mutta isä jumala, ei ole kyse lainkaan lajiimme kuuluneesta ihmisestä.
        Jotta voi määritellä tällaisen hahmon sen joutuu tekemään pelkästään teologisesti jossa tämä hahmo paikannetaankin jumalmaailmaan välivaiheena olleen inkarnaation jälkeen .


      • viimeinen.tuomio.2
        dikduk kirjoitti:

        Evoluutio tuossa tosin on ollut teologinen , ei biologinen. Jos joku uskotaan alkuperältään sellaiseksi että äiti on ollut ihmisnainen mutta isä jumala, ei ole kyse lainkaan lajiimme kuuluneesta ihmisestä.
        Jotta voi määritellä tällaisen hahmon sen joutuu tekemään pelkästään teologisesti jossa tämä hahmo paikannetaankin jumalmaailmaan välivaiheena olleen inkarnaation jälkeen .

        Ja siinä tuleekin sitten jo sekä teologille että biologille pakko tunnustaa jumalmaailman olemassaolo ja ne pitävät todisteet jumalista puuttuvat kummaltakin, vaikka jumalia sinänsä on tuhansia ja tuhansia kautta ihmisen historian ja nekin tarinat ovat kokeneet evoluutiota, samoin kuin Raamattu, jota on käännetty ja muokattu ja paranneltu kuhunkin aikaan sopivaksi (sekin on siis kokenut evoluutiota ajan myötä).


    • Mikä ihmeteko on ihmislajin evoluution pysäyttänyt?

      • Demokratian.puolesta

        Lääketiede varmasti hidastaa evoluutiota ainakin joiltain osin.

        Yksi ihmisen geenipooliin vaikuttava piirre on se, että "populaatiot" (eli valtiot tai kulttuurit) jotka kouluttavat naiset, lisääntyvät hitaammin kuin kulttuurit joissa naisen asema on alistetumpi. Tästä seuraa vääjäämättä se, että korkean syntyvyyden "populaatiot" vuotavat matalan syntyvyyden maihin. Esimerkiksi Suomen maahanmuutto ratkaistaan synnytyssaleissa eikä sillä äänestetäänkö äärioikeistoa vai "suvakkeja".

        Suomalaiset äänestävät genitaaleillaan maahanmuuton puolesta.

        En väitä, että kulttuuri olisi geneettinen ominaisuus, mutta toisaalta on selvää, että liberaalit demokratiat eivät geenipooliltaan ole samanlaisia kuin kehitysmaat.

        Huolestuttava kysymys on, pystyykö liberaalit demokriat säilyttämään kulttuurin, johon sen arvot pohjautuvat ja länsimaistamaan maahanmuuttajat, joita väistämättä tulee, kun syntyvyys on näin alhaisella tasolla kuin se Länsi-Euroopassa on.


      • kljlklkjlk
        Demokratian.puolesta kirjoitti:

        Lääketiede varmasti hidastaa evoluutiota ainakin joiltain osin.

        Yksi ihmisen geenipooliin vaikuttava piirre on se, että "populaatiot" (eli valtiot tai kulttuurit) jotka kouluttavat naiset, lisääntyvät hitaammin kuin kulttuurit joissa naisen asema on alistetumpi. Tästä seuraa vääjäämättä se, että korkean syntyvyyden "populaatiot" vuotavat matalan syntyvyyden maihin. Esimerkiksi Suomen maahanmuutto ratkaistaan synnytyssaleissa eikä sillä äänestetäänkö äärioikeistoa vai "suvakkeja".

        Suomalaiset äänestävät genitaaleillaan maahanmuuton puolesta.

        En väitä, että kulttuuri olisi geneettinen ominaisuus, mutta toisaalta on selvää, että liberaalit demokratiat eivät geenipooliltaan ole samanlaisia kuin kehitysmaat.

        Huolestuttava kysymys on, pystyykö liberaalit demokriat säilyttämään kulttuurin, johon sen arvot pohjautuvat ja länsimaistamaan maahanmuuttajat, joita väistämättä tulee, kun syntyvyys on näin alhaisella tasolla kuin se Länsi-Euroopassa on.

        "En väitä, että kulttuuri olisi geneettinen ominaisuus, mutta toisaalta on selvää, että liberaalit demokratiat eivät geenipooliltaan ole samanlaisia kuin kehitysmaat."

        Onko se selvää? Ihmiskunnan geneettinen homogeenisuus on huomattavan suurta. Kaksi samalla alueella elävää simpanssilaumaa poikkeavat geneettisesti toisistaan enemmän kuin mistään ihmispopulaatioista löytyy epähomogeenisuutta.

        Kehitysmaiden asukkaiden tilanne johtuu nähdäkseni heikosta koulutuksesta, yhteiskunnan asenteista ja ääriuskonnollisesta konservatismista enemmän kuin geeneistä.


      • Demokratian.puolesta
        kljlklkjlk kirjoitti:

        "En väitä, että kulttuuri olisi geneettinen ominaisuus, mutta toisaalta on selvää, että liberaalit demokratiat eivät geenipooliltaan ole samanlaisia kuin kehitysmaat."

        Onko se selvää? Ihmiskunnan geneettinen homogeenisuus on huomattavan suurta. Kaksi samalla alueella elävää simpanssilaumaa poikkeavat geneettisesti toisistaan enemmän kuin mistään ihmispopulaatioista löytyy epähomogeenisuutta.

        Kehitysmaiden asukkaiden tilanne johtuu nähdäkseni heikosta koulutuksesta, yhteiskunnan asenteista ja ääriuskonnollisesta konservatismista enemmän kuin geeneistä.

        Samaa mieltä. Ihmiset ovat perimältään harvinaisen yhtenäinen laji. Geenit eivät ratkaise kulttuuria, mutta kyllähän esimerkiksi immuunijärjestelmämme (johon evoluution vaikutus on ollut suurin ihmiskunnan levittäytymisen jälkeen) eroavat eri populaatioissa.


      • Demokratian.puolesta kirjoitti:

        Lääketiede varmasti hidastaa evoluutiota ainakin joiltain osin.

        Yksi ihmisen geenipooliin vaikuttava piirre on se, että "populaatiot" (eli valtiot tai kulttuurit) jotka kouluttavat naiset, lisääntyvät hitaammin kuin kulttuurit joissa naisen asema on alistetumpi. Tästä seuraa vääjäämättä se, että korkean syntyvyyden "populaatiot" vuotavat matalan syntyvyyden maihin. Esimerkiksi Suomen maahanmuutto ratkaistaan synnytyssaleissa eikä sillä äänestetäänkö äärioikeistoa vai "suvakkeja".

        Suomalaiset äänestävät genitaaleillaan maahanmuuton puolesta.

        En väitä, että kulttuuri olisi geneettinen ominaisuus, mutta toisaalta on selvää, että liberaalit demokratiat eivät geenipooliltaan ole samanlaisia kuin kehitysmaat.

        Huolestuttava kysymys on, pystyykö liberaalit demokriat säilyttämään kulttuurin, johon sen arvot pohjautuvat ja länsimaistamaan maahanmuuttajat, joita väistämättä tulee, kun syntyvyys on näin alhaisella tasolla kuin se Länsi-Euroopassa on.

        ”Lääketiede varmasti hidastaa evoluutiota ainakin joiltain osin.”

        Ei se mihinkään hidastu, kehitys kulkee koko ajan eteenpäin. Lääketiede voi muuttaa välillä sen suuntaa.

        ”Esimerkiksi Suomen maahanmuutto ratkaistaan synnytyssaleissa…”

        :D Ei todellakaan ratkaista. Ei se vähennä muualla syntyvyyttä, ei poista elinolojen kurjuutta eikä etenkään poista sotia yms.

        ”Huolestuttava kysymys on, pystyykö liberaalit demokriat säilyttämään kulttuurin, johon sen arvot pohjautuvat…”

        Jos vähän historiaa tarkkailee, niin demokratian voi katsoa alkaneen 1700 luvun lopulla. Vaikka maailmalla on tapahtunut paljon yrityksiä palata takaisin vanhaan, niin silti se on aina saanut lopulta valta-aseman. Hyvin moni sellaisista maista tulevat, joissa demokratiaa ja vapautta ei ole, tulevat juuri siksi Eurooppaan sekä Yhdysvaltoihin. Tai siten saavat pikkuhiljaa omiin maihinsa demokraattisen yhteiskunnan.


      • evlut101
        kljlklkjlk kirjoitti:

        "En väitä, että kulttuuri olisi geneettinen ominaisuus, mutta toisaalta on selvää, että liberaalit demokratiat eivät geenipooliltaan ole samanlaisia kuin kehitysmaat."

        Onko se selvää? Ihmiskunnan geneettinen homogeenisuus on huomattavan suurta. Kaksi samalla alueella elävää simpanssilaumaa poikkeavat geneettisesti toisistaan enemmän kuin mistään ihmispopulaatioista löytyy epähomogeenisuutta.

        Kehitysmaiden asukkaiden tilanne johtuu nähdäkseni heikosta koulutuksesta, yhteiskunnan asenteista ja ääriuskonnollisesta konservatismista enemmän kuin geeneistä.

        "Onko se selvää? Ihmiskunnan geneettinen homogeenisuus on huomattavan suurta. Kaksi samalla alueella elävää simpanssilaumaa poikkeavat geneettisesti toisistaan enemmän kuin mistään ihmispopulaatioista löytyy epähomogeenisuutta. "

        Totta ja tämäkin puhuu sen puolesta, että ihmiskunta ei ole kehittynyt apinoista. Jos olisi, niin eri maanosissa eläneet kansat olisivat geneettisesti hyvin erilaisia.

        Evolutionistit yrittävät selittää asiaa ns. pullonkaulateorialla eli että jossain (nykyarvauksen mukaan Afrikassa) olisi jäänyt vain muutaman tuhannen ihmisyksilön lauma eloon. Eli että kaikki muut siihen asti kehittyneet ihmisen kaltaiset apinat ja puoliapinat olisi noin 100 000 vuotta sitten kuolleet sukupuuttoon. Jos kuitenkin olisi tullut jokin globaali katastrofi, joka tappaa niin Aasiassa kuin Euroopassakin, niin miksi muut eläinlajit eivät kulkeneet saman pullonkaulan kautta? Ja miksi esim. simpanssi jäi eloon kuten oranki, mutta jokin joka on lähempänä ihmistä kuoli viimeistä yksilöä myöten? No arvatahan aina saa, mutta aika suuria uskonhyppyjä joutuu evoluutiouskovainen tässä ottamaan.


      • capiche
        evlut101 kirjoitti:

        "Onko se selvää? Ihmiskunnan geneettinen homogeenisuus on huomattavan suurta. Kaksi samalla alueella elävää simpanssilaumaa poikkeavat geneettisesti toisistaan enemmän kuin mistään ihmispopulaatioista löytyy epähomogeenisuutta. "

        Totta ja tämäkin puhuu sen puolesta, että ihmiskunta ei ole kehittynyt apinoista. Jos olisi, niin eri maanosissa eläneet kansat olisivat geneettisesti hyvin erilaisia.

        Evolutionistit yrittävät selittää asiaa ns. pullonkaulateorialla eli että jossain (nykyarvauksen mukaan Afrikassa) olisi jäänyt vain muutaman tuhannen ihmisyksilön lauma eloon. Eli että kaikki muut siihen asti kehittyneet ihmisen kaltaiset apinat ja puoliapinat olisi noin 100 000 vuotta sitten kuolleet sukupuuttoon. Jos kuitenkin olisi tullut jokin globaali katastrofi, joka tappaa niin Aasiassa kuin Euroopassakin, niin miksi muut eläinlajit eivät kulkeneet saman pullonkaulan kautta? Ja miksi esim. simpanssi jäi eloon kuten oranki, mutta jokin joka on lähempänä ihmistä kuoli viimeistä yksilöä myöten? No arvatahan aina saa, mutta aika suuria uskonhyppyjä joutuu evoluutiouskovainen tässä ottamaan.

        Jos mennään kymmeniä tuhansia vuosia taaksepäin, niin silloin kieltämättä mennään oletuksien ja uskon alueelle. En kuitenkaan ymmärrä, miksi jotkut kristityt ovat tehneet evoluution kieltämisestä uskonoppia?


      • evlut101
        capiche kirjoitti:

        Jos mennään kymmeniä tuhansia vuosia taaksepäin, niin silloin kieltämättä mennään oletuksien ja uskon alueelle. En kuitenkaan ymmärrä, miksi jotkut kristityt ovat tehneet evoluution kieltämisestä uskonoppia?

        Hyvä kysymys. Itse asiassa evoluutio ei ole minulle uskonnollinen kysymys vaan tieteellinen. Harmillisesti vain kristityt ovat niitä jotka uskaltavat kyseenalaistaa sen epätieteellisyyden ja epäloogiset kohdat.


      • evlut101 kirjoitti:

        "Onko se selvää? Ihmiskunnan geneettinen homogeenisuus on huomattavan suurta. Kaksi samalla alueella elävää simpanssilaumaa poikkeavat geneettisesti toisistaan enemmän kuin mistään ihmispopulaatioista löytyy epähomogeenisuutta. "

        Totta ja tämäkin puhuu sen puolesta, että ihmiskunta ei ole kehittynyt apinoista. Jos olisi, niin eri maanosissa eläneet kansat olisivat geneettisesti hyvin erilaisia.

        Evolutionistit yrittävät selittää asiaa ns. pullonkaulateorialla eli että jossain (nykyarvauksen mukaan Afrikassa) olisi jäänyt vain muutaman tuhannen ihmisyksilön lauma eloon. Eli että kaikki muut siihen asti kehittyneet ihmisen kaltaiset apinat ja puoliapinat olisi noin 100 000 vuotta sitten kuolleet sukupuuttoon. Jos kuitenkin olisi tullut jokin globaali katastrofi, joka tappaa niin Aasiassa kuin Euroopassakin, niin miksi muut eläinlajit eivät kulkeneet saman pullonkaulan kautta? Ja miksi esim. simpanssi jäi eloon kuten oranki, mutta jokin joka on lähempänä ihmistä kuoli viimeistä yksilöä myöten? No arvatahan aina saa, mutta aika suuria uskonhyppyjä joutuu evoluutiouskovainen tässä ottamaan.

        Kun Suomessa oli nälänhätä 1800- luvulla kuolivatko luonnossa elävät eläimetkin joiden ravintoketju ym oli toinen kuin meidän?
        Meillä ihmisilä ja ja apinoilla jotka nyt elävät on ollut yhteinen esi-isä, mutta sen paremmin me ihmiset emme ole kehittyneet apinoista kuin apinat ihmisistä, kumpikin laji on mennyt omaan suuntaansa yhteisestä lähtökohdasta .Kaikilla nyt elävillä lajeilla on yhteinen alkuperä ja olemme sukua niin valaille kuin orangeille.


      • Olet_ihan_pihalla
        evlut101 kirjoitti:

        "Onko se selvää? Ihmiskunnan geneettinen homogeenisuus on huomattavan suurta. Kaksi samalla alueella elävää simpanssilaumaa poikkeavat geneettisesti toisistaan enemmän kuin mistään ihmispopulaatioista löytyy epähomogeenisuutta. "

        Totta ja tämäkin puhuu sen puolesta, että ihmiskunta ei ole kehittynyt apinoista. Jos olisi, niin eri maanosissa eläneet kansat olisivat geneettisesti hyvin erilaisia.

        Evolutionistit yrittävät selittää asiaa ns. pullonkaulateorialla eli että jossain (nykyarvauksen mukaan Afrikassa) olisi jäänyt vain muutaman tuhannen ihmisyksilön lauma eloon. Eli että kaikki muut siihen asti kehittyneet ihmisen kaltaiset apinat ja puoliapinat olisi noin 100 000 vuotta sitten kuolleet sukupuuttoon. Jos kuitenkin olisi tullut jokin globaali katastrofi, joka tappaa niin Aasiassa kuin Euroopassakin, niin miksi muut eläinlajit eivät kulkeneet saman pullonkaulan kautta? Ja miksi esim. simpanssi jäi eloon kuten oranki, mutta jokin joka on lähempänä ihmistä kuoli viimeistä yksilöä myöten? No arvatahan aina saa, mutta aika suuria uskonhyppyjä joutuu evoluutiouskovainen tässä ottamaan.

        Miksi sinä jatkat tuota saman asian valehtelua kun sinulle on jo kerrottu täällä, ettei asia mene noin kuin esität? Sinulle on selostettu täällä, etteivät "evolutionistit" väitä tuommoista. Nyt keksit vielä uusia höpöjuttuja puoliapinoista. Katsohan mitä puoliapina tarkoittaa niin ymmärrät miten lööperiä puhut. Ei kukaan väitä, että puoliapinat olisivat hävinneet tuolloin! Eikä kukaan siis väitä, että kaikki nuo muutkaan olisivat hävinneet tuolloin. Minusta on pöyristyttävää, että aikuinen ihminen joka täällä esittää olevansa joku tohtori vielä, viitsii tuollaista lapsellista höperehtimistä toistella sen jälkeen kun häntä tässä asiassa valistuneemmat ovat häntä jo moneen kertaan ojentaneet.


    • Pudis55

      On aika hupaisaa että henkilöt jotka ovat lukeneet ainoastaan raamattua eivät sitäkään ole ymmärtäneet, kyseenalaistavat tuhansien ja taas tuhansien vakavaa tiedettä tehneiden ihmisten suurenmoiset saavutukset. Olen jo aikaisemmin pyytänyt että tämän lapsellisen länkytyksen sijaan käärisitte hihat ja esittäisitte kiistattomat todisteet jumalasta ja tästä luomiskertomuksesta. Aikaa ja rahaahan siihen menee ja osaamistakin pitäisi olla mutta onhan meillä aikaa Jesseä odotellessa. Rahat saatte varmaan kerättyä kolehdin muodossa helposti. Kannattaa muistaa että totuus maksaa mutta valhe on ilmaista. Käsittääkseni ette tähän mennessä ole montakaan ropoa laittaneet likoon luomiskertomuksen puolesta, vain mitä?

      • Tänä päivänä tieteessä eletään todellakin aivan uutta aikaa. Kaikki tieteelliset faktat vahvistavat Jumalan olemassa oloa.


      • Pudis55
        Yksi_usko kirjoitti:

        Tänä päivänä tieteessä eletään todellakin aivan uutta aikaa. Kaikki tieteelliset faktat vahvistavat Jumalan olemassa oloa.

        En viitsi edes vastata.


      • evlut101

        "On aika hupaisaa että henkilöt jotka ovat lukeneet ainoastaan raamattua eivät sitäkään ole ymmärtäneet, kyseenalaistavat tuhansien ja taas tuhansien vakavaa tiedettä tehneiden ihmisten suurenmoiset saavutukset."

        Minusta taas on hupaisaa, että useimmat evoluutioteorian kannattajat eivät edes osaa nimetä yhtäkään selvää todistetta siitä että evoluutio olisi tosi. Silti samat tyypit öyhöttää siitä kuinka kristinuskolle ei löydy mitään todistetta.

        Ateistit naureskelevat uskovien "uskonhypyille", mutta syyllistyvät vielä pahempiin uskonhyppyihin uskoessaan että materia järjestyi itsestään alkuräjähdyksen jälkeen auringoiksi, planeetoiksi ja lopulta supermonimutkaisiksi biologisiksi olennoiksi. Maan tomu kirjaimellisesti järjestyi elämäksi itsestään. Kuinka hullu ja epälooginen päätelmä.


      • evlut101 kirjoitti:

        "On aika hupaisaa että henkilöt jotka ovat lukeneet ainoastaan raamattua eivät sitäkään ole ymmärtäneet, kyseenalaistavat tuhansien ja taas tuhansien vakavaa tiedettä tehneiden ihmisten suurenmoiset saavutukset."

        Minusta taas on hupaisaa, että useimmat evoluutioteorian kannattajat eivät edes osaa nimetä yhtäkään selvää todistetta siitä että evoluutio olisi tosi. Silti samat tyypit öyhöttää siitä kuinka kristinuskolle ei löydy mitään todistetta.

        Ateistit naureskelevat uskovien "uskonhypyille", mutta syyllistyvät vielä pahempiin uskonhyppyihin uskoessaan että materia järjestyi itsestään alkuräjähdyksen jälkeen auringoiksi, planeetoiksi ja lopulta supermonimutkaisiksi biologisiksi olennoiksi. Maan tomu kirjaimellisesti järjestyi elämäksi itsestään. Kuinka hullu ja epälooginen päätelmä.

        Evoluutio ei kerro miten elämä on syntynyt vaan miten elämä tällä maapallolla on kehittynyt . Se on aikojen kuluessa kehittynyt sinuksi ja minuksi , koivuiksi ja ketuiksi, luteiksi ja lapamadoiksi,koko sen elämän lajikirjoksi joka täällä nyt on ja jonka tiedämme aiemmin olleen.


      • evlut101
        dikduk kirjoitti:

        Evoluutio ei kerro miten elämä on syntynyt vaan miten elämä tällä maapallolla on kehittynyt . Se on aikojen kuluessa kehittynyt sinuksi ja minuksi , koivuiksi ja ketuiksi, luteiksi ja lapamadoiksi,koko sen elämän lajikirjoksi joka täällä nyt on ja jonka tiedämme aiemmin olleen.

        Totta ja siksi se ei kelpaakaan maailman tai edes elämän syntyteoriaksi. Se kattaa elämän synnyn ja kehittymisen kannalta vain lyhyen pätkän ja jättää loput täydelliseen pimentoon. Tosin koulujen biologienkirjoissa opetetaan edelleen että salama iski alkulampeen, jossa sitten syntyi itsestään elävä solu. Että en sitten tiedä että haluaako Suomen biologiyhteisö tehdä itsestään tässä naurunalaisen esittäessään täysin naurettavia ideoita, jota on verrattavissa johonkin neukkujen propagandaan.


      • evlut101 kirjoitti:

        Totta ja siksi se ei kelpaakaan maailman tai edes elämän syntyteoriaksi. Se kattaa elämän synnyn ja kehittymisen kannalta vain lyhyen pätkän ja jättää loput täydelliseen pimentoon. Tosin koulujen biologienkirjoissa opetetaan edelleen että salama iski alkulampeen, jossa sitten syntyi itsestään elävä solu. Että en sitten tiedä että haluaako Suomen biologiyhteisö tehdä itsestään tässä naurunalaisen esittäessään täysin naurettavia ideoita, jota on verrattavissa johonkin neukkujen propagandaan.

        "Totta ja siksi se ei kelpaakaan maailman tai edes elämän syntyteoriaksi. Se kattaa elämän synnyn ja kehittymisen kannalta vain lyhyen pätkän ja jättää loput täydelliseen pimentoon. "
        Eihän kukaan evoluutiota tarjoakaan elämän syntyä selittämään, no jotkut uskovat ehkä koska eivät tiedä mistä evoluutiossa on kyse . Elämän kehittymisen se kattaa nykyisen elämän synnystä lähtien, miten se voi olla "lyhyt" pätkä?


      • evlut101 kirjoitti:

        Totta ja siksi se ei kelpaakaan maailman tai edes elämän syntyteoriaksi. Se kattaa elämän synnyn ja kehittymisen kannalta vain lyhyen pätkän ja jättää loput täydelliseen pimentoon. Tosin koulujen biologienkirjoissa opetetaan edelleen että salama iski alkulampeen, jossa sitten syntyi itsestään elävä solu. Että en sitten tiedä että haluaako Suomen biologiyhteisö tehdä itsestään tässä naurunalaisen esittäessään täysin naurettavia ideoita, jota on verrattavissa johonkin neukkujen propagandaan.

        "...kattaa elämän synnyn ja kehittymisen kannalta vain lyhyen pätkän ja jättää loput täydelliseen pimentoon. "

        Ei, vaan evoluutio on se, mitä koko ajan tapahtuu. Se on hiukan toinen juttu, mikä pyrkii selittämään miten kaikki alkoi.


      • evlut101
        dikduk kirjoitti:

        "Totta ja siksi se ei kelpaakaan maailman tai edes elämän syntyteoriaksi. Se kattaa elämän synnyn ja kehittymisen kannalta vain lyhyen pätkän ja jättää loput täydelliseen pimentoon. "
        Eihän kukaan evoluutiota tarjoakaan elämän syntyä selittämään, no jotkut uskovat ehkä koska eivät tiedä mistä evoluutiossa on kyse . Elämän kehittymisen se kattaa nykyisen elämän synnystä lähtien, miten se voi olla "lyhyt" pätkä?

        Siten että ateistinen maailmansyntyteoria lähtee alkuräjähdyksestä (sattumalta), jonka seurauksena materia järjestyi jännän mystisesti juuri elämän kannalta sopivaan järjestykseen pallolle, jonka lähellä on kivasti aurinko, joka on välttämätön elämän synnylle ja kehitykselle. Myös oikeita atomeita oli oikeassa suhteessa (mm. H2O). Lisäksi fysiikan lait on sattumalta sellaiset, että ylipäätään materia voi rakentaa jotain eläviä rakenteita. Sitten se materia jotenkin sattumalta järjestyi alkusoluksi, jolla oli DNA ja myös DNA:n lukulaite unohtamatta muita soluelimiä. Se osasi jakautua eikä tuhoutunut jos esimerkiksi alkulampi olisi miljoonan vuoden aikana kerrankin kuivunut tai sinne olisi esimerkiksi osunut meteori tai tullut tulivuorenpurkaus. No kuitenkin sitten tämä sattumalta syntynyt solu osasi myös monistaa itseään (osaako esim. älypuhelimet tai autot saman, vaikka ne on huolellisesti suunniteltu eikä sattumalta syntyneet).

        Sen jälkeen vasta voidaan puhua siitä, että evoluutio olisi käynnistynyt. Sitä ennen tapahtui jotain muuta äärettömän epätodennäköistä.


      • evlut101 kirjoitti:

        Siten että ateistinen maailmansyntyteoria lähtee alkuräjähdyksestä (sattumalta), jonka seurauksena materia järjestyi jännän mystisesti juuri elämän kannalta sopivaan järjestykseen pallolle, jonka lähellä on kivasti aurinko, joka on välttämätön elämän synnylle ja kehitykselle. Myös oikeita atomeita oli oikeassa suhteessa (mm. H2O). Lisäksi fysiikan lait on sattumalta sellaiset, että ylipäätään materia voi rakentaa jotain eläviä rakenteita. Sitten se materia jotenkin sattumalta järjestyi alkusoluksi, jolla oli DNA ja myös DNA:n lukulaite unohtamatta muita soluelimiä. Se osasi jakautua eikä tuhoutunut jos esimerkiksi alkulampi olisi miljoonan vuoden aikana kerrankin kuivunut tai sinne olisi esimerkiksi osunut meteori tai tullut tulivuorenpurkaus. No kuitenkin sitten tämä sattumalta syntynyt solu osasi myös monistaa itseään (osaako esim. älypuhelimet tai autot saman, vaikka ne on huolellisesti suunniteltu eikä sattumalta syntyneet).

        Sen jälkeen vasta voidaan puhua siitä, että evoluutio olisi käynnistynyt. Sitä ennen tapahtui jotain muuta äärettömän epätodennäköistä.

        Niin, olipa syntytapa mikä hyvänsä, evoluutio käynnistyi sitten.


    • Niin tämähän on hauska kysymys koska evoluutiohan ei lopu, vaan on alati jyllääävä voima. Evoluutio tai vaikka ihminen kehittyy kokoajan. Tosin jos suoraan sanon niin ei välttämättä parempaan suuntaan, mutta kehittyy kuitenkin koska evoluutio tapahtuu. Kyse on tavallaan asioista joille ei mahda kukaan yhtään mitään, olkoon uskovainen tai ei. Silti, evoluutio tapahtuu...

    • Tämän_oppinut

      Kyllä se vaan on sillä tavalla, että näistä asioista keskusteleminen kreationistien kanssa on aika lailla turhaa. Olen huomannut vuosien varrella, etteivät kreationistit ole yleensä ollenkaan selvillä siitä mistä he puhuvat ja mitä he yrittävät kritisoida tai minkä kiistää.

      Nämä asiat ovat yleensä monimutkaisia, mutta kreationistit vetävät näistä tahallaan lapselliseksi tehtyjä yksinkertaistuksia, joihin he vielä sotkevat asiavirheitä ja sotkevat käsitteet kaikki keskenään. Helppoahan on moinen, että pilailaan koko asia ja sitten nauretaan päälle, mutta mitään tekemistä asiasta tietämisen ja siitä keskustelemisen kanssa tuollaisella pelleilyllä ei tietenkään ole.

      Nyt pitäisi olla tullut jo jokaiselle tätä asiaa seuranneelle selväksi, etteivät kreationistit yksinkertaisesti halua hyväksyä tiettyjä tosiasioita, joten he eivät suostu edes katsomaan niitä. Sen siis näkee kyllä tästä touhusta, että kreationistit taistelevat lähinnä olkinukkeja vastaan.

      Joskus oli hauskaa seurata tätä keskustelua ja osallistua siihen, kun sai nauraa hekottaa kreationistien aivopieruille. Se lysti kuitenkin laantui lopulta, koska samojen typeryyksien toistuvuus alkoi pitkälti vain puuduttaa. Kreationistit kun vain toistelevat niitä samoja jo käsiteltyjä juttuja, joissa heidän virheensä on heille osoitettu - ja hehän vain alottavat litanian uudelleen alusta kun ovat saaneet väistettyä vastaukset jokaiseen muka kiusalliseen kysymykseensä tai väitteeseensä.

      Tämä show ei siis ota loppuakseen niin kauan kun löytyy herkkäuskoisia ihmisiä, joille kreationistinen propaganda menee läpi. Periaatteessa minulle ei koskaan ole ollut ongelmaa siinä, että ihmiset uskovat mitä haluavat, silloinkaan kun he uskovat jotain minkä minä tiedän vääräksi.

      Ongelma minulle oli lähinnä kreationistien ylimielisyys, koska on niin säälittävää seurata vierestä kun joku, joka ei tiedä mitään asiasta, paasaa asiasta kuin mikäkin mestari. Nyttemmin kun ikää on itsellä karttunut niin tiedän senkin, ettei minun tietenkään tarvitse ottaa nokkiini siitä, että joku kreationisti mesoaa täällä keisarina ilman vaatteita.

      Paljon on muuten tullut evoluutiostakin opittua uutta, kun on saanut lukea miten asiasta oikeasti jotain tietävät ovat ojentaneet kreationisteja. Ja silläkään ei ole väliä, että kreationistit ainoan tuntemansa tavan mukaan todennäköisesti pyrkivät vääristelemään tämänkin kommentin sisältöä mahdollisissa vastauksissaan. Ei se oikeasti ketään hämää.

      Kaikki se aika minkä tässä jättää tähän soppaan lusikkaansa enää työntämättä, niin voi käyttää tieteen opiskeluun ilman kreationistista älämölöä. Kreationismilla kun ei ole tieteelle mitään annettavaa, niin tieteitä opiskelevalle ja tieteellisesti harrastuneelle kreationismilla ei myöskään lopulta ole kuin se huumoriarvo, ja kaikki vitsit väljähtyvät lopulta. Että hyvää loppuelämää vaan itse kullekin, niin kreationisteille kuin rationalisteillekin.

      • evolutionisti

        Hyvää elämää vain sinullekin. Tuohon kreationistien käsityksiin palstalla sen verran lisäisin, etten usko kaikkien kreationistien olevan aitoja kreationistejä, vaan uskon heidän olevan trolleja.


      • viimeinen.tuomio.2
        evolutionisti kirjoitti:

        Hyvää elämää vain sinullekin. Tuohon kreationistien käsityksiin palstalla sen verran lisäisin, etten usko kaikkien kreationistien olevan aitoja kreationistejä, vaan uskon heidän olevan trolleja.

        Suuri osa on omiin sukkiinsa kompuroivia trolleja ja suuri osa paholaisen asianajajia, joille se oma käsitys on selkeä, mutta debatin takia heittelevät täkyjä ja aina joku nappaa.


      • evlut101

        "Kyllä se vaan on sillä tavalla, että näistä asioista keskusteleminen kreationistien kanssa on aika lailla turhaa. "

        Maailmallahan käydään ateistit vs. uskovat väittelyä liittyen maailman syntyyn ja mm. evoluutioteoriaan ja nykytrendi on se, että evoluutiouskovaiset tapaavat hävitä ne väittelyt ja siksi he vetäytyvät niistä. Tiede on tehnyt selväksi, ettei evoluution mekanismi olekaan mikään yksinkertainen itsestään etenevä.

        "Nämä asiat ovat yleensä monimutkaisia, mutta kreationistit vetävät näistä tahallaan lapselliseksi tehtyjä yksinkertaistuksia, joihin he vielä sotkevat asiavirheitä"

        Minun kokemukseni on että yleensä evoluutiouskovainen ei osaa edes nimetä yhtä ainoaa todistetta evoluution puolesta.

        "Nyt pitäisi olla tullut jo jokaiselle tätä asiaa seuranneelle selväksi, etteivät kreationistit yksinkertaisesti halua hyväksyä tiettyjä tosiasioita"

        Aivan samoin on evoluutiouskovaisten laita. Turha heille on heittää järkiperusteita kun usko on niin lujaa.

        " Periaatteessa minulle ei koskaan ole ollut ongelmaa siinä, että ihmiset uskovat mitä haluavat, silloinkaan kun he uskovat jotain minkä minä tiedän vääräksi."

        Ja sinäkö olet varmasti oikeassa ja muut väärässä? Jopa on kovat luulot itsestäsi. Luuletko ettei kukaan ole vielä koskaan erehtynyt? Luuletko että olet itse erehtymätön? Jos et niin voisit hieman nöyremmin suhtautua sellaisiin tiedonjyväsiin, joissa luulet tietäväsi miten maailmankaikkeus on syntynyt. Et ole varmaankaan sitä laboratoriossa pystynyt kiistattomasti osoittamaan.

        "Ongelma minulle oli lähinnä kreationistien ylimielisyys, koska on niin säälittävää seurata vierestä kun joku, joka ei tiedä mitään asiasta, paasaa asiasta kuin mikäkin mestari. "

        Eiköhän sitä ylimielisyyttä ole saman verran molemmilla puolilla. Onhan noita evoluutiokriitikoita, jotka on koulutukseltaan tohtoreita ja päässeet yliopistoon professoreiksi.

        "Paljon on muuten tullut evoluutiostakin opittua uutta, kun on saanut lukea miten asiasta oikeasti jotain tietävät ovat ojentaneet kreationisteja. Ja silläkään ei ole väliä, että kreationistit ainoan tuntemansa tavan mukaan todennäköisesti pyrkivät vääristelemään tämänkin kommentin sisältöä mahdollisissa vastauksissaan. Ei se oikeasti ketään hämää."

        Paljon on ollut myös professoreita, jotka ovat ojentaneet evoluutiouskovia ja silti ei oppi mene kaaliin. Auktoriteeteillä tässä on turha miekkailla kun niitä löytyy molemmista oppisuunnista.

        "Kreationismilla kun ei ole tieteelle mitään annettavaa, "

        Niin paitsi että valtaosa maailman suurista tieteentekijöistä on ollut kreationistejä. Näistä mainitakseni Isac Newton, J. C . Maxwell, Carl Friedrich Gauss,... lisää näistä kreationisteista voit lukea täältä
        https://en.wikipedia.org/wiki/List_of_Christians_in_science_and_technology


      • evlut101 kirjoitti:

        "Kyllä se vaan on sillä tavalla, että näistä asioista keskusteleminen kreationistien kanssa on aika lailla turhaa. "

        Maailmallahan käydään ateistit vs. uskovat väittelyä liittyen maailman syntyyn ja mm. evoluutioteoriaan ja nykytrendi on se, että evoluutiouskovaiset tapaavat hävitä ne väittelyt ja siksi he vetäytyvät niistä. Tiede on tehnyt selväksi, ettei evoluution mekanismi olekaan mikään yksinkertainen itsestään etenevä.

        "Nämä asiat ovat yleensä monimutkaisia, mutta kreationistit vetävät näistä tahallaan lapselliseksi tehtyjä yksinkertaistuksia, joihin he vielä sotkevat asiavirheitä"

        Minun kokemukseni on että yleensä evoluutiouskovainen ei osaa edes nimetä yhtä ainoaa todistetta evoluution puolesta.

        "Nyt pitäisi olla tullut jo jokaiselle tätä asiaa seuranneelle selväksi, etteivät kreationistit yksinkertaisesti halua hyväksyä tiettyjä tosiasioita"

        Aivan samoin on evoluutiouskovaisten laita. Turha heille on heittää järkiperusteita kun usko on niin lujaa.

        " Periaatteessa minulle ei koskaan ole ollut ongelmaa siinä, että ihmiset uskovat mitä haluavat, silloinkaan kun he uskovat jotain minkä minä tiedän vääräksi."

        Ja sinäkö olet varmasti oikeassa ja muut väärässä? Jopa on kovat luulot itsestäsi. Luuletko ettei kukaan ole vielä koskaan erehtynyt? Luuletko että olet itse erehtymätön? Jos et niin voisit hieman nöyremmin suhtautua sellaisiin tiedonjyväsiin, joissa luulet tietäväsi miten maailmankaikkeus on syntynyt. Et ole varmaankaan sitä laboratoriossa pystynyt kiistattomasti osoittamaan.

        "Ongelma minulle oli lähinnä kreationistien ylimielisyys, koska on niin säälittävää seurata vierestä kun joku, joka ei tiedä mitään asiasta, paasaa asiasta kuin mikäkin mestari. "

        Eiköhän sitä ylimielisyyttä ole saman verran molemmilla puolilla. Onhan noita evoluutiokriitikoita, jotka on koulutukseltaan tohtoreita ja päässeet yliopistoon professoreiksi.

        "Paljon on muuten tullut evoluutiostakin opittua uutta, kun on saanut lukea miten asiasta oikeasti jotain tietävät ovat ojentaneet kreationisteja. Ja silläkään ei ole väliä, että kreationistit ainoan tuntemansa tavan mukaan todennäköisesti pyrkivät vääristelemään tämänkin kommentin sisältöä mahdollisissa vastauksissaan. Ei se oikeasti ketään hämää."

        Paljon on ollut myös professoreita, jotka ovat ojentaneet evoluutiouskovia ja silti ei oppi mene kaaliin. Auktoriteeteillä tässä on turha miekkailla kun niitä löytyy molemmista oppisuunnista.

        "Kreationismilla kun ei ole tieteelle mitään annettavaa, "

        Niin paitsi että valtaosa maailman suurista tieteentekijöistä on ollut kreationistejä. Näistä mainitakseni Isac Newton, J. C . Maxwell, Carl Friedrich Gauss,... lisää näistä kreationisteista voit lukea täältä
        https://en.wikipedia.org/wiki/List_of_Christians_in_science_and_technology

        "...nykytrendi on se, että evoluutiouskovaiset tapaavat hävitä ne väittelyt ja siksi he vetäytyvät niistä."

        Tuota, asia taitaa olla juuri päinvastoin. Eräs tieteilijä sanoi, ettei hän käy näitä enemmän uskontoon liittyivä debatteja, sillä hänellä ei oikein ole riittävää uskonnon tuntemusta.


      • Tämän_oppinut
        evlut101 kirjoitti:

        "Kyllä se vaan on sillä tavalla, että näistä asioista keskusteleminen kreationistien kanssa on aika lailla turhaa. "

        Maailmallahan käydään ateistit vs. uskovat väittelyä liittyen maailman syntyyn ja mm. evoluutioteoriaan ja nykytrendi on se, että evoluutiouskovaiset tapaavat hävitä ne väittelyt ja siksi he vetäytyvät niistä. Tiede on tehnyt selväksi, ettei evoluution mekanismi olekaan mikään yksinkertainen itsestään etenevä.

        "Nämä asiat ovat yleensä monimutkaisia, mutta kreationistit vetävät näistä tahallaan lapselliseksi tehtyjä yksinkertaistuksia, joihin he vielä sotkevat asiavirheitä"

        Minun kokemukseni on että yleensä evoluutiouskovainen ei osaa edes nimetä yhtä ainoaa todistetta evoluution puolesta.

        "Nyt pitäisi olla tullut jo jokaiselle tätä asiaa seuranneelle selväksi, etteivät kreationistit yksinkertaisesti halua hyväksyä tiettyjä tosiasioita"

        Aivan samoin on evoluutiouskovaisten laita. Turha heille on heittää järkiperusteita kun usko on niin lujaa.

        " Periaatteessa minulle ei koskaan ole ollut ongelmaa siinä, että ihmiset uskovat mitä haluavat, silloinkaan kun he uskovat jotain minkä minä tiedän vääräksi."

        Ja sinäkö olet varmasti oikeassa ja muut väärässä? Jopa on kovat luulot itsestäsi. Luuletko ettei kukaan ole vielä koskaan erehtynyt? Luuletko että olet itse erehtymätön? Jos et niin voisit hieman nöyremmin suhtautua sellaisiin tiedonjyväsiin, joissa luulet tietäväsi miten maailmankaikkeus on syntynyt. Et ole varmaankaan sitä laboratoriossa pystynyt kiistattomasti osoittamaan.

        "Ongelma minulle oli lähinnä kreationistien ylimielisyys, koska on niin säälittävää seurata vierestä kun joku, joka ei tiedä mitään asiasta, paasaa asiasta kuin mikäkin mestari. "

        Eiköhän sitä ylimielisyyttä ole saman verran molemmilla puolilla. Onhan noita evoluutiokriitikoita, jotka on koulutukseltaan tohtoreita ja päässeet yliopistoon professoreiksi.

        "Paljon on muuten tullut evoluutiostakin opittua uutta, kun on saanut lukea miten asiasta oikeasti jotain tietävät ovat ojentaneet kreationisteja. Ja silläkään ei ole väliä, että kreationistit ainoan tuntemansa tavan mukaan todennäköisesti pyrkivät vääristelemään tämänkin kommentin sisältöä mahdollisissa vastauksissaan. Ei se oikeasti ketään hämää."

        Paljon on ollut myös professoreita, jotka ovat ojentaneet evoluutiouskovia ja silti ei oppi mene kaaliin. Auktoriteeteillä tässä on turha miekkailla kun niitä löytyy molemmista oppisuunnista.

        "Kreationismilla kun ei ole tieteelle mitään annettavaa, "

        Niin paitsi että valtaosa maailman suurista tieteentekijöistä on ollut kreationistejä. Näistä mainitakseni Isac Newton, J. C . Maxwell, Carl Friedrich Gauss,... lisää näistä kreationisteista voit lukea täältä
        https://en.wikipedia.org/wiki/List_of_Christians_in_science_and_technology

        "Maailmallahan käydään ateistit vs. uskovat väittelyä liittyen maailman syntyyn ja mm. evoluutioteoriaan ja nykytrendi on se, että evoluutiouskovaiset tapaavat hävitä ne väittelyt ja siksi he vetäytyvät niistä."

        Eivät kaikki suinkaan vetäydy, mutta hyvä syy vetäytyä on se, että kreationismi perustuu argumentointivirheisiin ja huijauksiin, eikä sitä myllyä pyörittävät kykene aikuisten väliseen keskusteluun.

        "Tiede on tehnyt selväksi, ettei evoluution mekanismi olekaan mikään yksinkertainen itsestään etenevä."

        Valehtelet:

        en.wikipedia.org/wiki/Evidence_of_common_descent
        en.wikipedia.org/wiki/Level_of_support_for_evolution
        en.wikipedia.org/wiki/List_of_scientific_bodies_explicitly_rejecting_intelligent_design

        "Minun kokemukseni on että yleensä evoluutiouskovainen ei osaa edes nimetä yhtä ainoaa todistetta evoluution puolesta."

        Evoluutiouskovaisuudesta puhuminen on kreationistien projisointia eikä mitään muuta. Tiedätte, että itse edustatte uskontoa, niin yritätte siten vähätellä vastapuolta nimittämällä senkin edustajia uskonnoksi; näin vahingossa myönnätte, että tiedätte omaa kantaanne pidettävän vähempiarvoisena.

        Ja on tosiaan nähty, että kun sinulle tarjotaan kaikki todisteet, niin huomautat, että pyysit vain yhtä: ikään kuin et voisi aikuinen (?) ihminen valita niistä kaikista tai mielummin vain katsoa ne kaikki läpi. Yhteensopivien osien kokonaisuudestahan tässä kuitenkin on kyse, ja silloin tuollainen, että vastaus on muka väärä kun lueteltiin enemmän kuin yksi todiste, on suorastaan lapsellista vinoilua.

        "Aivan samoin on evoluutiouskovaisten laita. Turha heille on heittää järkiperusteita kun usko on niin lujaa."

        Myönsit tuossa vahingossa, että kreationistit eivät halua kuulla tosiasioita, mutta peittelit sen halvalla projisointiyrityksellä. Toistat jo tuossa vaiheessa samaa argumentointivirhettä, eli projisointia uskosta.

        "Ja sinäkö olet varmasti oikeassa ja muut väärässä?"

        En ole sanonut tuommoista, joten asiaan liittymättömät vouhotuksesi laboratoriokokeista yms. tuota asiaa muka koskien voikin ohittaa. Kysehän oli siitä, että tiedän tietäväni jotain, mitä jotkut muut eivät tiedä, siinä kaikki.

        Teet itsestäsi naurunalaisen kun alat laittaa sanoja suuhuni. Mutta eivätpä kreationistit yleensä muuhun pysty. Joka kerta melkein sama juttu, että vääristellään sitä mistä oli puhe.

        "Eiköhän sitä ylimielisyyttä ole saman verran molemmilla puolilla. Onhan noita evoluutiokriitikoita, jotka on koulutukseltaan tohtoreita ja päässeet yliopistoon professoreiksi."

        Kas kun ne evoluutiokriitikot eivät yleensä ole biologeja ja jos ovatkin, niin eivät pysty osoittamaan, että "kritisoisivat" evoluutiota tieteellsin perustein. Se tiedetään, että evoluutiota vastustetaan uskonnollisin perustein ja kiista onkin lähinnä nyt siitä, että kreationistit teeskentelevät uskontonsa olevan tiedettä, mitä se ei tietenkään ole.

        "Paljon on ollut myös professoreita, jotka ovat ojentaneet evoluutiouskovia ja silti ei oppi mene kaaliin. Auktoriteeteillä tässä on turha miekkailla kun niitä löytyy molemmista oppisuunnista."

        Äläpäs innostu "auktoriteetteinesi." Evoluution kiistäminen uskonnollisista syistä, mikä ihan oikeudenkäyntejä myöten osoitetusti on se mitä kreationistiorganisaatiot tekevät, ja minkä myötä kreationismiin höpsähtäneet kreationismia paasaavat, käytännössä nimittäin vetää maton alta tuolta auktoriteettirehvastelultasi. Evoluutiokriittisyys ei perustu tieteeseen, vaan se perustuu uskontoon, ja silloin on turhaa koettaa ratsastaa "evoluutiokriittisten" tieteilijöiden auktoriteetilla.

        "Niin paitsi että valtaosa maailman suurista tieteentekijöistä on ollut kreationistejä. Näistä mainitakseni Isac Newton, J. C . Maxwell, Carl Friedrich Gauss."

        Eivät he antaneet tieteelle kreationismia, vaan tieteellisen työpanoksensa edustamillaan tieteenaloilla. Edelleen sekoitat ihmisten uskonnot ja heidän asiantuntijatyönsä jollakin muulla alueella.

        Mutta tuokin on eräs kreationismille tyypillinen argumentointivirhe, eli asioiden sekoittaminen keskenään, eli kun kreationistit tekee jotain tiedettä, tekee hän muka kreationistista tiedettä, vaikka oikeasti jos tehdään tiedettä, niin silloin ne uskonnot jätetään ulkovaatteiden mukana labran narikkaan, niin sanotusti. Tieteen kannalta ei ole väliä mihin tieteilijä uskoo tai ei usko, kunhan hoitaa tieteellisen työnsä asiallisesti.

        Ja kun minä sanoin, ettei kreationismilla ole mitään annettavaa tieteelle, tarkotin juuri sitä, että kreationismi on uskonnollisen apologian motivoimaa pseudotiedettä, mikä ei paini tieteen kanssa edes samassa sarjassa. Silloin minä en ole sanonut niin, etteivätkö luomisuskoiset voisi tehdä tiedettä.

        Ja kun minä nyt totean, ettei kaltaisesi vääristelijän kanssa tosiaan viitsi jatkaa tätä keskustelua, niin sinä varmaan väität, että luikin karkuun jotenkin. Katso kuitenkin sitä ennen peiliin ja mieti mitä olit itse minulle vastannut. Mieti sitten, että miksi kukaan viitsisi käyttää tämän enempää aikaansa tuommoiseen pelehtimiseen.


      • Tämän_oppinut kirjoitti:

        "Maailmallahan käydään ateistit vs. uskovat väittelyä liittyen maailman syntyyn ja mm. evoluutioteoriaan ja nykytrendi on se, että evoluutiouskovaiset tapaavat hävitä ne väittelyt ja siksi he vetäytyvät niistä."

        Eivät kaikki suinkaan vetäydy, mutta hyvä syy vetäytyä on se, että kreationismi perustuu argumentointivirheisiin ja huijauksiin, eikä sitä myllyä pyörittävät kykene aikuisten väliseen keskusteluun.

        "Tiede on tehnyt selväksi, ettei evoluution mekanismi olekaan mikään yksinkertainen itsestään etenevä."

        Valehtelet:

        en.wikipedia.org/wiki/Evidence_of_common_descent
        en.wikipedia.org/wiki/Level_of_support_for_evolution
        en.wikipedia.org/wiki/List_of_scientific_bodies_explicitly_rejecting_intelligent_design

        "Minun kokemukseni on että yleensä evoluutiouskovainen ei osaa edes nimetä yhtä ainoaa todistetta evoluution puolesta."

        Evoluutiouskovaisuudesta puhuminen on kreationistien projisointia eikä mitään muuta. Tiedätte, että itse edustatte uskontoa, niin yritätte siten vähätellä vastapuolta nimittämällä senkin edustajia uskonnoksi; näin vahingossa myönnätte, että tiedätte omaa kantaanne pidettävän vähempiarvoisena.

        Ja on tosiaan nähty, että kun sinulle tarjotaan kaikki todisteet, niin huomautat, että pyysit vain yhtä: ikään kuin et voisi aikuinen (?) ihminen valita niistä kaikista tai mielummin vain katsoa ne kaikki läpi. Yhteensopivien osien kokonaisuudestahan tässä kuitenkin on kyse, ja silloin tuollainen, että vastaus on muka väärä kun lueteltiin enemmän kuin yksi todiste, on suorastaan lapsellista vinoilua.

        "Aivan samoin on evoluutiouskovaisten laita. Turha heille on heittää järkiperusteita kun usko on niin lujaa."

        Myönsit tuossa vahingossa, että kreationistit eivät halua kuulla tosiasioita, mutta peittelit sen halvalla projisointiyrityksellä. Toistat jo tuossa vaiheessa samaa argumentointivirhettä, eli projisointia uskosta.

        "Ja sinäkö olet varmasti oikeassa ja muut väärässä?"

        En ole sanonut tuommoista, joten asiaan liittymättömät vouhotuksesi laboratoriokokeista yms. tuota asiaa muka koskien voikin ohittaa. Kysehän oli siitä, että tiedän tietäväni jotain, mitä jotkut muut eivät tiedä, siinä kaikki.

        Teet itsestäsi naurunalaisen kun alat laittaa sanoja suuhuni. Mutta eivätpä kreationistit yleensä muuhun pysty. Joka kerta melkein sama juttu, että vääristellään sitä mistä oli puhe.

        "Eiköhän sitä ylimielisyyttä ole saman verran molemmilla puolilla. Onhan noita evoluutiokriitikoita, jotka on koulutukseltaan tohtoreita ja päässeet yliopistoon professoreiksi."

        Kas kun ne evoluutiokriitikot eivät yleensä ole biologeja ja jos ovatkin, niin eivät pysty osoittamaan, että "kritisoisivat" evoluutiota tieteellsin perustein. Se tiedetään, että evoluutiota vastustetaan uskonnollisin perustein ja kiista onkin lähinnä nyt siitä, että kreationistit teeskentelevät uskontonsa olevan tiedettä, mitä se ei tietenkään ole.

        "Paljon on ollut myös professoreita, jotka ovat ojentaneet evoluutiouskovia ja silti ei oppi mene kaaliin. Auktoriteeteillä tässä on turha miekkailla kun niitä löytyy molemmista oppisuunnista."

        Äläpäs innostu "auktoriteetteinesi." Evoluution kiistäminen uskonnollisista syistä, mikä ihan oikeudenkäyntejä myöten osoitetusti on se mitä kreationistiorganisaatiot tekevät, ja minkä myötä kreationismiin höpsähtäneet kreationismia paasaavat, käytännössä nimittäin vetää maton alta tuolta auktoriteettirehvastelultasi. Evoluutiokriittisyys ei perustu tieteeseen, vaan se perustuu uskontoon, ja silloin on turhaa koettaa ratsastaa "evoluutiokriittisten" tieteilijöiden auktoriteetilla.

        "Niin paitsi että valtaosa maailman suurista tieteentekijöistä on ollut kreationistejä. Näistä mainitakseni Isac Newton, J. C . Maxwell, Carl Friedrich Gauss."

        Eivät he antaneet tieteelle kreationismia, vaan tieteellisen työpanoksensa edustamillaan tieteenaloilla. Edelleen sekoitat ihmisten uskonnot ja heidän asiantuntijatyönsä jollakin muulla alueella.

        Mutta tuokin on eräs kreationismille tyypillinen argumentointivirhe, eli asioiden sekoittaminen keskenään, eli kun kreationistit tekee jotain tiedettä, tekee hän muka kreationistista tiedettä, vaikka oikeasti jos tehdään tiedettä, niin silloin ne uskonnot jätetään ulkovaatteiden mukana labran narikkaan, niin sanotusti. Tieteen kannalta ei ole väliä mihin tieteilijä uskoo tai ei usko, kunhan hoitaa tieteellisen työnsä asiallisesti.

        Ja kun minä sanoin, ettei kreationismilla ole mitään annettavaa tieteelle, tarkotin juuri sitä, että kreationismi on uskonnollisen apologian motivoimaa pseudotiedettä, mikä ei paini tieteen kanssa edes samassa sarjassa. Silloin minä en ole sanonut niin, etteivätkö luomisuskoiset voisi tehdä tiedettä.

        Ja kun minä nyt totean, ettei kaltaisesi vääristelijän kanssa tosiaan viitsi jatkaa tätä keskustelua, niin sinä varmaan väität, että luikin karkuun jotenkin. Katso kuitenkin sitä ennen peiliin ja mieti mitä olit itse minulle vastannut. Mieti sitten, että miksi kukaan viitsisi käyttää tämän enempää aikaansa tuommoiseen pelehtimiseen.

        ”…evoluutiota vastustetaan uskonnollisin perustein ja kiista onkin lähinnä nyt siitä, että kreationistit teeskentelevät uskontonsa olevan tiedettä…”

        Heillä on sellainen paradoksi taustalla. Kun Raamattu ei yksi riitä, niin sitä varten on kehitelty tieteellisen kuuloisia selityksiä, jotta Raamatusta saataisiin enemmän ”tieteellinen” ja näin nykyihmiselle uskottavampi. He hylkäävät perinteisen Raamatun tutkimisen ja asettavat nämä omat teoreettisen kuuloisen selityksen niiden sijaan.


      • Tämän_oppinut
        mummomuori kirjoitti:

        ”…evoluutiota vastustetaan uskonnollisin perustein ja kiista onkin lähinnä nyt siitä, että kreationistit teeskentelevät uskontonsa olevan tiedettä…”

        Heillä on sellainen paradoksi taustalla. Kun Raamattu ei yksi riitä, niin sitä varten on kehitelty tieteellisen kuuloisia selityksiä, jotta Raamatusta saataisiin enemmän ”tieteellinen” ja näin nykyihmiselle uskottavampi. He hylkäävät perinteisen Raamatun tutkimisen ja asettavat nämä omat teoreettisen kuuloisen selityksen niiden sijaan.

        Ongelma on juuri siinä, että jos Raamattu ei riitä uskoon ja pitää keksiä muuta ad hoc, niin voidaan katsoa, että sitä uskoa ei oikein ole edes, vaan on pelkkää uskottelua. Kreationismi ei ole aitoa uskoa, vaan se on uskottelua. Jokainen uskova voisi kohdallaan miettiä, että lähteekö se oma usko sisimmästä vai ei; onko usko sitä, että joku väittää, että tuo väitetty kudosklimppi tuossa todistaa tätä ja tuo kivimuodostelma tuossa tuota, että kyllä oli luominen juuri 6000 vuotta sitten ja vedenpaisumus sen jälkeen...

        Valitettavasti ihmiset eivät tunnu ymmärtävän edes sitä, että mitä he itse uskovat ja mitä he uskottelevat itselleen ja toisilleen. Jotkut sitten uskottelevat, että humpuukiorganisaatiot kertovat totisinta totta, mikä ah niin mukavasti sopii siihen mitä uskotaan, eikä edes ymmärretä, että se mitä nuo organisaatiot väittävät, muokkaavat sitä mihin uskotaan eivätkä vain pönkitä sitä ennestään olevaa. Se on pässinä narussa olemista, olla rivikreationisti.

        Aito usko sinänsä on varmasti jotain omaansa, mutta kreationismi aika lailla vie pohjaa siltä. Kreationismihan on uskonnosta tehty ismi, aivan samoin kuin vaikka skientismi on tieteestä tehtyä uskontoa, kaiken överitieteellistämistä. Ongelma on se, etteivät kreationistit ymmärrä, ettei heidän todellinen vastapuoli ole esim. evoluution tai vanhan Maan hyväksminen tosiasioiksi, mutta niitä he vain vastustavat.


      • Tämän_oppinut kirjoitti:

        Ongelma on juuri siinä, että jos Raamattu ei riitä uskoon ja pitää keksiä muuta ad hoc, niin voidaan katsoa, että sitä uskoa ei oikein ole edes, vaan on pelkkää uskottelua. Kreationismi ei ole aitoa uskoa, vaan se on uskottelua. Jokainen uskova voisi kohdallaan miettiä, että lähteekö se oma usko sisimmästä vai ei; onko usko sitä, että joku väittää, että tuo väitetty kudosklimppi tuossa todistaa tätä ja tuo kivimuodostelma tuossa tuota, että kyllä oli luominen juuri 6000 vuotta sitten ja vedenpaisumus sen jälkeen...

        Valitettavasti ihmiset eivät tunnu ymmärtävän edes sitä, että mitä he itse uskovat ja mitä he uskottelevat itselleen ja toisilleen. Jotkut sitten uskottelevat, että humpuukiorganisaatiot kertovat totisinta totta, mikä ah niin mukavasti sopii siihen mitä uskotaan, eikä edes ymmärretä, että se mitä nuo organisaatiot väittävät, muokkaavat sitä mihin uskotaan eivätkä vain pönkitä sitä ennestään olevaa. Se on pässinä narussa olemista, olla rivikreationisti.

        Aito usko sinänsä on varmasti jotain omaansa, mutta kreationismi aika lailla vie pohjaa siltä. Kreationismihan on uskonnosta tehty ismi, aivan samoin kuin vaikka skientismi on tieteestä tehtyä uskontoa, kaiken överitieteellistämistä. Ongelma on se, etteivät kreationistit ymmärrä, ettei heidän todellinen vastapuoli ole esim. evoluution tai vanhan Maan hyväksminen tosiasioiksi, mutta niitä he vain vastustavat.

        "...humpuukiorganisaatiot kertovat totisinta totta, mikä ah niin mukavasti sopii siihen mitä uskotaan, eikä edes ymmärretä, että se mitä nuo organisaatiot väittävät, muokkaavat sitä mihin uskotaan eivätkä vain pönkitä sitä ennestään olevaa."

        Niissä yhteisössä, joissa nojataan vahvasti autoritääriseen vallankäyttöön, toistuvasti ongelmaksi muodostuu se, että nämä auktoriteetit jossain vaiheessa alkavat "pelata omaan pussiin". Se, joku kyseenalaistaa nämä ihmiset, manipuloidaan uskomaan, että he vastustavat oikeastaan Jumalaa kyseenalaistamalla johtajien mielipiteet ja ohjeet.


      • evlut101
        Tämän_oppinut kirjoitti:

        "Maailmallahan käydään ateistit vs. uskovat väittelyä liittyen maailman syntyyn ja mm. evoluutioteoriaan ja nykytrendi on se, että evoluutiouskovaiset tapaavat hävitä ne väittelyt ja siksi he vetäytyvät niistä."

        Eivät kaikki suinkaan vetäydy, mutta hyvä syy vetäytyä on se, että kreationismi perustuu argumentointivirheisiin ja huijauksiin, eikä sitä myllyä pyörittävät kykene aikuisten väliseen keskusteluun.

        "Tiede on tehnyt selväksi, ettei evoluution mekanismi olekaan mikään yksinkertainen itsestään etenevä."

        Valehtelet:

        en.wikipedia.org/wiki/Evidence_of_common_descent
        en.wikipedia.org/wiki/Level_of_support_for_evolution
        en.wikipedia.org/wiki/List_of_scientific_bodies_explicitly_rejecting_intelligent_design

        "Minun kokemukseni on että yleensä evoluutiouskovainen ei osaa edes nimetä yhtä ainoaa todistetta evoluution puolesta."

        Evoluutiouskovaisuudesta puhuminen on kreationistien projisointia eikä mitään muuta. Tiedätte, että itse edustatte uskontoa, niin yritätte siten vähätellä vastapuolta nimittämällä senkin edustajia uskonnoksi; näin vahingossa myönnätte, että tiedätte omaa kantaanne pidettävän vähempiarvoisena.

        Ja on tosiaan nähty, että kun sinulle tarjotaan kaikki todisteet, niin huomautat, että pyysit vain yhtä: ikään kuin et voisi aikuinen (?) ihminen valita niistä kaikista tai mielummin vain katsoa ne kaikki läpi. Yhteensopivien osien kokonaisuudestahan tässä kuitenkin on kyse, ja silloin tuollainen, että vastaus on muka väärä kun lueteltiin enemmän kuin yksi todiste, on suorastaan lapsellista vinoilua.

        "Aivan samoin on evoluutiouskovaisten laita. Turha heille on heittää järkiperusteita kun usko on niin lujaa."

        Myönsit tuossa vahingossa, että kreationistit eivät halua kuulla tosiasioita, mutta peittelit sen halvalla projisointiyrityksellä. Toistat jo tuossa vaiheessa samaa argumentointivirhettä, eli projisointia uskosta.

        "Ja sinäkö olet varmasti oikeassa ja muut väärässä?"

        En ole sanonut tuommoista, joten asiaan liittymättömät vouhotuksesi laboratoriokokeista yms. tuota asiaa muka koskien voikin ohittaa. Kysehän oli siitä, että tiedän tietäväni jotain, mitä jotkut muut eivät tiedä, siinä kaikki.

        Teet itsestäsi naurunalaisen kun alat laittaa sanoja suuhuni. Mutta eivätpä kreationistit yleensä muuhun pysty. Joka kerta melkein sama juttu, että vääristellään sitä mistä oli puhe.

        "Eiköhän sitä ylimielisyyttä ole saman verran molemmilla puolilla. Onhan noita evoluutiokriitikoita, jotka on koulutukseltaan tohtoreita ja päässeet yliopistoon professoreiksi."

        Kas kun ne evoluutiokriitikot eivät yleensä ole biologeja ja jos ovatkin, niin eivät pysty osoittamaan, että "kritisoisivat" evoluutiota tieteellsin perustein. Se tiedetään, että evoluutiota vastustetaan uskonnollisin perustein ja kiista onkin lähinnä nyt siitä, että kreationistit teeskentelevät uskontonsa olevan tiedettä, mitä se ei tietenkään ole.

        "Paljon on ollut myös professoreita, jotka ovat ojentaneet evoluutiouskovia ja silti ei oppi mene kaaliin. Auktoriteeteillä tässä on turha miekkailla kun niitä löytyy molemmista oppisuunnista."

        Äläpäs innostu "auktoriteetteinesi." Evoluution kiistäminen uskonnollisista syistä, mikä ihan oikeudenkäyntejä myöten osoitetusti on se mitä kreationistiorganisaatiot tekevät, ja minkä myötä kreationismiin höpsähtäneet kreationismia paasaavat, käytännössä nimittäin vetää maton alta tuolta auktoriteettirehvastelultasi. Evoluutiokriittisyys ei perustu tieteeseen, vaan se perustuu uskontoon, ja silloin on turhaa koettaa ratsastaa "evoluutiokriittisten" tieteilijöiden auktoriteetilla.

        "Niin paitsi että valtaosa maailman suurista tieteentekijöistä on ollut kreationistejä. Näistä mainitakseni Isac Newton, J. C . Maxwell, Carl Friedrich Gauss."

        Eivät he antaneet tieteelle kreationismia, vaan tieteellisen työpanoksensa edustamillaan tieteenaloilla. Edelleen sekoitat ihmisten uskonnot ja heidän asiantuntijatyönsä jollakin muulla alueella.

        Mutta tuokin on eräs kreationismille tyypillinen argumentointivirhe, eli asioiden sekoittaminen keskenään, eli kun kreationistit tekee jotain tiedettä, tekee hän muka kreationistista tiedettä, vaikka oikeasti jos tehdään tiedettä, niin silloin ne uskonnot jätetään ulkovaatteiden mukana labran narikkaan, niin sanotusti. Tieteen kannalta ei ole väliä mihin tieteilijä uskoo tai ei usko, kunhan hoitaa tieteellisen työnsä asiallisesti.

        Ja kun minä sanoin, ettei kreationismilla ole mitään annettavaa tieteelle, tarkotin juuri sitä, että kreationismi on uskonnollisen apologian motivoimaa pseudotiedettä, mikä ei paini tieteen kanssa edes samassa sarjassa. Silloin minä en ole sanonut niin, etteivätkö luomisuskoiset voisi tehdä tiedettä.

        Ja kun minä nyt totean, ettei kaltaisesi vääristelijän kanssa tosiaan viitsi jatkaa tätä keskustelua, niin sinä varmaan väität, että luikin karkuun jotenkin. Katso kuitenkin sitä ennen peiliin ja mieti mitä olit itse minulle vastannut. Mieti sitten, että miksi kukaan viitsisi käyttää tämän enempää aikaansa tuommoiseen pelehtimiseen.

        "Eivät kaikki suinkaan vetäydy, mutta hyvä syy vetäytyä on se, että kreationismi perustuu argumentointivirheisiin ja huijauksiin, eikä sitä myllyä pyörittävät kykene aikuisten väliseen keskusteluun."

        Ei kyse ole pelkästään kreationismista vaan tieteellisestä evoluutioteorian kritisoinnista. Sitä ei saisi tehdä, koska loukkaat pyhää lehmää ja olet tällöin kuten tästäkin kirjoituksesta nähdään epä-aikuinen huijari. Voisitko nyt kuitenkin sitten selittää, että mille argumentaatiovirheelle kukin evoluutiota koskeva kritiikki perustuu? Mainitse edes yksi esimerkki.
        "
        Valehtelet:

        en.wikipedia.org/wiki/Evidence_of_common_descent
        en.wikipedia.org/wiki/Level_of_support_for_evolution
        en.wikipedia.org/wiki/List_of_scientific_bodies_explicitly_rejecting_intelligent_design"

        Klassinen argumentaatiovirhe. Linkitetään läjäkaupalla tekstiä eikä sanota oikeasti mitään. Kerro ihan omin sanoin että mitä halusit tässä kertoa? Samanlainen argumentaatiovirhe olisi perustella väitteensä Raamatulla ja antaa sitten Raamattu kouraan ja sanoa että väitteen todiste löytyy sieltä. Että luo koko Raamattu läpi, niin sieltä se löytyy. Tieteelliseen metodiin kuuluu ajatus siitä, että jos väität jotain, niin esität väitettä koskevat todisteet yksityiskohtaisesti. Ei riitä että linkittelee jotain epämääräisiä sivustoja. Samalla tavalla toimii myös oikeusistuin, joka vetoaa aina tiettyyn lainkohtaan antaessaan tuomion.

        "Evoluutiouskovaisuudesta puhuminen on kreationistien projisointia eikä mitään muuta. Tiedätte, että itse edustatte uskontoa, niin yritätte siten vähätellä vastapuolta nimittämällä senkin edustajia uskonnoksi; näin vahingossa myönnätte, että tiedätte omaa kantaanne pidettävän vähempiarvoisena."

        Luokittelen evoluutioteorian nykyään uskonnoksi lähinnä tieteellisistä syistä. Koska se ei täytä tieteen tunnusmerkkejä ja sisältää läjäpäin dataa, joka kumoaa sen. Esimerkiksi fossiiliaineiston punktualismi todistaa, että lajit eivät muutu ajan saatossa, mikä puolestaan on jo riittävä todiste koko teorian kumoamiselle.

        Toisekseen esimerkikisi tämä mies on sitä mieltä että evoluutioteoria on valhetta. Ja kas kummaa hän on ateisti ja professori
        https://fi.wikipedia.org/wiki/Thomas_Nagel

        Onko hänkin siis kreationisti, kun ei kerran usko evoluutioon?

        "
        Ja on tosiaan nähty, että kun sinulle tarjotaan kaikki todisteet, niin huomautat, että pyysit vain yhtä: ikään kuin et voisi aikuinen (?) ihminen valita niistä kaikista tai mielummin vain katsoa ne kaikki läpi. Yhteensopivien osien kokonaisuudestahan tässä kuitenkin on kyse, ja silloin tuollainen, että vastaus on muka väärä kun lueteltiin enemmän kuin yksi todiste, on suorastaan lapsellista vinoilua."

        Ei vaan juuri se että heitetään tuhatsivuisia tekstipätkiä ja sanotaan että täällä se todistetaan. Se on todistus siitä, ettei yhtäkään kunnon todistetta löydy.

        "Myönsit tuossa vahingossa, että kreationistit eivät halua kuulla tosiasioita, mutta peittelit sen halvalla projisointiyrityksellä. Toistat jo tuossa vaiheessa samaa argumentointivirhettä, eli projisointia uskosta."

        Kyse ei ole mistään projisoinnista vaan rinnastan kaikki sellaiset teoriat/hypoteesit uskonnoiksi, joille ei löydy uskottavia tieteellisiä todisteita, mutta joihin silti uskotaan horjumatta. Jo se että evoluutioteoriaa ei saisi kritisoida antaa sille uskonnollisen leiman. Nimittäin tieteellisille teorioille on ominaista, että niitä saa ja kuuluu koetella. Jos koettelu johtaa heti hallitsemattomaan tunnereaktioon, niin kyse on uskonnosta.

        "En ole sanonut tuommoista, joten asiaan liittymättömät vouhotuksesi laboratoriokokeista yms. tuota asiaa muka koskien voikin ohittaa. Kysehän oli siitä, että tiedän tietäväni jotain, mitä jotkut muut eivät tiedä, siinä kaikki. "

        Turha tässä on alkaa raivoamaan. Ja millä perusteella tieteelliset kriteerit voi muka ohittaa ja silti väittää, että kyseinen teoria on täyttä faktaa?

        Maailma on muuten täynnä ihmisiä, jotka luulee tietävänsä. Ettet vaan olisi yksi heistä.

        "
        Teet itsestäsi naurunalaisen kun alat laittaa sanoja suuhuni. Mutta eivätpä kreationistit yleensä muuhun pysty. Joka kerta melkein sama juttu, että vääristellään sitä mistä oli puhe.
        "

        Argumentum ad hominem sanoisin puhtaimmassa muodossaan. :D
        Minä kun olen "kreationisti" ja siksi en pysty muuhun kun sanojen suuhun laittamiseen.

        "Äläpäs innostu "auktoriteetteinesi." Evoluution kiistäminen uskonnollisista syistä, mikä ihan oikeudenkäyntejä myöten osoitetusti on se mitä kreationistiorganisaatiot tekevät, ja minkä myötä kreationismiin höpsähtäneet kreationismia paasaavat, käytännössä nimittäin vetää maton alta tuolta auktoriteettirehvastelultasi. Evoluutiokriittisyys ei perustu tieteeseen, vaan se perustuu uskontoon, ja silloin on turhaa koettaa ratsastaa "evoluutiokriittisten" tieteilijöiden auktoriteetilla."

        Ja mistä lähtien oikeudenkäynnissä päätetään mikä on tieteellinen totuus ja mikä ei? Käsittääkseni yliopistot on sitä varten. Ja väitätkö vakavissasi että jenkkituomioistuin on sinusta erehtymätön.


      • evlut101
        Tämän_oppinut kirjoitti:

        "Maailmallahan käydään ateistit vs. uskovat väittelyä liittyen maailman syntyyn ja mm. evoluutioteoriaan ja nykytrendi on se, että evoluutiouskovaiset tapaavat hävitä ne väittelyt ja siksi he vetäytyvät niistä."

        Eivät kaikki suinkaan vetäydy, mutta hyvä syy vetäytyä on se, että kreationismi perustuu argumentointivirheisiin ja huijauksiin, eikä sitä myllyä pyörittävät kykene aikuisten väliseen keskusteluun.

        "Tiede on tehnyt selväksi, ettei evoluution mekanismi olekaan mikään yksinkertainen itsestään etenevä."

        Valehtelet:

        en.wikipedia.org/wiki/Evidence_of_common_descent
        en.wikipedia.org/wiki/Level_of_support_for_evolution
        en.wikipedia.org/wiki/List_of_scientific_bodies_explicitly_rejecting_intelligent_design

        "Minun kokemukseni on että yleensä evoluutiouskovainen ei osaa edes nimetä yhtä ainoaa todistetta evoluution puolesta."

        Evoluutiouskovaisuudesta puhuminen on kreationistien projisointia eikä mitään muuta. Tiedätte, että itse edustatte uskontoa, niin yritätte siten vähätellä vastapuolta nimittämällä senkin edustajia uskonnoksi; näin vahingossa myönnätte, että tiedätte omaa kantaanne pidettävän vähempiarvoisena.

        Ja on tosiaan nähty, että kun sinulle tarjotaan kaikki todisteet, niin huomautat, että pyysit vain yhtä: ikään kuin et voisi aikuinen (?) ihminen valita niistä kaikista tai mielummin vain katsoa ne kaikki läpi. Yhteensopivien osien kokonaisuudestahan tässä kuitenkin on kyse, ja silloin tuollainen, että vastaus on muka väärä kun lueteltiin enemmän kuin yksi todiste, on suorastaan lapsellista vinoilua.

        "Aivan samoin on evoluutiouskovaisten laita. Turha heille on heittää järkiperusteita kun usko on niin lujaa."

        Myönsit tuossa vahingossa, että kreationistit eivät halua kuulla tosiasioita, mutta peittelit sen halvalla projisointiyrityksellä. Toistat jo tuossa vaiheessa samaa argumentointivirhettä, eli projisointia uskosta.

        "Ja sinäkö olet varmasti oikeassa ja muut väärässä?"

        En ole sanonut tuommoista, joten asiaan liittymättömät vouhotuksesi laboratoriokokeista yms. tuota asiaa muka koskien voikin ohittaa. Kysehän oli siitä, että tiedän tietäväni jotain, mitä jotkut muut eivät tiedä, siinä kaikki.

        Teet itsestäsi naurunalaisen kun alat laittaa sanoja suuhuni. Mutta eivätpä kreationistit yleensä muuhun pysty. Joka kerta melkein sama juttu, että vääristellään sitä mistä oli puhe.

        "Eiköhän sitä ylimielisyyttä ole saman verran molemmilla puolilla. Onhan noita evoluutiokriitikoita, jotka on koulutukseltaan tohtoreita ja päässeet yliopistoon professoreiksi."

        Kas kun ne evoluutiokriitikot eivät yleensä ole biologeja ja jos ovatkin, niin eivät pysty osoittamaan, että "kritisoisivat" evoluutiota tieteellsin perustein. Se tiedetään, että evoluutiota vastustetaan uskonnollisin perustein ja kiista onkin lähinnä nyt siitä, että kreationistit teeskentelevät uskontonsa olevan tiedettä, mitä se ei tietenkään ole.

        "Paljon on ollut myös professoreita, jotka ovat ojentaneet evoluutiouskovia ja silti ei oppi mene kaaliin. Auktoriteeteillä tässä on turha miekkailla kun niitä löytyy molemmista oppisuunnista."

        Äläpäs innostu "auktoriteetteinesi." Evoluution kiistäminen uskonnollisista syistä, mikä ihan oikeudenkäyntejä myöten osoitetusti on se mitä kreationistiorganisaatiot tekevät, ja minkä myötä kreationismiin höpsähtäneet kreationismia paasaavat, käytännössä nimittäin vetää maton alta tuolta auktoriteettirehvastelultasi. Evoluutiokriittisyys ei perustu tieteeseen, vaan se perustuu uskontoon, ja silloin on turhaa koettaa ratsastaa "evoluutiokriittisten" tieteilijöiden auktoriteetilla.

        "Niin paitsi että valtaosa maailman suurista tieteentekijöistä on ollut kreationistejä. Näistä mainitakseni Isac Newton, J. C . Maxwell, Carl Friedrich Gauss."

        Eivät he antaneet tieteelle kreationismia, vaan tieteellisen työpanoksensa edustamillaan tieteenaloilla. Edelleen sekoitat ihmisten uskonnot ja heidän asiantuntijatyönsä jollakin muulla alueella.

        Mutta tuokin on eräs kreationismille tyypillinen argumentointivirhe, eli asioiden sekoittaminen keskenään, eli kun kreationistit tekee jotain tiedettä, tekee hän muka kreationistista tiedettä, vaikka oikeasti jos tehdään tiedettä, niin silloin ne uskonnot jätetään ulkovaatteiden mukana labran narikkaan, niin sanotusti. Tieteen kannalta ei ole väliä mihin tieteilijä uskoo tai ei usko, kunhan hoitaa tieteellisen työnsä asiallisesti.

        Ja kun minä sanoin, ettei kreationismilla ole mitään annettavaa tieteelle, tarkotin juuri sitä, että kreationismi on uskonnollisen apologian motivoimaa pseudotiedettä, mikä ei paini tieteen kanssa edes samassa sarjassa. Silloin minä en ole sanonut niin, etteivätkö luomisuskoiset voisi tehdä tiedettä.

        Ja kun minä nyt totean, ettei kaltaisesi vääristelijän kanssa tosiaan viitsi jatkaa tätä keskustelua, niin sinä varmaan väität, että luikin karkuun jotenkin. Katso kuitenkin sitä ennen peiliin ja mieti mitä olit itse minulle vastannut. Mieti sitten, että miksi kukaan viitsisi käyttää tämän enempää aikaansa tuommoiseen pelehtimiseen.

        "Eivät he antaneet tieteelle kreationismia, vaan tieteellisen työpanoksensa edustamillaan tieteenaloilla. Edelleen sekoitat ihmisten uskonnot ja heidän asiantuntijatyönsä jollakin muulla alueella. "

        Sanoit että kreationistit eivät anna mitään tieteelle. He olivat kreationisteja ja antoivat tieteelle sen merkittävimmät osat. Esim. Isac Newton laski myös Raamatun kautta maailman ikää jne. ja oli todellakin harras kristitty. Te ateistit tänä päivänä kunnoiotatte heitä ja nostatte ylös, mutta samalla peittelette sen että he olivat samanlaisia kreationisteja kuin nykyajan uskovatkin, joita halveksutte.

        "Mutta tuokin on eräs kreationismille tyypillinen argumentointivirhe, eli asioiden sekoittaminen keskenään, eli kun kreationistit tekee jotain tiedettä, tekee hän muka kreationistista tiedettä, vaikka oikeasti jos tehdään tiedettä, niin silloin ne uskonnot jätetään ulkovaatteiden mukana labran narikkaan, niin sanotusti. Tieteen kannalta ei ole väliä mihin tieteilijä uskoo tai ei usko, kunhan hoitaa tieteellisen työnsä asiallisesti."

        Tuo on ateisteille tyypillinen argumentaatiovirhe. Tämä tuli yliopistomaailman aatteeksi 1800-luvulla. Haluttiin leikata uskonto pois tiedemaailmasta ja syy oli ideologinen ei tieteellinen. Tieteen kannalta uskolla on paljon väliä. Esimerkiksi Isac Newton loi teoriansa uskoen, että Jumala vaikuttaa maailmassa. Ennen Newtonia oli voimassa ajatus, että kappale voi vaikuttaa toiseen vain kosketuksen kautta (Antiikin Kreikka). Newton ajatteli boxin ulkopuolelta, koska hän uskoi että Jumala voi toimia toisin. Kappale voi vaikuttaa toiseen ilman kosketusta. Tässä yksi lukuisista esimerkeistä, joissa tiede kehittyi käsi kädessä kristinuskon kanssa.

        "Ja kun minä sanoin, ettei kreationismilla ole mitään annettavaa tieteelle, tarkotin juuri sitä, että kreationismi on uskonnollisen apologian motivoimaa pseudotiedettä, mikä ei paini tieteen kanssa edes samassa sarjassa. Silloin minä en ole sanonut niin, etteivätkö luomisuskoiset voisi tehdä tiedettä."

        Yrität jotenkin tässä esittää, että olisi vain yksi tiede, joka on totuus ja sitten on jotain apologiaa tms. Todellisuudessa tieteessä on virheellisiä teorioita. Se on osa koko tiedettä. Niitä sitten koetellaan ja vertaillaan. Samalla tavalla tieteen sisällä on lukuisia eri metodeja. Esimerkiksi matemaatikko tekee eri metodien avulla tiedettä verrattuna vaikkapa yhteiskuntatieteilijään. Ei ole olemassa yhtä tiedettä, joka olisi täsmälleen tietynlaista ja totta.

        "Ja kun minä nyt totean, ettei kaltaisesi vääristelijän kanssa tosiaan viitsi jatkaa tätä keskustelua, niin sinä varmaan väität, että luikin karkuun jotenkin. Katso kuitenkin sitä ennen peiliin ja mieti mitä olit itse minulle vastannut. Mieti sitten, että miksi kukaan viitsisi käyttää tämän enempää aikaansa tuommoiseen pelehtimiseen. "

        En ajatellut että luikit karkuun. Enkä ole mitään vääristellyt. Sori jos aiheutin säröjä täydelliseen maailmankuvaasi. Yksi merkittävä syy miksi itse osallistun näihin väittelyihin on se, että haluaisin todella koetella omaa maailmankuvaani ja saada asiallisia argumentteja, jotka mahdollisesti kumoaisi omat virheelliset käsitykseni. Tämä kiinnostus maailmaan on vienyt minua pitkälle tällaisissa asioissa. Valtaosa vaan ei osaa keskustella vaan raivostuu, jos esitän jonkin vasta-argumentin. Itse lähdin tutkimaan evoluutiokritiikkiä täysin neutraalisti ja totesin, että kritiikki oli monessa kohtaa aiheellista. Harmi jos itse et jostain syystä pysty käsittelemään asiaa neutraalisti.


      • Tämän_oppinut
        evlut101 kirjoitti:

        "Eivät he antaneet tieteelle kreationismia, vaan tieteellisen työpanoksensa edustamillaan tieteenaloilla. Edelleen sekoitat ihmisten uskonnot ja heidän asiantuntijatyönsä jollakin muulla alueella. "

        Sanoit että kreationistit eivät anna mitään tieteelle. He olivat kreationisteja ja antoivat tieteelle sen merkittävimmät osat. Esim. Isac Newton laski myös Raamatun kautta maailman ikää jne. ja oli todellakin harras kristitty. Te ateistit tänä päivänä kunnoiotatte heitä ja nostatte ylös, mutta samalla peittelette sen että he olivat samanlaisia kreationisteja kuin nykyajan uskovatkin, joita halveksutte.

        "Mutta tuokin on eräs kreationismille tyypillinen argumentointivirhe, eli asioiden sekoittaminen keskenään, eli kun kreationistit tekee jotain tiedettä, tekee hän muka kreationistista tiedettä, vaikka oikeasti jos tehdään tiedettä, niin silloin ne uskonnot jätetään ulkovaatteiden mukana labran narikkaan, niin sanotusti. Tieteen kannalta ei ole väliä mihin tieteilijä uskoo tai ei usko, kunhan hoitaa tieteellisen työnsä asiallisesti."

        Tuo on ateisteille tyypillinen argumentaatiovirhe. Tämä tuli yliopistomaailman aatteeksi 1800-luvulla. Haluttiin leikata uskonto pois tiedemaailmasta ja syy oli ideologinen ei tieteellinen. Tieteen kannalta uskolla on paljon väliä. Esimerkiksi Isac Newton loi teoriansa uskoen, että Jumala vaikuttaa maailmassa. Ennen Newtonia oli voimassa ajatus, että kappale voi vaikuttaa toiseen vain kosketuksen kautta (Antiikin Kreikka). Newton ajatteli boxin ulkopuolelta, koska hän uskoi että Jumala voi toimia toisin. Kappale voi vaikuttaa toiseen ilman kosketusta. Tässä yksi lukuisista esimerkeistä, joissa tiede kehittyi käsi kädessä kristinuskon kanssa.

        "Ja kun minä sanoin, ettei kreationismilla ole mitään annettavaa tieteelle, tarkotin juuri sitä, että kreationismi on uskonnollisen apologian motivoimaa pseudotiedettä, mikä ei paini tieteen kanssa edes samassa sarjassa. Silloin minä en ole sanonut niin, etteivätkö luomisuskoiset voisi tehdä tiedettä."

        Yrität jotenkin tässä esittää, että olisi vain yksi tiede, joka on totuus ja sitten on jotain apologiaa tms. Todellisuudessa tieteessä on virheellisiä teorioita. Se on osa koko tiedettä. Niitä sitten koetellaan ja vertaillaan. Samalla tavalla tieteen sisällä on lukuisia eri metodeja. Esimerkiksi matemaatikko tekee eri metodien avulla tiedettä verrattuna vaikkapa yhteiskuntatieteilijään. Ei ole olemassa yhtä tiedettä, joka olisi täsmälleen tietynlaista ja totta.

        "Ja kun minä nyt totean, ettei kaltaisesi vääristelijän kanssa tosiaan viitsi jatkaa tätä keskustelua, niin sinä varmaan väität, että luikin karkuun jotenkin. Katso kuitenkin sitä ennen peiliin ja mieti mitä olit itse minulle vastannut. Mieti sitten, että miksi kukaan viitsisi käyttää tämän enempää aikaansa tuommoiseen pelehtimiseen. "

        En ajatellut että luikit karkuun. Enkä ole mitään vääristellyt. Sori jos aiheutin säröjä täydelliseen maailmankuvaasi. Yksi merkittävä syy miksi itse osallistun näihin väittelyihin on se, että haluaisin todella koetella omaa maailmankuvaani ja saada asiallisia argumentteja, jotka mahdollisesti kumoaisi omat virheelliset käsitykseni. Tämä kiinnostus maailmaan on vienyt minua pitkälle tällaisissa asioissa. Valtaosa vaan ei osaa keskustella vaan raivostuu, jos esitän jonkin vasta-argumentin. Itse lähdin tutkimaan evoluutiokritiikkiä täysin neutraalisti ja totesin, että kritiikki oli monessa kohtaa aiheellista. Harmi jos itse et jostain syystä pysty käsittelemään asiaa neutraalisti.

        "Sanoit että kreationistit eivät anna mitään tieteelle."

        Ei pidä paikkaansa. Kuten yllä näkyy, niin sanoin, ettei kreationismilla ole mitään annettavaa tieteelle. Myöhemmin selvensin vielä sinulle, että on eri asia kun kreationisti tekee jotain tiedettä kuin se, että se tiede olisi kreationistista. Kreationistista tiedettä ei siis ole, mutta kreationisti voi tehdä jotain tiedettä. Elikkä se, että kreationisti on tehnyt tiedettä ei tarkoita sitä, että kreationismilla olisi ollut tieteelle jotain annettavaa.

        Arvaa huvittaako jatkaa keskustelua kun kommenttisi alkaa suoranaisella vääristelyllä? Minun pitäisi siis varautua siihen, että jokaikisestä asiasta tulee tuplaus sillä tavalla, että joudun selittämään sinulle, etten ole sanonut jotain mitä sinä väität minun sanoneen. Ei tuollaiseen yksinkertaisesti riitä aika eikä tahto, ja se on yksi asia minkä olen teidän kreationistien kanssa keskustelusta täällä oppinut.

        Eivät nämä keskustelut tuolla tavalla etene kun joudutaan heti sivuraiteille teidän vääristelynne takia. KJirjoitat vuolaasti, mikä olisi ihan hyvä asia, mutta ei minua ainakaan huvita käydä tekstejäsi läpi kun odotus on, että pääasiallinen tarkoituksesi niissä on vääristellä sitä mitä juuri yllä sanottiin. En käsitä sitä, että miksi te haluatte tehdä tätä noin. Ainoa selitys minkä keksin on, että maailmankuvanne on sisäisesti ristiriitanen ja vailla faktuaalista perustaa, joten ette pysty käymään siitä keskustelua rehellisesti. Että se siitä, että muka kaipaat asiallista argumentaatiota.


      • Tämän_oppinut
        evlut101 kirjoitti:

        "Eivät kaikki suinkaan vetäydy, mutta hyvä syy vetäytyä on se, että kreationismi perustuu argumentointivirheisiin ja huijauksiin, eikä sitä myllyä pyörittävät kykene aikuisten väliseen keskusteluun."

        Ei kyse ole pelkästään kreationismista vaan tieteellisestä evoluutioteorian kritisoinnista. Sitä ei saisi tehdä, koska loukkaat pyhää lehmää ja olet tällöin kuten tästäkin kirjoituksesta nähdään epä-aikuinen huijari. Voisitko nyt kuitenkin sitten selittää, että mille argumentaatiovirheelle kukin evoluutiota koskeva kritiikki perustuu? Mainitse edes yksi esimerkki.
        "
        Valehtelet:

        en.wikipedia.org/wiki/Evidence_of_common_descent
        en.wikipedia.org/wiki/Level_of_support_for_evolution
        en.wikipedia.org/wiki/List_of_scientific_bodies_explicitly_rejecting_intelligent_design"

        Klassinen argumentaatiovirhe. Linkitetään läjäkaupalla tekstiä eikä sanota oikeasti mitään. Kerro ihan omin sanoin että mitä halusit tässä kertoa? Samanlainen argumentaatiovirhe olisi perustella väitteensä Raamatulla ja antaa sitten Raamattu kouraan ja sanoa että väitteen todiste löytyy sieltä. Että luo koko Raamattu läpi, niin sieltä se löytyy. Tieteelliseen metodiin kuuluu ajatus siitä, että jos väität jotain, niin esität väitettä koskevat todisteet yksityiskohtaisesti. Ei riitä että linkittelee jotain epämääräisiä sivustoja. Samalla tavalla toimii myös oikeusistuin, joka vetoaa aina tiettyyn lainkohtaan antaessaan tuomion.

        "Evoluutiouskovaisuudesta puhuminen on kreationistien projisointia eikä mitään muuta. Tiedätte, että itse edustatte uskontoa, niin yritätte siten vähätellä vastapuolta nimittämällä senkin edustajia uskonnoksi; näin vahingossa myönnätte, että tiedätte omaa kantaanne pidettävän vähempiarvoisena."

        Luokittelen evoluutioteorian nykyään uskonnoksi lähinnä tieteellisistä syistä. Koska se ei täytä tieteen tunnusmerkkejä ja sisältää läjäpäin dataa, joka kumoaa sen. Esimerkiksi fossiiliaineiston punktualismi todistaa, että lajit eivät muutu ajan saatossa, mikä puolestaan on jo riittävä todiste koko teorian kumoamiselle.

        Toisekseen esimerkikisi tämä mies on sitä mieltä että evoluutioteoria on valhetta. Ja kas kummaa hän on ateisti ja professori
        https://fi.wikipedia.org/wiki/Thomas_Nagel

        Onko hänkin siis kreationisti, kun ei kerran usko evoluutioon?

        "
        Ja on tosiaan nähty, että kun sinulle tarjotaan kaikki todisteet, niin huomautat, että pyysit vain yhtä: ikään kuin et voisi aikuinen (?) ihminen valita niistä kaikista tai mielummin vain katsoa ne kaikki läpi. Yhteensopivien osien kokonaisuudestahan tässä kuitenkin on kyse, ja silloin tuollainen, että vastaus on muka väärä kun lueteltiin enemmän kuin yksi todiste, on suorastaan lapsellista vinoilua."

        Ei vaan juuri se että heitetään tuhatsivuisia tekstipätkiä ja sanotaan että täällä se todistetaan. Se on todistus siitä, ettei yhtäkään kunnon todistetta löydy.

        "Myönsit tuossa vahingossa, että kreationistit eivät halua kuulla tosiasioita, mutta peittelit sen halvalla projisointiyrityksellä. Toistat jo tuossa vaiheessa samaa argumentointivirhettä, eli projisointia uskosta."

        Kyse ei ole mistään projisoinnista vaan rinnastan kaikki sellaiset teoriat/hypoteesit uskonnoiksi, joille ei löydy uskottavia tieteellisiä todisteita, mutta joihin silti uskotaan horjumatta. Jo se että evoluutioteoriaa ei saisi kritisoida antaa sille uskonnollisen leiman. Nimittäin tieteellisille teorioille on ominaista, että niitä saa ja kuuluu koetella. Jos koettelu johtaa heti hallitsemattomaan tunnereaktioon, niin kyse on uskonnosta.

        "En ole sanonut tuommoista, joten asiaan liittymättömät vouhotuksesi laboratoriokokeista yms. tuota asiaa muka koskien voikin ohittaa. Kysehän oli siitä, että tiedän tietäväni jotain, mitä jotkut muut eivät tiedä, siinä kaikki. "

        Turha tässä on alkaa raivoamaan. Ja millä perusteella tieteelliset kriteerit voi muka ohittaa ja silti väittää, että kyseinen teoria on täyttä faktaa?

        Maailma on muuten täynnä ihmisiä, jotka luulee tietävänsä. Ettet vaan olisi yksi heistä.

        "
        Teet itsestäsi naurunalaisen kun alat laittaa sanoja suuhuni. Mutta eivätpä kreationistit yleensä muuhun pysty. Joka kerta melkein sama juttu, että vääristellään sitä mistä oli puhe.
        "

        Argumentum ad hominem sanoisin puhtaimmassa muodossaan. :D
        Minä kun olen "kreationisti" ja siksi en pysty muuhun kun sanojen suuhun laittamiseen.

        "Äläpäs innostu "auktoriteetteinesi." Evoluution kiistäminen uskonnollisista syistä, mikä ihan oikeudenkäyntejä myöten osoitetusti on se mitä kreationistiorganisaatiot tekevät, ja minkä myötä kreationismiin höpsähtäneet kreationismia paasaavat, käytännössä nimittäin vetää maton alta tuolta auktoriteettirehvastelultasi. Evoluutiokriittisyys ei perustu tieteeseen, vaan se perustuu uskontoon, ja silloin on turhaa koettaa ratsastaa "evoluutiokriittisten" tieteilijöiden auktoriteetilla."

        Ja mistä lähtien oikeudenkäynnissä päätetään mikä on tieteellinen totuus ja mikä ei? Käsittääkseni yliopistot on sitä varten. Ja väitätkö vakavissasi että jenkkituomioistuin on sinusta erehtymätön.

        Mutta vielä tästä vähän, kun satuin huomaamaan, että olit kommentoinut enemmänkin:

        "Linkitetään läjäkaupalla tekstiä eikä sanota oikeasti mitään. Kerro ihan omin sanoin että mitä halusit tässä kertoa? Samanlainen argumentaatiovirhe olisi perustella väitteensä Raamatulla ja antaa sitten Raamattu kouraan ja sanoa että väitteen todiste löytyy sieltä. Että luo koko Raamattu läpi, niin sieltä se löytyy. Tieteelliseen metodiin kuuluu ajatus siitä, että jos väität jotain, niin esität väitettä koskevat todisteet yksityiskohtaisesti. Ei riitä että linkittelee jotain epämääräisiä sivustoja."

        Älä jauha paskaa. Sait muutaman linkin, joissa nimenomaan erittäin yksityiskohtaisesti asia kerrallaan käydään läpi niitä tekijöitä, joiden mukaan evoluutio on hyvin havaittu ja dokumentointu ilmiö, josta ollaan asiantuntijain kesken yksimielisiä, ja lisäksi osoitettiin, että tiedemaailmassa merkittävät tahot nimenomaan irtisanoutuvat älykkään suunnittelun ideoista jne.

        Ja sinullehan oli jo kerrottu siitä, että on argumentointivirhe vaatia evoluutiolle vain yhtä todistetta, kun kyse on monien eri asioiden muodostamasta kokonaisuudesta. Yrität siis pedata asian niin, että jos et saa vain yhtä todistetta, niin hylkäät kaiken, vaikka mitään todellista sääntöä sille, että jokin asia pitäisi voida esittää vain yhdellä todisteella ei tietenkään ole!

        Se on pelkkä argumentointivirhe sinun taholtasi, ja olemukseltaan se on vielä omiaan osoittamaan, että pelkäät nähdä todisteita ja varmasti inhoat sitä, että niitä on kuin onkin oikein roppakaupalla, kuten sinulle on jo osoitettu! Ja vaikka sinä et niitä lue, niin tajua nyt hyvä ihminen se, että joku muu kyllä lukee, ja siinä tullaan tehdyksi tyhjäksi sinun pseudoskeptismisi evoluutiota kohtaan.

        Sait siis vain muutaman linkin missä on koostettuna paljon tietoa juuri tarkoituksella valikoidusti noista aiheista, mitkä ovat oleellisia koskien sitä mistä keskustellaan. Ei ole todellakaan sama asia lätkäistä pelkkä Raamattu tiskiin ja sanoa, että lue tuosta! Jos sinä tosissasi väität tuommoista, niin sinun täytyy olla ihan höppänä. Eihän kukaan rinnasta muutamaa asiakokonaisuudeltaan varsin tiivistettyä linkkiä johonkin mytologiateokseen... eihän?

        Ei sinulle linkitetty jotain epäyksityiskohtaisia ja epämääräisiä sivustoja, vaan sait tietosanakirjatasoista matskua, missä on kullekin asialle omat viitteensä. Tämä asia on monimutkainen, etkä voi onnistua - ainakaan jos minä olen vahtivuorossa, niin sanotusti - vakuuttaa yhtäkään normaalijärkistä ja rehellistä lukijaa täällä siitä, että olisi jotenkin väärin linkittää tuollainen kokoelma, mistä pääsee tutkimaan eri asioita viitteineen.

        Ei ellet tosiaan ole niin epäreilu kuin annoit jo olettaa, että haluat vain yhden todisteen kerrallaan, jotta pääsisit vähemmällä ja voisit vain halvalla väittää, että "ei tuo ole uskottava". Totuushan on, että yhdessä kaikki tuo evidenssi osoittaa jotain mitä sinä et voi hyväksyä, niin yrität joko pilkkoa sen pienempiin, vähäisemmällä kintaanheilautuksella muka kuitattaviin osiin tai sitten vaihtoehtoisesti väittää, että olisi muka joku argumentointivirhe antaa kerralla koko setti. Salli mun nauraa!

        Olen ihmetellyt tuota kreationismin perustavanlaatuista epärehellisyyttä ja kieroutta kaiken sen ajan kuin olen sitä kohdannut, ja olen periaatteessa ollut valmis hyväksymään sen, että tietämättömät ihmiset voivat moiseen sortua. Mutta sinä kutienkin kirjoitat jotain perushakkaraista nasevammin ja selkeästi pystyt käsittelemään asioita, vaikka et selkeästikään ole sitä tehdessäsi rehellinen. Olen pettynyt siihen, että periaatteessa OK-tasoisen ihmisenkin voi pilata fundamentalistinen usko noin, että olet valmis tuollaiseen lapselliseen kieroiluun ja pelehtimiseen.

        Ei minulla ole tosiaan tahtoa ja aikaa paljoa kuten olen jo sanonut, ja ehdottomasti teen mielummin jotain muuta kuin kilpailen jonkun pänkkipään kanssa netissä. Voin itse opiskella asioita ja kehittyä edelleen. Vuoden päästä jos käyn katsomassa täällä, niin sinä varmasti edelleen jankutat sitä jo monta kertaa kumottua väärinkäsitystäsi "pullonkaulateoriasta". Tätä on ainakin se kreationismi, mitä olen itse vuosien varrella oppinut tuntemaan, enkä itse enää oikein jaksa tuota oravanpyöräänne.

        Mitään pahaa en tietenkään sinullekaan tahdo. Tahtoisin vain, että ryhdistäytyisit, mutta toisaalta asia ei kuulu minulle. Tee mitä lystäät. Olen toistaiseksi puhunut.


      • evlut101
        evlut101 kirjoitti:

        "Eivät he antaneet tieteelle kreationismia, vaan tieteellisen työpanoksensa edustamillaan tieteenaloilla. Edelleen sekoitat ihmisten uskonnot ja heidän asiantuntijatyönsä jollakin muulla alueella. "

        Sanoit että kreationistit eivät anna mitään tieteelle. He olivat kreationisteja ja antoivat tieteelle sen merkittävimmät osat. Esim. Isac Newton laski myös Raamatun kautta maailman ikää jne. ja oli todellakin harras kristitty. Te ateistit tänä päivänä kunnoiotatte heitä ja nostatte ylös, mutta samalla peittelette sen että he olivat samanlaisia kreationisteja kuin nykyajan uskovatkin, joita halveksutte.

        "Mutta tuokin on eräs kreationismille tyypillinen argumentointivirhe, eli asioiden sekoittaminen keskenään, eli kun kreationistit tekee jotain tiedettä, tekee hän muka kreationistista tiedettä, vaikka oikeasti jos tehdään tiedettä, niin silloin ne uskonnot jätetään ulkovaatteiden mukana labran narikkaan, niin sanotusti. Tieteen kannalta ei ole väliä mihin tieteilijä uskoo tai ei usko, kunhan hoitaa tieteellisen työnsä asiallisesti."

        Tuo on ateisteille tyypillinen argumentaatiovirhe. Tämä tuli yliopistomaailman aatteeksi 1800-luvulla. Haluttiin leikata uskonto pois tiedemaailmasta ja syy oli ideologinen ei tieteellinen. Tieteen kannalta uskolla on paljon väliä. Esimerkiksi Isac Newton loi teoriansa uskoen, että Jumala vaikuttaa maailmassa. Ennen Newtonia oli voimassa ajatus, että kappale voi vaikuttaa toiseen vain kosketuksen kautta (Antiikin Kreikka). Newton ajatteli boxin ulkopuolelta, koska hän uskoi että Jumala voi toimia toisin. Kappale voi vaikuttaa toiseen ilman kosketusta. Tässä yksi lukuisista esimerkeistä, joissa tiede kehittyi käsi kädessä kristinuskon kanssa.

        "Ja kun minä sanoin, ettei kreationismilla ole mitään annettavaa tieteelle, tarkotin juuri sitä, että kreationismi on uskonnollisen apologian motivoimaa pseudotiedettä, mikä ei paini tieteen kanssa edes samassa sarjassa. Silloin minä en ole sanonut niin, etteivätkö luomisuskoiset voisi tehdä tiedettä."

        Yrität jotenkin tässä esittää, että olisi vain yksi tiede, joka on totuus ja sitten on jotain apologiaa tms. Todellisuudessa tieteessä on virheellisiä teorioita. Se on osa koko tiedettä. Niitä sitten koetellaan ja vertaillaan. Samalla tavalla tieteen sisällä on lukuisia eri metodeja. Esimerkiksi matemaatikko tekee eri metodien avulla tiedettä verrattuna vaikkapa yhteiskuntatieteilijään. Ei ole olemassa yhtä tiedettä, joka olisi täsmälleen tietynlaista ja totta.

        "Ja kun minä nyt totean, ettei kaltaisesi vääristelijän kanssa tosiaan viitsi jatkaa tätä keskustelua, niin sinä varmaan väität, että luikin karkuun jotenkin. Katso kuitenkin sitä ennen peiliin ja mieti mitä olit itse minulle vastannut. Mieti sitten, että miksi kukaan viitsisi käyttää tämän enempää aikaansa tuommoiseen pelehtimiseen. "

        En ajatellut että luikit karkuun. Enkä ole mitään vääristellyt. Sori jos aiheutin säröjä täydelliseen maailmankuvaasi. Yksi merkittävä syy miksi itse osallistun näihin väittelyihin on se, että haluaisin todella koetella omaa maailmankuvaani ja saada asiallisia argumentteja, jotka mahdollisesti kumoaisi omat virheelliset käsitykseni. Tämä kiinnostus maailmaan on vienyt minua pitkälle tällaisissa asioissa. Valtaosa vaan ei osaa keskustella vaan raivostuu, jos esitän jonkin vasta-argumentin. Itse lähdin tutkimaan evoluutiokritiikkiä täysin neutraalisti ja totesin, että kritiikki oli monessa kohtaa aiheellista. Harmi jos itse et jostain syystä pysty käsittelemään asiaa neutraalisti.

        "Elikkä se, että kreationisti on tehnyt tiedettä ei tarkoita sitä, että kreationismilla olisi ollut tieteelle jotain annettavaa. "

        Tiede on täynnä hypoteeseja, jotka eivät ole kovin vahvalla pohjalla. Se on oleellinen osa tiedettä. Miksi joku yksittäinen teoria on aivan mahdotonta hyväksyä, mikäli sen pohjana on Jumala? Kai ymmärrät että tuollainen ajattelu ei ole mitenkään tieteellistä. Et voi poimia rusinoita pullasta sen mukaan, että mikä tieteellinen teoria tukis parhaiten omaa maailmankuvaasi. Jos kreationisti tekee tiedettä ja haastaa esim. nykyiset ajoitusmenetelmät, niin hän a) saattaa tarkentaa ja kehittää ajoitusmenetelmiä ja b) myös mahdollisesti paljastaa virheellistä ajattelua. Siinä ei ole mitään pahaa. Jokainen tieteentekijä on ihminen ja siksi asenteellinen. Yksi uskoo Jumalaan ja toinen ei. Oleellista on lopuksi pistää eri mallit vastakkain ja päättää että mikä on uskottavin vai onko mikään.

        "Arvaa huvittaako jatkaa keskustelua kun kommenttisi alkaa suoranaisella vääristelyllä?"

        No itse a) kutsuit minua kreationistiksi ja b) sitten haukuit kreationistit. Eikös tuo ole sivuraide jos ajatellaan evoluutioteoriaa koskevaa kritiikkiä? Siis näin

        " että kreationismi perustuu argumentointivirheisiin ja huijauksiin, eikä sitä myllyä pyörittävät kykene aikuisten väliseen keskusteluun.""

        "Älä jauha paskaa. Sait muutaman linkin, joissa nimenomaan erittäin yksityiskohtaisesti asia kerrallaan käydään läpi niitä tekijöitä, joiden mukaan evoluutio on hyvin havaittu ja dokumentointu ilmiö, josta ollaan asiantuntijain kesken yksimielisiä, ja lisäksi osoitettiin, että tiedemaailmassa merkittävät tahot nimenomaan irtisanoutuvat älykkään suunnittelun ideoista jne. "

        Niin eli miten sitten jatketaan tuosta? Aletaanko käymään kohta kohdalta läpi noita "todisteita"? Todeksi luettavat tieteelliset teoriat perustuvat aina johonkin vankkaan todistusaineistoon. Ei riitä että on monta heikkoa todistusta vaan pitää olla ainakin yksi vahva todiste. Ja sellaista ei sinulta löydy, jota ei olisi helposti kumottu.

        "Yrität siis pedata asian niin, että jos et saa vain yhtä todistetta, niin hylkäät kaiken, vaikka mitään todellista sääntöä sille, että jokin asia pitäisi voida esittää vain yhdellä todisteella ei tietenkään ole! "

        Olenhan minä lähes kaikki nuo todisteet lukenut läpi. Valtaosa on aihetodisteita. Ensin oletetaan että evoluutio on tosi ja sitten todetaan, että havainnot tukee teoriaa. Esimerkiksi lajien samankaltaisuuteen liittyvät todisteet eivät ole todiste yhdentyyppisestä lajien synnystä vaan ne voidaan selittää monen eri teorian kautta. Ne eivät yksinkertaisesti todista evoluutiota, vaikka sopivat evoluution teoriaan. Käsitätkö?

        Ja on myös vastatodisteita kuten punktualismi. Lajithan eivät muutu fossiilidatassa teorian mukaisesti


      • evlut101 kirjoitti:

        "Elikkä se, että kreationisti on tehnyt tiedettä ei tarkoita sitä, että kreationismilla olisi ollut tieteelle jotain annettavaa. "

        Tiede on täynnä hypoteeseja, jotka eivät ole kovin vahvalla pohjalla. Se on oleellinen osa tiedettä. Miksi joku yksittäinen teoria on aivan mahdotonta hyväksyä, mikäli sen pohjana on Jumala? Kai ymmärrät että tuollainen ajattelu ei ole mitenkään tieteellistä. Et voi poimia rusinoita pullasta sen mukaan, että mikä tieteellinen teoria tukis parhaiten omaa maailmankuvaasi. Jos kreationisti tekee tiedettä ja haastaa esim. nykyiset ajoitusmenetelmät, niin hän a) saattaa tarkentaa ja kehittää ajoitusmenetelmiä ja b) myös mahdollisesti paljastaa virheellistä ajattelua. Siinä ei ole mitään pahaa. Jokainen tieteentekijä on ihminen ja siksi asenteellinen. Yksi uskoo Jumalaan ja toinen ei. Oleellista on lopuksi pistää eri mallit vastakkain ja päättää että mikä on uskottavin vai onko mikään.

        "Arvaa huvittaako jatkaa keskustelua kun kommenttisi alkaa suoranaisella vääristelyllä?"

        No itse a) kutsuit minua kreationistiksi ja b) sitten haukuit kreationistit. Eikös tuo ole sivuraide jos ajatellaan evoluutioteoriaa koskevaa kritiikkiä? Siis näin

        " että kreationismi perustuu argumentointivirheisiin ja huijauksiin, eikä sitä myllyä pyörittävät kykene aikuisten väliseen keskusteluun.""

        "Älä jauha paskaa. Sait muutaman linkin, joissa nimenomaan erittäin yksityiskohtaisesti asia kerrallaan käydään läpi niitä tekijöitä, joiden mukaan evoluutio on hyvin havaittu ja dokumentointu ilmiö, josta ollaan asiantuntijain kesken yksimielisiä, ja lisäksi osoitettiin, että tiedemaailmassa merkittävät tahot nimenomaan irtisanoutuvat älykkään suunnittelun ideoista jne. "

        Niin eli miten sitten jatketaan tuosta? Aletaanko käymään kohta kohdalta läpi noita "todisteita"? Todeksi luettavat tieteelliset teoriat perustuvat aina johonkin vankkaan todistusaineistoon. Ei riitä että on monta heikkoa todistusta vaan pitää olla ainakin yksi vahva todiste. Ja sellaista ei sinulta löydy, jota ei olisi helposti kumottu.

        "Yrität siis pedata asian niin, että jos et saa vain yhtä todistetta, niin hylkäät kaiken, vaikka mitään todellista sääntöä sille, että jokin asia pitäisi voida esittää vain yhdellä todisteella ei tietenkään ole! "

        Olenhan minä lähes kaikki nuo todisteet lukenut läpi. Valtaosa on aihetodisteita. Ensin oletetaan että evoluutio on tosi ja sitten todetaan, että havainnot tukee teoriaa. Esimerkiksi lajien samankaltaisuuteen liittyvät todisteet eivät ole todiste yhdentyyppisestä lajien synnystä vaan ne voidaan selittää monen eri teorian kautta. Ne eivät yksinkertaisesti todista evoluutiota, vaikka sopivat evoluution teoriaan. Käsitätkö?

        Ja on myös vastatodisteita kuten punktualismi. Lajithan eivät muutu fossiilidatassa teorian mukaisesti

        Suosittelen, että lähdet tieteen tekijäksi ja opiskelet näitä kohta kohdalta tarkemmin. Perehdyt siis tieteelliseen aineistoon, etkä mihinkään uskonnollisiin toisen mielipiteisiin tai päätelmiin.


      • evlut101
        mummomuori kirjoitti:

        Suosittelen, että lähdet tieteen tekijäksi ja opiskelet näitä kohta kohdalta tarkemmin. Perehdyt siis tieteelliseen aineistoon, etkä mihinkään uskonnollisiin toisen mielipiteisiin tai päätelmiin.

        Mistä tiedät etten olisikin tieteen tekijä.

        Joka tapaukessa on olemassa professoreita, jotka uskoo Jumalaan ja jotka eivät usko evoluutioon. On jopa ateistisia tieteentekijöitä, jotka eivät pidä evoluutioteoriaa uskottavana. Turha siis vedota siihen, että tieteen tekeminen tekisi ihmisestä automaattisesti ateistin ja evoluutiouskovaisen.

        Myös tiedeusko on yhdenlainen uskonto. Sitä kutsutaan myös nimellä skientismi. Sille tyypillisiä opinkappaleita on käsitys siitä, että tiede pystyy ennemmin tai myöhemmin ratkaisemaan kaikki ongelmat (kuten että miten maailma on syntynyt ja mistä ihmise tulevat). Skientisti ei näe eroa eri tieteellisten teorioiden luotettavuuden välillä. Hänelle Newtonin painovoimalaki ja Darwinin evoluutioteoria on samanarvoista totuutta samoin kuin alkuräjähdysteoria. Hän kuvittelee, että tiede on yksimielinen kaikessa ja että tieteellinen metodi on yksikäsitteinen ja erehtymätön tai että se vähintäänkin tarjoaa aina parhaan totuuden.


      • evlut101 kirjoitti:

        Mistä tiedät etten olisikin tieteen tekijä.

        Joka tapaukessa on olemassa professoreita, jotka uskoo Jumalaan ja jotka eivät usko evoluutioon. On jopa ateistisia tieteentekijöitä, jotka eivät pidä evoluutioteoriaa uskottavana. Turha siis vedota siihen, että tieteen tekeminen tekisi ihmisestä automaattisesti ateistin ja evoluutiouskovaisen.

        Myös tiedeusko on yhdenlainen uskonto. Sitä kutsutaan myös nimellä skientismi. Sille tyypillisiä opinkappaleita on käsitys siitä, että tiede pystyy ennemmin tai myöhemmin ratkaisemaan kaikki ongelmat (kuten että miten maailma on syntynyt ja mistä ihmise tulevat). Skientisti ei näe eroa eri tieteellisten teorioiden luotettavuuden välillä. Hänelle Newtonin painovoimalaki ja Darwinin evoluutioteoria on samanarvoista totuutta samoin kuin alkuräjähdysteoria. Hän kuvittelee, että tiede on yksimielinen kaikessa ja että tieteellinen metodi on yksikäsitteinen ja erehtymätön tai että se vähintäänkin tarjoaa aina parhaan totuuden.

        On varmaan olemassa " professoreita, jotka uskoo Jumalaan ja jotka eivät usko evoluutioon." mutta eihän se ole todiste mistään,ei siitä ettei ole evoluutiota eikä siitä on olemassa Jumala.Jumalaan vain uskotaan mutta evoluutiosta on olemassa todisteet eikä kukaan ole pystynyt esittäämään evoluutioteorian sijalle korvaavaa teoriaa jolle olisi todisteet . Sinä vain et nyt ymärrä tätä.


      • Tämän_oppinut
        evlut101 kirjoitti:

        "Elikkä se, että kreationisti on tehnyt tiedettä ei tarkoita sitä, että kreationismilla olisi ollut tieteelle jotain annettavaa. "

        Tiede on täynnä hypoteeseja, jotka eivät ole kovin vahvalla pohjalla. Se on oleellinen osa tiedettä. Miksi joku yksittäinen teoria on aivan mahdotonta hyväksyä, mikäli sen pohjana on Jumala? Kai ymmärrät että tuollainen ajattelu ei ole mitenkään tieteellistä. Et voi poimia rusinoita pullasta sen mukaan, että mikä tieteellinen teoria tukis parhaiten omaa maailmankuvaasi. Jos kreationisti tekee tiedettä ja haastaa esim. nykyiset ajoitusmenetelmät, niin hän a) saattaa tarkentaa ja kehittää ajoitusmenetelmiä ja b) myös mahdollisesti paljastaa virheellistä ajattelua. Siinä ei ole mitään pahaa. Jokainen tieteentekijä on ihminen ja siksi asenteellinen. Yksi uskoo Jumalaan ja toinen ei. Oleellista on lopuksi pistää eri mallit vastakkain ja päättää että mikä on uskottavin vai onko mikään.

        "Arvaa huvittaako jatkaa keskustelua kun kommenttisi alkaa suoranaisella vääristelyllä?"

        No itse a) kutsuit minua kreationistiksi ja b) sitten haukuit kreationistit. Eikös tuo ole sivuraide jos ajatellaan evoluutioteoriaa koskevaa kritiikkiä? Siis näin

        " että kreationismi perustuu argumentointivirheisiin ja huijauksiin, eikä sitä myllyä pyörittävät kykene aikuisten väliseen keskusteluun.""

        "Älä jauha paskaa. Sait muutaman linkin, joissa nimenomaan erittäin yksityiskohtaisesti asia kerrallaan käydään läpi niitä tekijöitä, joiden mukaan evoluutio on hyvin havaittu ja dokumentointu ilmiö, josta ollaan asiantuntijain kesken yksimielisiä, ja lisäksi osoitettiin, että tiedemaailmassa merkittävät tahot nimenomaan irtisanoutuvat älykkään suunnittelun ideoista jne. "

        Niin eli miten sitten jatketaan tuosta? Aletaanko käymään kohta kohdalta läpi noita "todisteita"? Todeksi luettavat tieteelliset teoriat perustuvat aina johonkin vankkaan todistusaineistoon. Ei riitä että on monta heikkoa todistusta vaan pitää olla ainakin yksi vahva todiste. Ja sellaista ei sinulta löydy, jota ei olisi helposti kumottu.

        "Yrität siis pedata asian niin, että jos et saa vain yhtä todistetta, niin hylkäät kaiken, vaikka mitään todellista sääntöä sille, että jokin asia pitäisi voida esittää vain yhdellä todisteella ei tietenkään ole! "

        Olenhan minä lähes kaikki nuo todisteet lukenut läpi. Valtaosa on aihetodisteita. Ensin oletetaan että evoluutio on tosi ja sitten todetaan, että havainnot tukee teoriaa. Esimerkiksi lajien samankaltaisuuteen liittyvät todisteet eivät ole todiste yhdentyyppisestä lajien synnystä vaan ne voidaan selittää monen eri teorian kautta. Ne eivät yksinkertaisesti todista evoluutiota, vaikka sopivat evoluution teoriaan. Käsitätkö?

        Ja on myös vastatodisteita kuten punktualismi. Lajithan eivät muutu fossiilidatassa teorian mukaisesti

        "Tiede on täynnä hypoteeseja, jotka eivät ole kovin vahvalla pohjalla. Se on oleellinen osa tiedettä. Miksi joku yksittäinen teoria on aivan mahdotonta hyväksyä, mikäli sen pohjana on Jumala?"

        Sinä yrität edelleen tarjota väärää ekvivalenssia, että kreationismin ja "evolutionismin" vastakkainasettelun tapauksessa olisi vain erilaisia tieteellisiä teorioita, joista osa hyväksytään ja osaa ei. Kyse ei ole siitä!

        Kyse on siitä, että on tiedettä, ja sitten on uskontoa. Ei ole mitään tieteellisiä hypoteeseja saatikka teorioita, joissa esiintyisi Jumala. Koeta nyt käsittää, että usko siihen, että jokin Jumala olisi kaiken takana tai sellaisen uskon puuttuminen eivät ole tiedettä tai mukana tieteessä.

        Tiede on sitä mitä tehdään, kun havaintoja selitetään. Uskomukset siitä, mitä tuon taustalla tai ulkopuolella on tai niiden uskomusten puute, ovat maailmankatsomuksellisia seikkoja, eivät itse sitä tiedettä.

        "Kai ymmärrät että tuollainen ajattelu ei ole mitenkään tieteellistä. Et voi poimia rusinoita pullasta sen mukaan, että mikä tieteellinen teoria tukis parhaiten omaa maailmankuvaasi."

        Naurettavaa projisointia. Sinä itse teet tuota. Minä en tee tuota, vaan minä tiedostan, ettei ole edes mitään kreationistista tiedettä, jonka voisin sivuuttaa.

        "Jos kreationisti tekee tiedettä ja haastaa esim. nykyiset ajoitusmenetelmät, niin hän a) saattaa tarkentaa ja kehittää ajoitusmenetelmiä ja b) myös mahdollisesti paljastaa virheellistä ajattelua. Siinä ei ole mitään pahaa."

        On osoitettu kerta toisensa jälkeen, ettei tuossa ole ollut kyse tieteellisestä toiminnasta, vaan kreationistit ovat pseudotieteilleet mitä sattuu. Ja siinä on pahaa, että sitä tehden yritetään kusettaa ihmisiä luulemaan, että kyse olisi tieteestä.

        "Jokainen tieteentekijä on ihminen ja siksi asenteellinen. Yksi uskoo Jumalaan ja toinen ei."

        Tämä pitää paikkansa.

        "Oleellista on lopuksi pistää eri mallit vastakkain ja päättää että mikä on uskottavin vai onko mikään."

        Jumala ei ole mikään tieteellinen malli, vaan se on yliluonnollinen uskomus.

        "No itse a) kutsuit minua kreationistiksi ja b) sitten haukuit kreationistit. Eikös tuo ole sivuraide jos ajatellaan evoluutioteoriaa koskevaa kritiikkiä?"

        Jos sinä uskot, kuten annat ymmärtää, että olisi olemassa jotain luomistiedettä ja olet kritisoivinasi evoluutioteoriaa niillä perusteilla, olet kreationisti sanan pseudotieteellisen apologetiikan piirteiden merkityksessä, ja sellainen kreationismi on täsmälleen sitä miksikä sitä kuvasin. Tämä on nähty kerta toisensa jälkeen.

        "Niin eli miten sitten jatketaan tuosta? Aletaanko käymään kohta kohdalta läpi noita "todisteita"? Todeksi luettavat tieteelliset teoriat perustuvat aina johonkin vankkaan todistusaineistoon. Ei riitä että on monta heikkoa todistusta vaan pitää olla ainakin yksi vahva todiste. Ja sellaista ei sinulta löydy, jota ei olisi helposti kumottu."

        Tuo on pelkkää denialismia. Sinulle on osoitettu eräs vankimmista tieteellisistä teorioista taustatietoineen. Ainoa keinosi taistella tätä vastaan on keksiä omia sääntöjä, että pitää olla jokin yksi supertodiste, vaikka niin ei ole. Samalla yrität esittää, että ne monet todisteet olisivat jotenkin heikkoja, mutta eiväthän ne sitä ole. Koko toimintasi perustuu juuri siihen lapselliseen epärehellisyyteen, jollaiseksi kreationismia jo luonnehdin. Siksi minä nimitän sinua kreationistiksi juuri tuossa merkityksessä.

        "Olenhan minä lähes kaikki nuo todisteet lukenut läpi. Valtaosa on aihetodisteita. Ensin oletetaan että evoluutio on tosi ja sitten todetaan, että havainnot tukee teoriaa."

        Ei vaan nuo Darwinin jälkeiset havainnot osoittavat, että Darwin oli oikeassa, ja hänen evoluutioteoriansa pitää paikkansa; että siis lajit muuttuvat luonnonvalinnan takia, ja nyt tiedetään mikä aiheuttaa ne piirteet, joihin luonnonvalinta kohdistuu.

        "Esimerkiksi lajien samankaltaisuuteen liittyvät todisteet eivät ole todiste yhdentyyppisestä lajien synnystä vaan ne voidaan selittää monen eri teorian kautta. Ne eivät yksinkertaisesti todista evoluutiota, vaikka sopivat evoluution teoriaan. Käsitätkö?"

        Mutta kun ei ole mitään muita teorioita. Sinä et näköjään ymmärrä mitä teoria tieteessä tarkoittaa. Se, että kaatumaseurasi julistaja kertoo, että amerikkalainen uskonpönkitystä varten muodostettu ajoituspaja on tehnyt "tutkimuksia", ja että heillä on "teoria", ei tarkoita sitä, että olisi olemassa jokin vaihtoehtoinen teoria evoluutiolle.

        "Ja on myös vastatodisteita kuten punktualismi. Lajithan eivät muutu fossiilidatassa teorian mukaisesti"

        Ja tämähän sinulle on myös korjattu, ettei Gouldin jaksoittaisen tasapainon malli tarkoita sitä mitä sinä siitä johdat, ja Gould itse on myös turhautunut siihen, että kreationistit ovat vääristelleet hänen malliaan noin mustavalkoisesti, että lajit eivät muka yhtään muuttuisi ja ilmestyisivät sinne tänne sellaisenaan. Et ole perehtynyt siihen Gouldin juttuun, vaan olet perehtynyt siitä tehtyyn kreationistiseen lainauslouhintaan ja/tai vääristelyyn.


      • Tämän_oppinut kirjoitti:

        "Tiede on täynnä hypoteeseja, jotka eivät ole kovin vahvalla pohjalla. Se on oleellinen osa tiedettä. Miksi joku yksittäinen teoria on aivan mahdotonta hyväksyä, mikäli sen pohjana on Jumala?"

        Sinä yrität edelleen tarjota väärää ekvivalenssia, että kreationismin ja "evolutionismin" vastakkainasettelun tapauksessa olisi vain erilaisia tieteellisiä teorioita, joista osa hyväksytään ja osaa ei. Kyse ei ole siitä!

        Kyse on siitä, että on tiedettä, ja sitten on uskontoa. Ei ole mitään tieteellisiä hypoteeseja saatikka teorioita, joissa esiintyisi Jumala. Koeta nyt käsittää, että usko siihen, että jokin Jumala olisi kaiken takana tai sellaisen uskon puuttuminen eivät ole tiedettä tai mukana tieteessä.

        Tiede on sitä mitä tehdään, kun havaintoja selitetään. Uskomukset siitä, mitä tuon taustalla tai ulkopuolella on tai niiden uskomusten puute, ovat maailmankatsomuksellisia seikkoja, eivät itse sitä tiedettä.

        "Kai ymmärrät että tuollainen ajattelu ei ole mitenkään tieteellistä. Et voi poimia rusinoita pullasta sen mukaan, että mikä tieteellinen teoria tukis parhaiten omaa maailmankuvaasi."

        Naurettavaa projisointia. Sinä itse teet tuota. Minä en tee tuota, vaan minä tiedostan, ettei ole edes mitään kreationistista tiedettä, jonka voisin sivuuttaa.

        "Jos kreationisti tekee tiedettä ja haastaa esim. nykyiset ajoitusmenetelmät, niin hän a) saattaa tarkentaa ja kehittää ajoitusmenetelmiä ja b) myös mahdollisesti paljastaa virheellistä ajattelua. Siinä ei ole mitään pahaa."

        On osoitettu kerta toisensa jälkeen, ettei tuossa ole ollut kyse tieteellisestä toiminnasta, vaan kreationistit ovat pseudotieteilleet mitä sattuu. Ja siinä on pahaa, että sitä tehden yritetään kusettaa ihmisiä luulemaan, että kyse olisi tieteestä.

        "Jokainen tieteentekijä on ihminen ja siksi asenteellinen. Yksi uskoo Jumalaan ja toinen ei."

        Tämä pitää paikkansa.

        "Oleellista on lopuksi pistää eri mallit vastakkain ja päättää että mikä on uskottavin vai onko mikään."

        Jumala ei ole mikään tieteellinen malli, vaan se on yliluonnollinen uskomus.

        "No itse a) kutsuit minua kreationistiksi ja b) sitten haukuit kreationistit. Eikös tuo ole sivuraide jos ajatellaan evoluutioteoriaa koskevaa kritiikkiä?"

        Jos sinä uskot, kuten annat ymmärtää, että olisi olemassa jotain luomistiedettä ja olet kritisoivinasi evoluutioteoriaa niillä perusteilla, olet kreationisti sanan pseudotieteellisen apologetiikan piirteiden merkityksessä, ja sellainen kreationismi on täsmälleen sitä miksikä sitä kuvasin. Tämä on nähty kerta toisensa jälkeen.

        "Niin eli miten sitten jatketaan tuosta? Aletaanko käymään kohta kohdalta läpi noita "todisteita"? Todeksi luettavat tieteelliset teoriat perustuvat aina johonkin vankkaan todistusaineistoon. Ei riitä että on monta heikkoa todistusta vaan pitää olla ainakin yksi vahva todiste. Ja sellaista ei sinulta löydy, jota ei olisi helposti kumottu."

        Tuo on pelkkää denialismia. Sinulle on osoitettu eräs vankimmista tieteellisistä teorioista taustatietoineen. Ainoa keinosi taistella tätä vastaan on keksiä omia sääntöjä, että pitää olla jokin yksi supertodiste, vaikka niin ei ole. Samalla yrität esittää, että ne monet todisteet olisivat jotenkin heikkoja, mutta eiväthän ne sitä ole. Koko toimintasi perustuu juuri siihen lapselliseen epärehellisyyteen, jollaiseksi kreationismia jo luonnehdin. Siksi minä nimitän sinua kreationistiksi juuri tuossa merkityksessä.

        "Olenhan minä lähes kaikki nuo todisteet lukenut läpi. Valtaosa on aihetodisteita. Ensin oletetaan että evoluutio on tosi ja sitten todetaan, että havainnot tukee teoriaa."

        Ei vaan nuo Darwinin jälkeiset havainnot osoittavat, että Darwin oli oikeassa, ja hänen evoluutioteoriansa pitää paikkansa; että siis lajit muuttuvat luonnonvalinnan takia, ja nyt tiedetään mikä aiheuttaa ne piirteet, joihin luonnonvalinta kohdistuu.

        "Esimerkiksi lajien samankaltaisuuteen liittyvät todisteet eivät ole todiste yhdentyyppisestä lajien synnystä vaan ne voidaan selittää monen eri teorian kautta. Ne eivät yksinkertaisesti todista evoluutiota, vaikka sopivat evoluution teoriaan. Käsitätkö?"

        Mutta kun ei ole mitään muita teorioita. Sinä et näköjään ymmärrä mitä teoria tieteessä tarkoittaa. Se, että kaatumaseurasi julistaja kertoo, että amerikkalainen uskonpönkitystä varten muodostettu ajoituspaja on tehnyt "tutkimuksia", ja että heillä on "teoria", ei tarkoita sitä, että olisi olemassa jokin vaihtoehtoinen teoria evoluutiolle.

        "Ja on myös vastatodisteita kuten punktualismi. Lajithan eivät muutu fossiilidatassa teorian mukaisesti"

        Ja tämähän sinulle on myös korjattu, ettei Gouldin jaksoittaisen tasapainon malli tarkoita sitä mitä sinä siitä johdat, ja Gould itse on myös turhautunut siihen, että kreationistit ovat vääristelleet hänen malliaan noin mustavalkoisesti, että lajit eivät muka yhtään muuttuisi ja ilmestyisivät sinne tänne sellaisenaan. Et ole perehtynyt siihen Gouldin juttuun, vaan olet perehtynyt siitä tehtyyn kreationistiseen lainauslouhintaan ja/tai vääristelyyn.

        " Et ole perehtynyt siihen Gouldin juttuun, vaan olet perehtynyt siitä tehtyyn kreationistiseen lainauslouhintaan ja/tai vääristelyyn. "

        Ei ole ensimmäinen kerta. Näitä vääristelyjä on tullut vastaan jo muutama. Vahelita levitellään vaikka alkuperäinen tutkija on monta kertaa yrittänyt oikaista.


      • Tämän_oppinut
        mummomuori kirjoitti:

        " Et ole perehtynyt siihen Gouldin juttuun, vaan olet perehtynyt siitä tehtyyn kreationistiseen lainauslouhintaan ja/tai vääristelyyn. "

        Ei ole ensimmäinen kerta. Näitä vääristelyjä on tullut vastaan jo muutama. Vahelita levitellään vaikka alkuperäinen tutkija on monta kertaa yrittänyt oikaista.

        Eräs uudempi mutta jankutuksen vuoksi jo klassikkomainetta saavuttanut vääristelyhän on se tohtori Mary Schweitzerin T. rex -löytö, josta kreationistien mukaan oikein purskahteli mätää verta, vaikka kyse oli jostain mikroskooppisista jäämistä verisuonten seinämien rakenteessa esiintyvien proteiinien osia. Uskova kristitty paleontologi Schweitzer on ilmaissut närkästyksensä kreationisteja kohtaan, kun he ovat vääristelleet hänen työnsä tuloksia. Tämä juttu on käyty tälläkin palstalla jo varmaan 100 kertaa ja tullaan varmaan käymään 1000 kertaa.

        Jos sattuu tuo Gouldin jaksoittaisen tasapainon malli, jota nimim. evlut101 väärinkäyttää takuuvarmasti joka kerta kun hän luulee selittävänsä siitä jollekin uudelle kommentoijalle, kiinnostamaan, niin kannattaa katsoa oiva perehdytysvideo Tony Reediltä. Hän on tehnyt pitkän sarjan siitä, kuin kreationismi on opettanut hänelle oikeaa tiedettä. Sarjan nimi saattaa hämätä joitain, ja näin ainakin kerran YEV:n suhtautuvan täällä negatiivisesti erääseen sarjan videoon, koska ilmeisesti luuli sitä kreationismin edustukseksi.

        Homman nimi Reedin videoissa on kuitenkin se, että hän selostaa miten kreationistien esittämään evoluutiota tai vanhaa maata koskevaan "ongelmaan" perehtyminen opetti hänelle miten asia oikeasti on, ja sehän ei koskaan ole kuten kreationistit esittävät. Hän on videoissaan varmasti käsitellyt pitkälti suurimman osan kaikista kreationistien asia- ja argumentointivirheistä. Tässä linkki jaksoittaiseen tasapainoon:

        Reed: youtube.com/watch?v=G8mE2EKdojs
        Wiki: fi.wikipedia.org/wiki/Jaksoittaisen_tasapainon_malli

        PS: Nimim. evlut101 jos luet tämän, toivoisin, että viimeinkin ottaisit opiksesi mistä "punktualismissasi" on oikeasti kyse ja lopettaisit sitä koskevien väärinymmärrysten levittelyn. Jos et uskosi velvoitteesta tahdo ymmärtää mistä on kyse, niin sille ei voi kukaan mitään ja siihen sinulla on oikeus, mutta vuoksesi ja luoksesi on ainakin tietoa kannettu.


    • Miksi se olisi loppunut?

      • väärätulkinta

        Eihän se ole edes vielä alkanut!!! Monet eläinlajit ovat pysyneet samanlaisina satoja miljoonia vuosia!


      • väärätulkinta kirjoitti:

        Eihän se ole edes vielä alkanut!!! Monet eläinlajit ovat pysyneet samanlaisina satoja miljoonia vuosia!

        Evoluutio on koko ajan käynnissä.


      • viimeinen.tuomio.2
        mummomuori kirjoitti:

        Evoluutio on koko ajan käynnissä.

        Suoli24:n "kehitystiimi" tekee koko ajan taustalla evoluutiota tehdäkseen meistä parempia kirjoittajia ja ihmisiä :D


      • viimeinen.tuomio.2 kirjoitti:

        Suoli24:n "kehitystiimi" tekee koko ajan taustalla evoluutiota tehdäkseen meistä parempia kirjoittajia ja ihmisiä :D

        :D No voi sentään, onko se sinusta kurjaa, kun kehitetään asioita? Evoluutio on sen verran hidasta, ettei sitä ihan noin vain voi aina havaita. Tosin puhutaanhan sitä kultturin evoluutiostakin.


      • sanokaas

        Sanokaa mistä ja miksi näitä pilkkuviivaviiva-kopioita ilmantuu aina vaan? Miksi haluatte olla pilkkuviivaviivoja?


    • Jeesus_tulee_pian

      Kent Hovind peräsi hänet 'väittelyyn' haastaneelta Aron Nelsonilta edes yhtä todistetta evoluutiouskon tueksi. Nelson antoi kolme: 1. Se, että evoluutioilmiö 'on olemassa', 2. Se, että eläin- ja kasvilajit voidaan luokitella ja 3. Se, että evokeilla on Teoria.

      Siis ei yhden yhtäkään todistetta, silti patsastellaan pönäkkänä ollen olevinaan niin varmoja, vaikka ei esitetä yhtään todistetta, koska todisteita ei ole. Hovindin uusin, edelleen jatkuva puheiden sarja kertoo luomakunnan ihmeistä ja kysyy jokaisen yhteydessä, miten sekin olisi voinut itsestään syntyä ja kehittyä mistä. Eipä ole tullut vastauksia hänen kysymykseensä, mutta noita hyökkäyksiä hänen persoonaansa vastaan kyllä.

      • Jeesus_ei_tule

        Ja kun tullaan harhoistasi takaisin tosiasiain maailmaan niin nähdään, että Aron Ra (ei Nelson, ellet sitten halua, että Hovind nimetään vankinumeronsa perusteella) syö hovindeja aamupalaksi, ja tästä väittelyasiasta on tallennetta kyllä:

        Osa 1) youtube.com/watch?v=du0xy3omB-4
        Osa 2) youtube.com/watch?v=aZaa-HbvaEw

        PS. Hovind on naurettava teeskentelijä "dinotohtori" (LOL) ja Aron sentään on paleontologian opiskellut ja selkeästi tietää mistä puhuu. Hovind on hihhulihuuhaan edustaja eikä mitään muuta.


    • Jos ei ymmärrä evoluution automaattista mekanismia, ei voi ymmärtää, miten kehitys lajin sisällä ja miten lajiutuminen tapahtuu. Hän ei voi myöskään ymmärtää olevansa pihalla, koska evoluutio on tuottanut myös evoluutiota ymmärtämättömälle pöhöttyneen itsetunnon. Se on ollut savannioloissa hyödyllinen ja siten monistunut populaatiossa, mutta tiedollisesti se on väärä.

      Vajaaymmärryksistä, jolla on pöhöttynyt itsetunto, ei voi saada ymmärtämään, että hänen ymmärryksens on vajaa, ja millä tavalla.

      Evoluutiota ymmärtämättömätönta ei voi saada ymmärtämään, vaikka päällään seisoisi.

      • Ehkä se voi olla vaikeaa, ymmärtää näitä elämän monimutkaisia ilmiöitä, mutta en usko että se olisi mahdotonta. Ehkä olen vain toivorikas?


      • evlut101

        "evoluutio on tuottanut myös evoluutiota ymmärtämättömälle pöhöttyneen itsetunnon. "

        Onkin esitetty argumentti sen puolesta, että evoluutioteoria itse asiassa kumoaa itsensä. Nimittäin jos ihminen on sattuman prosessien ja luonnonvalinnan tulosta, niin miten voimme luottaa siihen että ihminen ylipäätään osaa rakentaa loogisia päättelyketjuja. Kuten tuossa itsekin vetoat. Esimerkiksi rotta pärjää mainiosti luonnossa, mutta ei silti ymmärrä maailmankaikkeuden synnystä tuon taivaallista. Sama koskee kärpäsiä jne. Eli jos evoluutioteoria on totta, niin täällä ei ole sellaisia olentoja, jotka voisivat ymmärtää mitään lajien synnystä.

        Jos taas Jumala on luonut ihmisen, niin voimme luottaa siihen, että ihmisessä on pieni osa samaa kykyä ja siksi havaintomme ja järkemme on luotettava.


      • evlut101 kirjoitti:

        "evoluutio on tuottanut myös evoluutiota ymmärtämättömälle pöhöttyneen itsetunnon. "

        Onkin esitetty argumentti sen puolesta, että evoluutioteoria itse asiassa kumoaa itsensä. Nimittäin jos ihminen on sattuman prosessien ja luonnonvalinnan tulosta, niin miten voimme luottaa siihen että ihminen ylipäätään osaa rakentaa loogisia päättelyketjuja. Kuten tuossa itsekin vetoat. Esimerkiksi rotta pärjää mainiosti luonnossa, mutta ei silti ymmärrä maailmankaikkeuden synnystä tuon taivaallista. Sama koskee kärpäsiä jne. Eli jos evoluutioteoria on totta, niin täällä ei ole sellaisia olentoja, jotka voisivat ymmärtää mitään lajien synnystä.

        Jos taas Jumala on luonut ihmisen, niin voimme luottaa siihen, että ihmisessä on pieni osa samaa kykyä ja siksi havaintomme ja järkemme on luotettava.

        Kyky päätellä loogisesti on samalla tavalla kuin muutkin kyvyt eriasteinen eri ihmisillä. Looginen ajattelu on samaa kuin matemaattinen ajattelu. Toiset ymmärtävät enemmän, toiset ovat pysyvästi ymmärtämättä, Ongelma on siinä, että sellaista, joka ei ymmärrä, ei voi saada ymmärtämän, koska hänen looginen ajattelunsa ei riitä.

        Tämä ei tarkoita sitä, että kykenemättömyyttä loogiseen ajatteluun pitäisi hävetä. Moni kyky on paljon tärkeämpi kuin looginen ajattelu.

        <<Eli jos evoluutioteoria on totta, niin täällä ei ole sellaisia olentoja, jotka voisivat ymmärtää mitään lajien synnystä.<<

        Tässä on juuri kyse virhepäättelystä, mutta ei ole mahdollista saada toista ymmärtämään, että päättely on virheellinen. Silloin täytyy viitata vain havaintoihin ja auktoriteettiin. Koko maailmassa ei ole ketään tutkijatason biologia, joka hylkäisi evoluutioteorian.

        <<Jos taas Jumala on luonut ihmisen, niin voimme luottaa siihen, että ihmisessä on pieni osa samaa kykyä ja siksi havaintomme ja järkemme on luotettava.>>

        Ehkä tämä on helpompi ymmärtää virhepäätelmäksi. Koska on ihmisiä, joilta puuttuu kyky tehdä loogisia päätelmiä, se johtaisi siihen, että heidänkin on lupa pitää päätelmiään oikeina.

        x x x

        Taistelu evoluutioteriaa vastaan johtuu teologisista syistä. Heti teorian julkisuuteen tulon jälkeen Raamattuun kirjaimellisesti uskovat ymmärsivät, että jos evoluutio on tapahtunut, ihmiskunnan kantaisää ja -äitiä ei ole ollut olemassa.

        Toiset uskovat taas selittivät kertomuksen symbolisesti, vertauksenomaisesti. Sen kautta tulee esiin koko ihmiskuntaa pysyvästi vaivaava tarve tehdä syntiä. Ja tämän tarpeen voittamiseen saamme apua Jumalalta, joka ei enää pidä synnynnäistä tarvettamme esteenä taivaan valtakuntaan Kristuksen suorittaman sovintotyön ansiosta.

        Kirjaimellisesta tulkinnasta kiinnipitävät ovat joutuneet yhä kauemmaksi tieteellisestä tiedosta, silla tieto on koko ajan kehittynyt ja Raamattu pysähtynyt kirjoitusten kirjoitusajan tasolle.

        Luterilainen kirkko on määrännyt papeillensa pakolliseksi koulutukseksi teologian maisterin tutkinnon. Jokainen yliopisto Suomessa lähtee myös teologian opetuksessa siitä, että evoluutioteoria on tosi. En tunne myöskään ketään pappia, joka tulkitsisi evoluutioteorian vastaisesti Raamattua.

        Meillä, suurella enemmistöllä, jotka pidämme eveluutiota muiden teteellisten löydösten tavalla totena, ei ole pienintäkään syytä epäillä. että kirjaimellisesti tulkitsevat ovat myös kristittyjä. Toivomme, että ajatus olisi molemminpuolinen.


      • evlut101
        Dogmatikos kirjoitti:

        Kyky päätellä loogisesti on samalla tavalla kuin muutkin kyvyt eriasteinen eri ihmisillä. Looginen ajattelu on samaa kuin matemaattinen ajattelu. Toiset ymmärtävät enemmän, toiset ovat pysyvästi ymmärtämättä, Ongelma on siinä, että sellaista, joka ei ymmärrä, ei voi saada ymmärtämän, koska hänen looginen ajattelunsa ei riitä.

        Tämä ei tarkoita sitä, että kykenemättömyyttä loogiseen ajatteluun pitäisi hävetä. Moni kyky on paljon tärkeämpi kuin looginen ajattelu.

        <<Eli jos evoluutioteoria on totta, niin täällä ei ole sellaisia olentoja, jotka voisivat ymmärtää mitään lajien synnystä.<<

        Tässä on juuri kyse virhepäättelystä, mutta ei ole mahdollista saada toista ymmärtämään, että päättely on virheellinen. Silloin täytyy viitata vain havaintoihin ja auktoriteettiin. Koko maailmassa ei ole ketään tutkijatason biologia, joka hylkäisi evoluutioteorian.

        <<Jos taas Jumala on luonut ihmisen, niin voimme luottaa siihen, että ihmisessä on pieni osa samaa kykyä ja siksi havaintomme ja järkemme on luotettava.>>

        Ehkä tämä on helpompi ymmärtää virhepäätelmäksi. Koska on ihmisiä, joilta puuttuu kyky tehdä loogisia päätelmiä, se johtaisi siihen, että heidänkin on lupa pitää päätelmiään oikeina.

        x x x

        Taistelu evoluutioteriaa vastaan johtuu teologisista syistä. Heti teorian julkisuuteen tulon jälkeen Raamattuun kirjaimellisesti uskovat ymmärsivät, että jos evoluutio on tapahtunut, ihmiskunnan kantaisää ja -äitiä ei ole ollut olemassa.

        Toiset uskovat taas selittivät kertomuksen symbolisesti, vertauksenomaisesti. Sen kautta tulee esiin koko ihmiskuntaa pysyvästi vaivaava tarve tehdä syntiä. Ja tämän tarpeen voittamiseen saamme apua Jumalalta, joka ei enää pidä synnynnäistä tarvettamme esteenä taivaan valtakuntaan Kristuksen suorittaman sovintotyön ansiosta.

        Kirjaimellisesta tulkinnasta kiinnipitävät ovat joutuneet yhä kauemmaksi tieteellisestä tiedosta, silla tieto on koko ajan kehittynyt ja Raamattu pysähtynyt kirjoitusten kirjoitusajan tasolle.

        Luterilainen kirkko on määrännyt papeillensa pakolliseksi koulutukseksi teologian maisterin tutkinnon. Jokainen yliopisto Suomessa lähtee myös teologian opetuksessa siitä, että evoluutioteoria on tosi. En tunne myöskään ketään pappia, joka tulkitsisi evoluutioteorian vastaisesti Raamattua.

        Meillä, suurella enemmistöllä, jotka pidämme eveluutiota muiden teteellisten löydösten tavalla totena, ei ole pienintäkään syytä epäillä. että kirjaimellisesti tulkitsevat ovat myös kristittyjä. Toivomme, että ajatus olisi molemminpuolinen.

        " Kyky päätellä loogisesti on samalla tavalla kuin muutkin kyvyt eriasteinen eri ihmisillä. Looginen ajattelu on samaa kuin matemaattinen ajattelu. Toiset ymmärtävät enemmän, toiset ovat pysyvästi ymmärtämättä, Ongelma on siinä, että sellaista, joka ei ymmärrä, ei voi saada ymmärtämän, koska hänen looginen ajattelunsa ei riitä."

        Ja tällä uskomuksella vahvistat käsitystäsi siitä, että sinä kuulut siihen älykkäiden ryhmään ja kaikki uskovat kuuluvat siihen tyhmien ryhmään. Eri mieltä minun kanssa olevat ovat tyhmiä. MIten klassinen argumentti.

        "Tässä on juuri kyse virhepäättelystä, mutta ei ole mahdollista saada toista ymmärtämään, että päättely on virheellinen. Silloin täytyy viitata vain havaintoihin ja auktoriteettiin. Koko maailmassa ei ole ketään tutkijatason biologia, joka hylkäisi evoluutioteorian."

        Jos ihminen on sattumalta kehittynyt tällaiseksi, niin et voi kuitenkaan luottaa siihen, että sen kyky tehdä loogisia päättelyketjuja olisi luotettava. Ja jos ei voida luottaa, niin silloin koko teorialta putoaa pohja. Voi tietysti olla että olemme sattumalta oikeassa, mutta sellainen perusta on todella hutera.

        Ja biologeista puheenollen.... Evoluutioteoria ei ole pelkästään biologinen kysymys. Siihen liittyy lukuisia tieteenaloja. Biologeilla ei esimerkiksi ole mitään käsitystä ohjelmoinnista, joten he sivuttavat DNA datan itsestäänsyntymisen ongelman olankohautuksella, koska eivät sitä ymmärrä. Matemaatikot ja ohjelmoijat ymmärtävät tämän ongelman biologeja paremmin. Samoin biologit eivät ymmärrä mitään todennäköisyyslaskennasta ja siksi heille on luontaista uskoa, että materia voi järjestyä supermonimutkaisiin rakenteisiin itsestään. Ja nyt kun biologeja nostat, niin kannattaa muistaa että Darwin itse oli teologi, joka tosin keskeytti opintonsa.

        "Ehkä tämä on helpompi ymmärtää virhepäätelmäksi. Koska on ihmisiä, joilta puuttuu kyky tehdä loogisia päätelmiä, se johtaisi siihen, että heidänkin on lupa pitää päätelmiään oikeina."

        Ihmisten välillä ei kuitenkaan todellisuudessa ole niin valtavia eroja. Maailman syntyteorioita koskevat erimielisyydet on pikemminkin kulttuurisia eikä älyllisiä. Tieteellisesti yksikäsitteistä vastausta niihin ei löydy.

        "Taistelu evoluutioteriaa vastaan johtuu teologisista syistä. Heti teorian julkisuuteen tulon jälkeen Raamattuun kirjaimellisesti uskovat ymmärsivät, että jos evoluutio on tapahtunut, ihmiskunnan kantaisää ja -äitiä ei ole ollut olemassa."

        Evoluutioteoriaa vastustettii tiedepiireissä kiivaasti ja syyt olivat tieteelliset. Darwin ei pystynyt selittämään evoluution mekanismia (mitä tapahtuu solutasolla). Evoluutioteoriaa ennen oli aikaisempi Lamarckin teoria, joka erosi Darwinin teoriasta siten, että Lamrckin mukaan esim. kirahvin kaula pitenee kun se kurottelee korkealla olevia lehtiä ja sen poikaset perivät ominaisuuden. Lamarckin teoria kärsi uskottavuusongelmasta, koska esimerkiksi sepän paksu käsivarsi ei periytynyt sen lapselle.

        Mutta myös evoluutioteoria on muuttunut Darwinin ajasta aivan täysin. Kaunis evoluutiopuu ei sopinut fossiilidataan vaan lajit pysyvät muuttumattomina fossiilidatassa vuosimiljoonia ja ilmestyvät fossiilidataan yhtäkkiä. Vain teorian nimi on säilytetty....

        Mutta siinä missä kristitty vastustaa evoluutioteorian elämänsyntymallia, niin yhtä lailla ateisti kokee sen itselleen pyhäksi opinkappaleeksi, joka tukee hänen uskoaan ja maailmankuvaansa. Asia toimii siis molempiin suuntiin.

        "Toiset uskovat taas selittivät kertomuksen symbolisesti, vertauksenomaisesti. Sen kautta tulee esiin koko ihmiskuntaa pysyvästi vaivaava tarve tehdä syntiä. Ja tämän tarpeen voittamiseen saamme apua Jumalalta, joka ei enää pidä synnynnäistä tarvettamme esteenä taivaan valtakuntaan Kristuksen suorittaman sovintotyön ansiosta."

        Perisyntiopissa on silti jotain viisautta. Marx ajatteli että ihminen on pohjimmiltaan hyvä. Hän oli väärässä. Ihmiset tekee pahoja asioita ja myös hyvät ihmiset. Sikäli perisyntioppi edelleen kuvaa ihmistä paremmin kuin mikään muu teoria. Siis että jokaisen ihmisen sisällä on kamppailua pahuuden kanssa. On myös niitä, jotka ovat nuoruutensa olleet häikäilemättömiä rikollisia, mutta joku on saanut heidät muuttumaan täysin ihmisenä. Evoluutioteoria ei pysty antamaan tälle käyttäytymiselle mitään kuvausta.


      • evlut101
        evlut101 kirjoitti:

        " Kyky päätellä loogisesti on samalla tavalla kuin muutkin kyvyt eriasteinen eri ihmisillä. Looginen ajattelu on samaa kuin matemaattinen ajattelu. Toiset ymmärtävät enemmän, toiset ovat pysyvästi ymmärtämättä, Ongelma on siinä, että sellaista, joka ei ymmärrä, ei voi saada ymmärtämän, koska hänen looginen ajattelunsa ei riitä."

        Ja tällä uskomuksella vahvistat käsitystäsi siitä, että sinä kuulut siihen älykkäiden ryhmään ja kaikki uskovat kuuluvat siihen tyhmien ryhmään. Eri mieltä minun kanssa olevat ovat tyhmiä. MIten klassinen argumentti.

        "Tässä on juuri kyse virhepäättelystä, mutta ei ole mahdollista saada toista ymmärtämään, että päättely on virheellinen. Silloin täytyy viitata vain havaintoihin ja auktoriteettiin. Koko maailmassa ei ole ketään tutkijatason biologia, joka hylkäisi evoluutioteorian."

        Jos ihminen on sattumalta kehittynyt tällaiseksi, niin et voi kuitenkaan luottaa siihen, että sen kyky tehdä loogisia päättelyketjuja olisi luotettava. Ja jos ei voida luottaa, niin silloin koko teorialta putoaa pohja. Voi tietysti olla että olemme sattumalta oikeassa, mutta sellainen perusta on todella hutera.

        Ja biologeista puheenollen.... Evoluutioteoria ei ole pelkästään biologinen kysymys. Siihen liittyy lukuisia tieteenaloja. Biologeilla ei esimerkiksi ole mitään käsitystä ohjelmoinnista, joten he sivuttavat DNA datan itsestäänsyntymisen ongelman olankohautuksella, koska eivät sitä ymmärrä. Matemaatikot ja ohjelmoijat ymmärtävät tämän ongelman biologeja paremmin. Samoin biologit eivät ymmärrä mitään todennäköisyyslaskennasta ja siksi heille on luontaista uskoa, että materia voi järjestyä supermonimutkaisiin rakenteisiin itsestään. Ja nyt kun biologeja nostat, niin kannattaa muistaa että Darwin itse oli teologi, joka tosin keskeytti opintonsa.

        "Ehkä tämä on helpompi ymmärtää virhepäätelmäksi. Koska on ihmisiä, joilta puuttuu kyky tehdä loogisia päätelmiä, se johtaisi siihen, että heidänkin on lupa pitää päätelmiään oikeina."

        Ihmisten välillä ei kuitenkaan todellisuudessa ole niin valtavia eroja. Maailman syntyteorioita koskevat erimielisyydet on pikemminkin kulttuurisia eikä älyllisiä. Tieteellisesti yksikäsitteistä vastausta niihin ei löydy.

        "Taistelu evoluutioteriaa vastaan johtuu teologisista syistä. Heti teorian julkisuuteen tulon jälkeen Raamattuun kirjaimellisesti uskovat ymmärsivät, että jos evoluutio on tapahtunut, ihmiskunnan kantaisää ja -äitiä ei ole ollut olemassa."

        Evoluutioteoriaa vastustettii tiedepiireissä kiivaasti ja syyt olivat tieteelliset. Darwin ei pystynyt selittämään evoluution mekanismia (mitä tapahtuu solutasolla). Evoluutioteoriaa ennen oli aikaisempi Lamarckin teoria, joka erosi Darwinin teoriasta siten, että Lamrckin mukaan esim. kirahvin kaula pitenee kun se kurottelee korkealla olevia lehtiä ja sen poikaset perivät ominaisuuden. Lamarckin teoria kärsi uskottavuusongelmasta, koska esimerkiksi sepän paksu käsivarsi ei periytynyt sen lapselle.

        Mutta myös evoluutioteoria on muuttunut Darwinin ajasta aivan täysin. Kaunis evoluutiopuu ei sopinut fossiilidataan vaan lajit pysyvät muuttumattomina fossiilidatassa vuosimiljoonia ja ilmestyvät fossiilidataan yhtäkkiä. Vain teorian nimi on säilytetty....

        Mutta siinä missä kristitty vastustaa evoluutioteorian elämänsyntymallia, niin yhtä lailla ateisti kokee sen itselleen pyhäksi opinkappaleeksi, joka tukee hänen uskoaan ja maailmankuvaansa. Asia toimii siis molempiin suuntiin.

        "Toiset uskovat taas selittivät kertomuksen symbolisesti, vertauksenomaisesti. Sen kautta tulee esiin koko ihmiskuntaa pysyvästi vaivaava tarve tehdä syntiä. Ja tämän tarpeen voittamiseen saamme apua Jumalalta, joka ei enää pidä synnynnäistä tarvettamme esteenä taivaan valtakuntaan Kristuksen suorittaman sovintotyön ansiosta."

        Perisyntiopissa on silti jotain viisautta. Marx ajatteli että ihminen on pohjimmiltaan hyvä. Hän oli väärässä. Ihmiset tekee pahoja asioita ja myös hyvät ihmiset. Sikäli perisyntioppi edelleen kuvaa ihmistä paremmin kuin mikään muu teoria. Siis että jokaisen ihmisen sisällä on kamppailua pahuuden kanssa. On myös niitä, jotka ovat nuoruutensa olleet häikäilemättömiä rikollisia, mutta joku on saanut heidät muuttumaan täysin ihmisenä. Evoluutioteoria ei pysty antamaan tälle käyttäytymiselle mitään kuvausta.

        "Kirjaimellisesta tulkinnasta kiinnipitävät ovat joutuneet yhä kauemmaksi tieteellisestä tiedosta, silla tieto on koko ajan kehittynyt ja Raamattu pysähtynyt kirjoitusten kirjoitusajan tasolle."

        Olen sitä mieltä, että tiede on itse asiassa viime vuosikymmeninä mullistanut asian päälaelleen. 1900-luvun alussa ei ymmärretty informaatiosta yhtään mitään. Silloin oli luontaista ymmärtää, että materia nyt vaan jotenkin järjestyy. Nykyään jokainen ymmärtää, että informaation syntyminen itsestään kohinasta on täysin järjetön ajatus. Toisin sanoen evoluutioprosessin alkumetrit alkumeressä on niin noloja teorioita, ettei niistä tosissaan kehtaa yksikään professori pitää puhetta.

        "Luterilainen kirkko on määrännyt papeillensa pakolliseksi koulutukseksi teologian maisterin tutkinnon. Jokainen yliopisto Suomessa lähtee myös teologian opetuksessa siitä, että evoluutioteoria on tosi. En tunne myöskään ketään pappia, joka tulkitsisi evoluutioteorian vastaisesti Raamattua."

        Pappien ydinaluetta on silti teologia eri muodoissaan. Itselläni on luonnontiedetausta ja en koe että pappi olisi se henkilö, joka keskimäärin hallitsisi ne asiat minua paremmin.

        "Meillä, suurella enemmistöllä, jotka pidämme eveluutiota muiden teteellisten löydösten tavalla totena, ei ole pienintäkään syytä epäillä. että kirjaimellisesti tulkitsevat ovat myös kristittyjä. Toivomme, että ajatus olisi molemminpuolinen."

        Totta kai on... Evoluutioteoria ei ole kristinuskon ydinasia, mutta siitä on kiinnostava käydä keskusteluja.


      • Dogmatikos kirjoitti:

        Kyky päätellä loogisesti on samalla tavalla kuin muutkin kyvyt eriasteinen eri ihmisillä. Looginen ajattelu on samaa kuin matemaattinen ajattelu. Toiset ymmärtävät enemmän, toiset ovat pysyvästi ymmärtämättä, Ongelma on siinä, että sellaista, joka ei ymmärrä, ei voi saada ymmärtämän, koska hänen looginen ajattelunsa ei riitä.

        Tämä ei tarkoita sitä, että kykenemättömyyttä loogiseen ajatteluun pitäisi hävetä. Moni kyky on paljon tärkeämpi kuin looginen ajattelu.

        <<Eli jos evoluutioteoria on totta, niin täällä ei ole sellaisia olentoja, jotka voisivat ymmärtää mitään lajien synnystä.<<

        Tässä on juuri kyse virhepäättelystä, mutta ei ole mahdollista saada toista ymmärtämään, että päättely on virheellinen. Silloin täytyy viitata vain havaintoihin ja auktoriteettiin. Koko maailmassa ei ole ketään tutkijatason biologia, joka hylkäisi evoluutioteorian.

        <<Jos taas Jumala on luonut ihmisen, niin voimme luottaa siihen, että ihmisessä on pieni osa samaa kykyä ja siksi havaintomme ja järkemme on luotettava.>>

        Ehkä tämä on helpompi ymmärtää virhepäätelmäksi. Koska on ihmisiä, joilta puuttuu kyky tehdä loogisia päätelmiä, se johtaisi siihen, että heidänkin on lupa pitää päätelmiään oikeina.

        x x x

        Taistelu evoluutioteriaa vastaan johtuu teologisista syistä. Heti teorian julkisuuteen tulon jälkeen Raamattuun kirjaimellisesti uskovat ymmärsivät, että jos evoluutio on tapahtunut, ihmiskunnan kantaisää ja -äitiä ei ole ollut olemassa.

        Toiset uskovat taas selittivät kertomuksen symbolisesti, vertauksenomaisesti. Sen kautta tulee esiin koko ihmiskuntaa pysyvästi vaivaava tarve tehdä syntiä. Ja tämän tarpeen voittamiseen saamme apua Jumalalta, joka ei enää pidä synnynnäistä tarvettamme esteenä taivaan valtakuntaan Kristuksen suorittaman sovintotyön ansiosta.

        Kirjaimellisesta tulkinnasta kiinnipitävät ovat joutuneet yhä kauemmaksi tieteellisestä tiedosta, silla tieto on koko ajan kehittynyt ja Raamattu pysähtynyt kirjoitusten kirjoitusajan tasolle.

        Luterilainen kirkko on määrännyt papeillensa pakolliseksi koulutukseksi teologian maisterin tutkinnon. Jokainen yliopisto Suomessa lähtee myös teologian opetuksessa siitä, että evoluutioteoria on tosi. En tunne myöskään ketään pappia, joka tulkitsisi evoluutioteorian vastaisesti Raamattua.

        Meillä, suurella enemmistöllä, jotka pidämme eveluutiota muiden teteellisten löydösten tavalla totena, ei ole pienintäkään syytä epäillä. että kirjaimellisesti tulkitsevat ovat myös kristittyjä. Toivomme, että ajatus olisi molemminpuolinen.

        ”Kirjaimellisesta tulkinnasta kiinnipitävät ovat joutuneet yhä kauemmaksi tieteellisestä tiedosta, silla tieto on koko ajan kehittynyt ja Raamattu pysähtynyt kirjoitusten kirjoitusajan tasolle.”

        Löysin tuossa ohimennen kiinnostavan tiedon:

        ”Esimerkiksi Martti Lutherin (1483–1546) mukaan uskonasioita ei pidä ymmärtää kirjaimellisesti samalla tavalla kuin arkikielen asiat ymmärretään. Luther opettaa, että jumalauskoa koskevissa lauseissa luonnollisen kielen sanat saavat uuden merkityksen.”
        ”On jumalanpilkkaa uskoa uskontunnustuksen osiin kirjaimellisesti.”

        ”…voi sanoa, että omaksuessaan 1700-luvulta peräisin olevan skotlantilaisen empirismin uskonasioiden tulkintamalliksi fundamentalistit hylkäsivät klassisen kristinuskon.”
        K&K ”Raamatulla päähän” 20.09.2010 Juhani Huttunen

        Totta on, että kirjaimellisen tulkinnan kannattajat joutuvat yhä uudestaan ja uudestaan ristiriitaan todellisuuden suhteen. Siksi syntyy yhä uusia ja entistä monimutkaisempia selitysmalleja.

        ”Meillä, suurella enemmistöllä, jotka pidämme eveluutiota muiden teteellisten löydösten tavalla totena, ei ole pienintäkään syytä epäillä. että kirjaimellisesti tulkitsevat ovat myös kristittyjä.”

        Totta. Vaikka he ovat luoneet uudenlaisen oppijärjestelmän, ei ole mitään syytä väheksyä kristillisyyttä heidän kohdallaan.


    • Edelleenjaedelleen

      Kehitys kehittyy!

      • Elämä menee eteenpäin, eikä se koskaan voi kääntyä takaisin. Se vain pitäisi oppia hyväksymään.


    • onko.loppu

      Ketju onkin venähtänyt pitkäksi ja melko asiallisena pysynyt, ateisteilla tosin se tyypillinen ad hominem uskovista ja älystään nosti päätään taaskin. Minulla oli ajatukseni aloituksen laittaessani, mikä oli kysymykseni taustalla. Nyt kuitenkin olen harmittavasti unohtanut sen ajatuksen. Mummomuorin kanssa päiväkausia vienyt vänkääminen sekoitti niin, että ajatus katosi. Jos se palautuu, niin palaan vielä asiaan joskus.

      Väite, että Jumala olisi käyttänyt evoluutiota luomiseen, on itsessään mahdoton ja täysin ristiriitainen. Jumala ei voinut käyttää kuolemaa luomisensa välineenä, koska kuolema tuli maailmaan ihmisen syntiinlankeemuksen jälkeen, se oli rangaistus tottelemattomuuden synnistä. Raamatun ilmoituksen mukaan kuolema tuhotaan vihollisista viimeisimpänä. Se on vihollinen kaikelle elämälle.

      Ajatus siitä, että maailmankaikkeuden Luoja olisi käyttänyt julmaa kuoleman ja heikkojen eliminoimisen kehitysprosessia saadakseen itsensä kaltaisen olennon, on täysin Jumalan itsestään Raamatussa ilmoittaman olemuksen vastainen. Jos kuolema ja kärsimys muokkasi miljoonia vuosia eliökuntaa ennen Aatamin ja Eevan syntymää, miten tulokset olisi olleet erittäin hyviä, kuten Jumala aikaansaannoksiaan kutsui.

      Kirkkoisät uskoivat nuoreen maahan.
      https://luominen.fi/kirkkoisat-uskoivat-nuoreen-maahan

      Vielä fundamentalismin alkuperäinen merkitys, kun sitä niin helposti käytetään holtittomasta ja pelkkänä haukkumasanana:
      Historiallisesti fundamentalismilla on kuvattu henkilöä, joka omaksuu viisi presbyteerisen kirkon vuonna 1910 järjestetyn kokouksen perustavanlaatuista näkemystä. Nämä viisi näkemystä olivat: Kristuksen ihmeet, Kristuksen neitseellinen syntymä, Kristuksen sijaiskärsivyys, Kristuksen ruumiillinen ylösnousemus ja Kirjoitusten jumalallinen innoitus.

      Tätä fundamentteihin eli perusteisiin menevää olen painottanut. Nykyisin siihen on sekoitettu kaikkien uskontojen fundamentteja ja siten saanut huonon kaiun, kun ihmiset yhdistää islamin fundamentalistit ja kristinuskon fundamentalistit.

      • "Jumala ei voinut käyttää kuolemaa luomisensa välineenä, koska kuolema tuli maailmaan ihmisen syntiinlankeemuksen jälkeen, se oli rangaistus tottelemattomuuden synnistä. Raamatun ilmoituksen mukaan kuolema tuhotaan vihollisista viimeisimpänä. Se on vihollinen kaikelle elämälle."
        Kuolemaa on ollut yhtä kauan kuin elämääkin, kuolema on itse asiassa elämän edellytys, ei meitäkään olisi jos mikään ennen meitä syntynyt elämänmuoto olisi ollut kuolematon mutta kuntenkin lisääntynyt. Emme yksinkertaisesti mahtuisi tällä pallolle, alussa syntynyt elämä olisi sen täyttänyt.Miten sekään selviäsi loputtomiin, lisääntyminen jatkuisi mutta tilaa ei tulisi lisää kuoleman kautta?

        Kuolema on solutason tapahtuma ei teologinen, jos uskotaan että on Jumala joka on luonut kaiken muun, elämän ja olevan, miksi kuolema voisi olla luojansa vihollinen? Myöskin vaikka uskoisi että on tapahtunutt jokin ihmisen aiheuttama "lankeemus" niin siitähän nyt kärsivät myös viattomat eläimet ja sen jumala joka tapauksessa sallii .Ongelma on silloin se miten selittää hyväksi uskottu jumala ja maailma joka on täynnä kärsimystä jota seuraa kaikkivaltias olento joka voisi sen halutessaan poistaa .

        Nyt kuitenkin jo tiedämme varmuudella että lajimme on myöhäinen tulokas tällä maapallolla ja että elämää on ollut kauan ennen lajiamme, joten emme voi selittää mitään niin että pidämme kirjaimellisesti totena yhtäkään raamatun luomiskertomuksista.Ei niitä raamatussakaan olisi useita jos kirjoittajat itse olisivat uskoneet vain yhteen .


      • ”Jumala ei voinut käyttää kuolemaa luomisensa välineenä, koska kuolema tuli maailmaan ihmisen syntiinlankeemuksen jälkeen…”

        Lähdet aika ihmiskeskeisesti asiaa lähestymään. Kuten ”dikduk ” 24.11.2018 12:21 kirjoittaa ”…kuolema on itse asiassa elämän edellytys…”. Jokainen, joka vähänkään on perehtynyt luonnon kiertokulkuun tietää tämän.

        Sillä on siis vankka merkitys, että palaamme maaksi jälleen. Osaksi tuota suurenmoista luonnonkiertokulkua. Ihminen lisääntyy, kasvaa, vanhenee ja lopulta kuolee. Niin tekee kaikki elollinen.

        En ymmärrä, miksi pidät sitä rangaistuksena? Ehkä sinulta puuttuu nöyryys kaiken tämän edessä?

        ”Vielä fundamentalismin alkuperäinen merkitys…”
        ”Historiallisesti fundamentalismilla on kuvattu henkilöä, joka omaksuu viisi presbyteerisen kirkon vuonna 1910 järjestetyn kokouksen perustavanlaatuista näkemystä.”

        Niin, Yhdysvalloissa oli protestantteja, jotka alkoivat vastustaa Darwinin käsityksiä sekä Raamatun historiallista tutkimuksia. He kirjoittivat yhdessä ”The Fundamentals” 12 osaisen kirjasarjan 1915.

        Tästä kirjasarjasta otti baptisti Curtis Lee Laws ”fundamentalisti” käyttöön, joka yleistyi. Hän halusi tuon uuden termin siksi, että konservatiivi kuulosti aivan liian taantumukselliselta.

        Kuten kaikki ilmiöt, niin tämäkin jatkoi kehitystään. Jo alkuaikoina liike alkoi toimia siten, että se otti osaa yhteiskunnalliseen politiikkaan sekä päätöksen tekoon. Toiminta kulminoitui tähän: ”…Daytonin kaupungissa Tennesseen osavaltiossa vuonna 1925 käytyyn niin kutsuttuun apinaoikeudenkäyntiin. Siinä syytettynä oli opettaja, joka vastoin osavaltion lakeja oli opettanut koulussa evoluutioteoriaa.”

        ” Juttu oli täysin järjestetty: Kyseinen opettaja, John Scopes (1900–1970), oli todellisuudessa jalkapallovalmentaja ja liikunnanopettaja, joka heti biologian opettajan sijaiseksi jouduttuaan oli tahallaan opettanut evoluutiota, jotta juttu päätyisi oikeuteen. Fundamentalistit voittivat oikeudenkäynnin…”

        Mutta tuota seurannut mediasirkus teki fundamentalistit naurunalaiseksi. Tämä liike hiljeni pitkäksi aikaa. Sitten se sai uutta energiaa ja jälleen nimeä muutettiin; ”Sanakirjoihin fundamentalismi otettiin vasta toisen maailmansodan jälkeen, jolloin fundamentalistit jo alkoivat käyttää itsestään ilmaisua evangelical (suomessa evankelikaalinen), koska fundamentalistinen liike oli joutunut julkisesti naurunalaiseksi.”
        K&K ”Raamatulla päähän” 20.09.2010 Juhani Huttunen

        ”Tätä fundamentteihin eli perusteisiin…”

        Koska jokaisessa uskonnossa on oma liikkeensä, joka tosiaan painottaa omia perusteitaan, ja heidän asenteensa sekä toimintamallinsa on hyvin samankaltainen, alettiin käyttämään heistäkin ilmaisua ”fundamentalisti”.

        ”…suurin osa fundamentalismia käsitelleistä uskontotieteilijöistä on sitä mieltä, että käsite viittaa selkeästi rajattavissa olevaan ilmiökokonaisuuteen, joka voidaan tunnistaa monista eri uskontotraditioista. Käsite on myös mahdollista määritellä varsin neutraalisti ja deskriptiivisesti siten, että se tavoittaa joukon reaalimaailman ilmiöitä, joilla on yhteisiä piirteitä ja joille voidaan etsiä erilaisia selitysmalleja.”
        Ketola ”Mitä on fundamentalismi”

        Joten, jos haluat käyttää tuota omaperäistä määritelmää, se on hyvä ilmaista siksi, ettet joudu jatkuvasti väärinymmärretyksi. Koska yleensä termiä käytetään laajemmassa tarkoituksessa, ja toki voi eritellä ilmaisemalla vaikka evankelikaaliset – jolloin tiedetään, keitä fundamentalisteja erityisesti tarkoitetaan.


      • onko.loppu
        dikduk kirjoitti:

        "Jumala ei voinut käyttää kuolemaa luomisensa välineenä, koska kuolema tuli maailmaan ihmisen syntiinlankeemuksen jälkeen, se oli rangaistus tottelemattomuuden synnistä. Raamatun ilmoituksen mukaan kuolema tuhotaan vihollisista viimeisimpänä. Se on vihollinen kaikelle elämälle."
        Kuolemaa on ollut yhtä kauan kuin elämääkin, kuolema on itse asiassa elämän edellytys, ei meitäkään olisi jos mikään ennen meitä syntynyt elämänmuoto olisi ollut kuolematon mutta kuntenkin lisääntynyt. Emme yksinkertaisesti mahtuisi tällä pallolle, alussa syntynyt elämä olisi sen täyttänyt.Miten sekään selviäsi loputtomiin, lisääntyminen jatkuisi mutta tilaa ei tulisi lisää kuoleman kautta?

        Kuolema on solutason tapahtuma ei teologinen, jos uskotaan että on Jumala joka on luonut kaiken muun, elämän ja olevan, miksi kuolema voisi olla luojansa vihollinen? Myöskin vaikka uskoisi että on tapahtunutt jokin ihmisen aiheuttama "lankeemus" niin siitähän nyt kärsivät myös viattomat eläimet ja sen jumala joka tapauksessa sallii .Ongelma on silloin se miten selittää hyväksi uskottu jumala ja maailma joka on täynnä kärsimystä jota seuraa kaikkivaltias olento joka voisi sen halutessaan poistaa .

        Nyt kuitenkin jo tiedämme varmuudella että lajimme on myöhäinen tulokas tällä maapallolla ja että elämää on ollut kauan ennen lajiamme, joten emme voi selittää mitään niin että pidämme kirjaimellisesti totena yhtäkään raamatun luomiskertomuksista.Ei niitä raamatussakaan olisi useita jos kirjoittajat itse olisivat uskoneet vain yhteen .

        Toteamuksellasi "Kuolemaa on ollut yhtä kauan kuin elämääkin" kerrot sen, minkä Raamattu ilmoittaa. Kuolema tuli synnin seurauksena. Ihminen luotiin kuolemattomaksi, mutta synnin vuoksi piti fyysinen elämä katkaista kuolemalla, muutenhan synti olisi jatkunut ikuisuuksia. Emme tiedä, miten elämä olisi jatkunut ilman syntiinlankeemusta, mutta kyllä ihmisen tekijä tiesi mitä teki. Tiesi senkin, että ihminen lankeaa ja siihen oli ratkaisu valmiina: Jeesus, joka oli jo luomisessa mukana. Meidän äly ei vain yksinkertaisesti riitä käsittämään kaikkea eikä varsinkaan Jumalan älyä.

        Koko luotu maapallo kärsii ihmisen lankeemuksesta. Jumalahan ne solut on luonut ja määrännyt ne heikkenemään tietyn ajanjakson jälkeen kun lyhensi ihmisen elinikää. Sehän taas pitenee tuhatvuotisessa valtakunnassa kun “Ei siellä ole enää lasta, joka eläisi vain muutaman päivän, ei vanhusta, joka ei täyttäisi päiviensä määrää; sillä nuorin kuolee satavuotiaana, ja vasta satavuotiaana synnintekijä joutuu kiroukseen.” Jes. 65:20. Jumala poistaa kärsimyksen kerran, suunnitelmansa mene koko ajan eteenpäin, nyt elämme vahvaa antikristukselle sijaa tekevää maailman yhdentymisen aikaa. Profetiat toteutuvat silmiemme edessä.

        “Sentähden, niinkuin yhden ihmisen kautta synti tuli maailmaan, ja synnin kautta kuolema, niin kuolema on tullut kaikkien ihmisten osaksi, koska kaikki ovat syntiä tehneet.” Room. 5:12

        Tiedämme varmuudella? Kuinka voi tietää varmuudella, kun ei ole mitään varmuutta? Ei ole havaintoja miljoonien vuosien kehityksestä. Kun sanotaan, ettei kehitystä voi nähdä sen hitauden vuoksi, niin miten ihmeessä voidaan nähdä todisteita siitä kehityksestä? Pelkkää tarinankerrontaa eikä tieteellistä todistusta. Erilaiset olennot ei todista, että ne kaikki polveutuisi yksinkertaisemmasta kantamuodosta.

        Ei ole kahta eri luomiskertomusta vaan yksi ja sitä seuraa tarkempi kuvaus ihmisen luomisesta.


      • onko.loppu
        mummomuori kirjoitti:

        ”Jumala ei voinut käyttää kuolemaa luomisensa välineenä, koska kuolema tuli maailmaan ihmisen syntiinlankeemuksen jälkeen…”

        Lähdet aika ihmiskeskeisesti asiaa lähestymään. Kuten ”dikduk ” 24.11.2018 12:21 kirjoittaa ”…kuolema on itse asiassa elämän edellytys…”. Jokainen, joka vähänkään on perehtynyt luonnon kiertokulkuun tietää tämän.

        Sillä on siis vankka merkitys, että palaamme maaksi jälleen. Osaksi tuota suurenmoista luonnonkiertokulkua. Ihminen lisääntyy, kasvaa, vanhenee ja lopulta kuolee. Niin tekee kaikki elollinen.

        En ymmärrä, miksi pidät sitä rangaistuksena? Ehkä sinulta puuttuu nöyryys kaiken tämän edessä?

        ”Vielä fundamentalismin alkuperäinen merkitys…”
        ”Historiallisesti fundamentalismilla on kuvattu henkilöä, joka omaksuu viisi presbyteerisen kirkon vuonna 1910 järjestetyn kokouksen perustavanlaatuista näkemystä.”

        Niin, Yhdysvalloissa oli protestantteja, jotka alkoivat vastustaa Darwinin käsityksiä sekä Raamatun historiallista tutkimuksia. He kirjoittivat yhdessä ”The Fundamentals” 12 osaisen kirjasarjan 1915.

        Tästä kirjasarjasta otti baptisti Curtis Lee Laws ”fundamentalisti” käyttöön, joka yleistyi. Hän halusi tuon uuden termin siksi, että konservatiivi kuulosti aivan liian taantumukselliselta.

        Kuten kaikki ilmiöt, niin tämäkin jatkoi kehitystään. Jo alkuaikoina liike alkoi toimia siten, että se otti osaa yhteiskunnalliseen politiikkaan sekä päätöksen tekoon. Toiminta kulminoitui tähän: ”…Daytonin kaupungissa Tennesseen osavaltiossa vuonna 1925 käytyyn niin kutsuttuun apinaoikeudenkäyntiin. Siinä syytettynä oli opettaja, joka vastoin osavaltion lakeja oli opettanut koulussa evoluutioteoriaa.”

        ” Juttu oli täysin järjestetty: Kyseinen opettaja, John Scopes (1900–1970), oli todellisuudessa jalkapallovalmentaja ja liikunnanopettaja, joka heti biologian opettajan sijaiseksi jouduttuaan oli tahallaan opettanut evoluutiota, jotta juttu päätyisi oikeuteen. Fundamentalistit voittivat oikeudenkäynnin…”

        Mutta tuota seurannut mediasirkus teki fundamentalistit naurunalaiseksi. Tämä liike hiljeni pitkäksi aikaa. Sitten se sai uutta energiaa ja jälleen nimeä muutettiin; ”Sanakirjoihin fundamentalismi otettiin vasta toisen maailmansodan jälkeen, jolloin fundamentalistit jo alkoivat käyttää itsestään ilmaisua evangelical (suomessa evankelikaalinen), koska fundamentalistinen liike oli joutunut julkisesti naurunalaiseksi.”
        K&K ”Raamatulla päähän” 20.09.2010 Juhani Huttunen

        ”Tätä fundamentteihin eli perusteisiin…”

        Koska jokaisessa uskonnossa on oma liikkeensä, joka tosiaan painottaa omia perusteitaan, ja heidän asenteensa sekä toimintamallinsa on hyvin samankaltainen, alettiin käyttämään heistäkin ilmaisua ”fundamentalisti”.

        ”…suurin osa fundamentalismia käsitelleistä uskontotieteilijöistä on sitä mieltä, että käsite viittaa selkeästi rajattavissa olevaan ilmiökokonaisuuteen, joka voidaan tunnistaa monista eri uskontotraditioista. Käsite on myös mahdollista määritellä varsin neutraalisti ja deskriptiivisesti siten, että se tavoittaa joukon reaalimaailman ilmiöitä, joilla on yhteisiä piirteitä ja joille voidaan etsiä erilaisia selitysmalleja.”
        Ketola ”Mitä on fundamentalismi”

        Joten, jos haluat käyttää tuota omaperäistä määritelmää, se on hyvä ilmaista siksi, ettet joudu jatkuvasti väärinymmärretyksi. Koska yleensä termiä käytetään laajemmassa tarkoituksessa, ja toki voi eritellä ilmaisemalla vaikka evankelikaaliset – jolloin tiedetään, keitä fundamentalisteja erityisesti tarkoitetaan.

        Elämän alku oli ihmiskeskeistä, ihminen oli Jumalan luomistyön tärkein hedelmä, meidät luotiin yhteyteen Jumalan kanssa. Kuolemaa ei luotu elämän edellytykseksi vaan elämän väärinkäytön vuoksi.

        Kaikkea alkuperäistä merkitystä vääristellään, varsinkin nykyään tänä kulttuurimarksilaisena pakkoistamisen aikana. Mutta ei se muuta alkuperäistä merkitystä vaan siinä tehdään väkivaltaa eikä kunnioiteta käsitteen luoneita. Olet mieltynyt lohkoihin ja lahkoihin, erotat evankelikaaliset
        fundamentalisteista vaikka kerrot, että fundamentalistit ottivat uuden nimen evankelikaaliset. En voi kertakaikkiaan ymmärtää tuollaista lohko-ajattelua. Mihinkähän lohkoon itsesi asetat?

        Tuollainen mieltyneisyys lahko-jaotteluun on lihan aikaansaannoksia, josta Paavali varoittaa: ne, jotka syyllistyvät tällaiseen, eivät saa omakseen Jumalan valtakuntaa. Gal. 5:19-21.

        Uskontotieteilijät voivat määritellä omiaan, mutta kohteet, joita heidän luomansa käsitteet koskevat, ovat oikeutettuja määrittelemään itse itsensä eikä alistumaan muiden määritelmiin.

        Viesteistä kyllä näkee, jos puhutaan kristillisistä fundamentalisteista. Sinä kyllä sekoilet ja yhdistät kristityt ja muslimit fundamentalismi-letkautuksillasi.


      • onko.loppu kirjoitti:

        Toteamuksellasi "Kuolemaa on ollut yhtä kauan kuin elämääkin" kerrot sen, minkä Raamattu ilmoittaa. Kuolema tuli synnin seurauksena. Ihminen luotiin kuolemattomaksi, mutta synnin vuoksi piti fyysinen elämä katkaista kuolemalla, muutenhan synti olisi jatkunut ikuisuuksia. Emme tiedä, miten elämä olisi jatkunut ilman syntiinlankeemusta, mutta kyllä ihmisen tekijä tiesi mitä teki. Tiesi senkin, että ihminen lankeaa ja siihen oli ratkaisu valmiina: Jeesus, joka oli jo luomisessa mukana. Meidän äly ei vain yksinkertaisesti riitä käsittämään kaikkea eikä varsinkaan Jumalan älyä.

        Koko luotu maapallo kärsii ihmisen lankeemuksesta. Jumalahan ne solut on luonut ja määrännyt ne heikkenemään tietyn ajanjakson jälkeen kun lyhensi ihmisen elinikää. Sehän taas pitenee tuhatvuotisessa valtakunnassa kun “Ei siellä ole enää lasta, joka eläisi vain muutaman päivän, ei vanhusta, joka ei täyttäisi päiviensä määrää; sillä nuorin kuolee satavuotiaana, ja vasta satavuotiaana synnintekijä joutuu kiroukseen.” Jes. 65:20. Jumala poistaa kärsimyksen kerran, suunnitelmansa mene koko ajan eteenpäin, nyt elämme vahvaa antikristukselle sijaa tekevää maailman yhdentymisen aikaa. Profetiat toteutuvat silmiemme edessä.

        “Sentähden, niinkuin yhden ihmisen kautta synti tuli maailmaan, ja synnin kautta kuolema, niin kuolema on tullut kaikkien ihmisten osaksi, koska kaikki ovat syntiä tehneet.” Room. 5:12

        Tiedämme varmuudella? Kuinka voi tietää varmuudella, kun ei ole mitään varmuutta? Ei ole havaintoja miljoonien vuosien kehityksestä. Kun sanotaan, ettei kehitystä voi nähdä sen hitauden vuoksi, niin miten ihmeessä voidaan nähdä todisteita siitä kehityksestä? Pelkkää tarinankerrontaa eikä tieteellistä todistusta. Erilaiset olennot ei todista, että ne kaikki polveutuisi yksinkertaisemmasta kantamuodosta.

        Ei ole kahta eri luomiskertomusta vaan yksi ja sitä seuraa tarkempi kuvaus ihmisen luomisesta.

        Kyllä on kaksi erilaista luomiskertomusta ja vielä muistumia kolmannestakin.
        Kuolema on elämän ehto ei paha asia vaikka yksilöistä se sellaiselta tuntuukin, mutta ei aina: "Ja Aabraham vaipui kuolemaan korkeassa iässä, vanhana ja elämästä kyllänsä saaneena"


      • onko.loppu kirjoitti:

        Elämän alku oli ihmiskeskeistä, ihminen oli Jumalan luomistyön tärkein hedelmä, meidät luotiin yhteyteen Jumalan kanssa. Kuolemaa ei luotu elämän edellytykseksi vaan elämän väärinkäytön vuoksi.

        Kaikkea alkuperäistä merkitystä vääristellään, varsinkin nykyään tänä kulttuurimarksilaisena pakkoistamisen aikana. Mutta ei se muuta alkuperäistä merkitystä vaan siinä tehdään väkivaltaa eikä kunnioiteta käsitteen luoneita. Olet mieltynyt lohkoihin ja lahkoihin, erotat evankelikaaliset
        fundamentalisteista vaikka kerrot, että fundamentalistit ottivat uuden nimen evankelikaaliset. En voi kertakaikkiaan ymmärtää tuollaista lohko-ajattelua. Mihinkähän lohkoon itsesi asetat?

        Tuollainen mieltyneisyys lahko-jaotteluun on lihan aikaansaannoksia, josta Paavali varoittaa: ne, jotka syyllistyvät tällaiseen, eivät saa omakseen Jumalan valtakuntaa. Gal. 5:19-21.

        Uskontotieteilijät voivat määritellä omiaan, mutta kohteet, joita heidän luomansa käsitteet koskevat, ovat oikeutettuja määrittelemään itse itsensä eikä alistumaan muiden määritelmiin.

        Viesteistä kyllä näkee, jos puhutaan kristillisistä fundamentalisteista. Sinä kyllä sekoilet ja yhdistät kristityt ja muslimit fundamentalismi-letkautuksillasi.

        ”Kuolemaa ei luotu elämän edellytykseksi vaan elämän väärinkäytön vuoksi.”

        ET siis ole koskaan ajatellut luonnossa olevaa kiertokulkua? Miten kasvit itävät, juurtuvat, kasvavat, tuottavat siemeniä ja lopulta lahoavat tai kaatuvat ja maatuvat. Näin kaikki elävä tekee – on alku, kasvu, lisääntyminen, vanhuus ja kuolema. Oli kyse ihmisestä, eläimestä, kasvista jne.

        Olen joskus yrittänyt hahmottaa, millaista olisi, kun ei olisi kehitystä tai kuolemaa. Kukaan ei kasvaisi, kukaan ei syntyisi, kukaan muuttuisi – mutta mihinkä ikään sitä juututtaisiin? Vai oltaisiinko tosiaan vanhuksia monta sataa vuotta? Vai ikuisesti?

        Mitenkä sitten ravinto? Sehän on aina jonkun kuolema – oli kyse kasveista tai eläinravinnosta. Kasvitkin kuolevat ruuansulatuselimistössä ja näin tuottavat elämää. Vai eikö enää syötäisi mitään?
        No, kestämättömältä tuo kuulostaa.

        ”En voi kertakaikkiaan ymmärtää tuollaista lohko-ajattelua. Mihinkähän lohkoon itsesi asetat?”

        No, ehkä et voi ymmärtää mutta se on meidän ihmisten tapa hahmottaa suurempia kokonaisuuksia. Jaamme erilaisiin ryhmiin jonkun perusteen mukaan. Uskonnoissa noita perusteita on monia. Kristinuskossakin on opillisia juttuja, jotka ovat synnyttäneet eri kirkkokuntia.
        Kyllä se mielestäni kuuluu yleissivistykseen esim. tietää, miten protestantit erottuivat katolisissa ja miten meitä protestantteja on lukuisia – osa opeista on samoja mutta osa erilaisia.

        ”…mutta kohteet, joita heidän luomansa käsitteet koskevat, ovat oikeutettuja määrittelemään itse itsensä eikä alistumaan muiden määritelmiin.”

        Niin, sitä kutsutaan kieleksi, millä ilmaistaan se, mitä tarkemmin tarkoitetaan. Saahan sitä jokainen keksiä vaikka oman kielensä, ei se siitä ole kiinni. Paremmin vain ymmärrämme toisiamme, jos edes pyrimme käyttämään samaa kieltä ja samansuuntaisia termejä.

        Toisaalta, noinhan nyt tässä seksuaalisessa identiteetti kysymyksessäkin jotkut haluaa, että he itse saavat sen määritellä ja samoin saavat määritellä oman sukupuolensa.


      • onko.loppu
        dikduk kirjoitti:

        Kyllä on kaksi erilaista luomiskertomusta ja vielä muistumia kolmannestakin.
        Kuolema on elämän ehto ei paha asia vaikka yksilöistä se sellaiselta tuntuukin, mutta ei aina: "Ja Aabraham vaipui kuolemaan korkeassa iässä, vanhana ja elämästä kyllänsä saaneena"

        Miten se voi olla elämän ehto, kun kuolema kerran kukistetaan? Miten elämää voi olla sitten, kun ei ole kuolemaa? Siinäpä dilemma ihmisen ratkaistavaksi.

        Vesa Annala kirjoittaa, että modernin raamattukriittisyyden myötä ruvettiin spekuloimaan ja hajottamaan raamatuntekstejä. Teologit alkoivat "nähdä" kaksi luomiskertomusta Raamatussa niin myöhään kuin 1700- ja 1800-luvulla, tieteellisen ja filosofisen kehityksen seurauksena. Tämän mukaan polveutuu "ensimmäinen luomiskertomus" n.k. pappiskirjoituksesta ("P", oletettu kirjoittaja) ja toinen Jahve-kirjoituksesta "J" (myös joku oletettu kirjoittaja).

        Monenlaista käsitystä on historia tuottanut.


      • onko.loppu
        mummomuori kirjoitti:

        ”Kuolemaa ei luotu elämän edellytykseksi vaan elämän väärinkäytön vuoksi.”

        ET siis ole koskaan ajatellut luonnossa olevaa kiertokulkua? Miten kasvit itävät, juurtuvat, kasvavat, tuottavat siemeniä ja lopulta lahoavat tai kaatuvat ja maatuvat. Näin kaikki elävä tekee – on alku, kasvu, lisääntyminen, vanhuus ja kuolema. Oli kyse ihmisestä, eläimestä, kasvista jne.

        Olen joskus yrittänyt hahmottaa, millaista olisi, kun ei olisi kehitystä tai kuolemaa. Kukaan ei kasvaisi, kukaan ei syntyisi, kukaan muuttuisi – mutta mihinkä ikään sitä juututtaisiin? Vai oltaisiinko tosiaan vanhuksia monta sataa vuotta? Vai ikuisesti?

        Mitenkä sitten ravinto? Sehän on aina jonkun kuolema – oli kyse kasveista tai eläinravinnosta. Kasvitkin kuolevat ruuansulatuselimistössä ja näin tuottavat elämää. Vai eikö enää syötäisi mitään?
        No, kestämättömältä tuo kuulostaa.

        ”En voi kertakaikkiaan ymmärtää tuollaista lohko-ajattelua. Mihinkähän lohkoon itsesi asetat?”

        No, ehkä et voi ymmärtää mutta se on meidän ihmisten tapa hahmottaa suurempia kokonaisuuksia. Jaamme erilaisiin ryhmiin jonkun perusteen mukaan. Uskonnoissa noita perusteita on monia. Kristinuskossakin on opillisia juttuja, jotka ovat synnyttäneet eri kirkkokuntia.
        Kyllä se mielestäni kuuluu yleissivistykseen esim. tietää, miten protestantit erottuivat katolisissa ja miten meitä protestantteja on lukuisia – osa opeista on samoja mutta osa erilaisia.

        ”…mutta kohteet, joita heidän luomansa käsitteet koskevat, ovat oikeutettuja määrittelemään itse itsensä eikä alistumaan muiden määritelmiin.”

        Niin, sitä kutsutaan kieleksi, millä ilmaistaan se, mitä tarkemmin tarkoitetaan. Saahan sitä jokainen keksiä vaikka oman kielensä, ei se siitä ole kiinni. Paremmin vain ymmärrämme toisiamme, jos edes pyrimme käyttämään samaa kieltä ja samansuuntaisia termejä.

        Toisaalta, noinhan nyt tässä seksuaalisessa identiteetti kysymyksessäkin jotkut haluaa, että he itse saavat sen määritellä ja samoin saavat määritellä oman sukupuolensa.

        Elämän kiertokulku on nyt se minkä Jumala on laittanut, kun kuolema tuli maailmaan lankeemuksen seurauksena. Olet rajoittunut näkemään vain tämän nykyisen maailman. Et voi tietää, milaista olisi ilman kuolemaa. Rajoitat Jumalaa.

        Kasvit olivat eläinten ja ihmisten ravintona aluksi, mutta vedenpaisumuksen jälkeen maailma oli karu, josta ei riittänyt alkuperäinen ravinto. Elämet alkoivat saalistaa muita eläimiä ja ihmisille annettiin myös eläimet syötäväksi, koska kuolema oli tullut maailmaan.

        Sivistykseen ei kuulu jatkuvasti haukkua lohkoista ja lahkoista eikä solvata nimeltä mainiten muita ihmisiä natseiksi tai rasisteiksi perusteetta, vain omien luulojen pohjalta.

        Seksuaalisuuden loi Jumala, ei ihminen sitä määritä. Uskon määritelmät loi myös Jumala ja ihmisen kuuluu ottaa ne Raamatusta. Jos Raamattu-uskolliset ovat määritelleet fundamentalistisen näkemyksen Raamatun perustalta, sitä ei muut voi määritellä, eikä varsinkaan ympätä muiden uskontojen määritelmiä siihen. Se ei ole oman kielen luomista vaan pysyttäytymisessä Jumalan sanaan. Uskontotielteilijä luovat oman kielensä ja määritelmänsä.


      • onko.loppu kirjoitti:

        Elämän kiertokulku on nyt se minkä Jumala on laittanut, kun kuolema tuli maailmaan lankeemuksen seurauksena. Olet rajoittunut näkemään vain tämän nykyisen maailman. Et voi tietää, milaista olisi ilman kuolemaa. Rajoitat Jumalaa.

        Kasvit olivat eläinten ja ihmisten ravintona aluksi, mutta vedenpaisumuksen jälkeen maailma oli karu, josta ei riittänyt alkuperäinen ravinto. Elämet alkoivat saalistaa muita eläimiä ja ihmisille annettiin myös eläimet syötäväksi, koska kuolema oli tullut maailmaan.

        Sivistykseen ei kuulu jatkuvasti haukkua lohkoista ja lahkoista eikä solvata nimeltä mainiten muita ihmisiä natseiksi tai rasisteiksi perusteetta, vain omien luulojen pohjalta.

        Seksuaalisuuden loi Jumala, ei ihminen sitä määritä. Uskon määritelmät loi myös Jumala ja ihmisen kuuluu ottaa ne Raamatusta. Jos Raamattu-uskolliset ovat määritelleet fundamentalistisen näkemyksen Raamatun perustalta, sitä ei muut voi määritellä, eikä varsinkaan ympätä muiden uskontojen määritelmiä siihen. Se ei ole oman kielen luomista vaan pysyttäytymisessä Jumalan sanaan. Uskontotielteilijä luovat oman kielensä ja määritelmänsä.

        ”Olet rajoittunut näkemään vain tämän nykyisen maailman. Et voi tietää, milaista olisi ilman kuolemaa. Rajoitat Jumalaa.”

        :D En ole rajoittunut. Siispä en ajattele ikuista elämää tällaisessa olomuodossa, missä nyt olemme. Se ei yksikertaisesti ole mahdollista. Se, mitä se voisi olla, voimme vain arvailla.

        ”Sivistykseen ei kuulu jatkuvasti haukkua lohkoista ja lahkoista eikä solvata nimeltä mainiten muita ihmisiä natseiksi tai rasisteiksi perusteetta, vain omien luulojen pohjalta.”

        Olen aivan samaa mieltä. Siksi tulee käyttää asiallisia termejä. Samoin tulee perustella, miksi katsoo jonkun esim. olevan uusnatsi tai rasisti – eli kun henkilö selkeästi tekee tai sanoo jotain, mikä kuuluu tuon tietyn aatteen sisältöön.
        Samoin auttaa paljon kun ymmärtää tietyn kristinuskon suutauksesta jotakin. Näin voi paremmin havaita, mistä mm. joku erimielisyys johtuu – vaikka kasteen, ehtoollisen, pappeuden jne. suhteen. Toki on monia muita opillisia erityispiirteitä.

        ”Jos Raamattu-uskolliset ovat määritelleet fundamentalistisen näkemyksen Raamatun perustalta, sitä ei muut voi määritellä, eikä varsinkaan ympätä muiden uskontojen määritelmiä siihen.”

        Kun tietää fundamentalismin historian 1800 luvun lopusta ja sen, mihin suuntaan se on kehittynyt, voi toki muutkin määritellä asiaa. Koska aivan vastaavia piirteitä löytyy muidenkin uskontojen parista, hyvällä syyllä voi nuo ryhmitellä saman ilmaisun alle.

        ”Se ei ole oman kielen luomista vaan pysyttäytymisessä Jumalan sanaan. Uskontotielteilijä luovat oman kielensä ja määritelmänsä…”

        Kyllä se on kielen luomista. Sinällään siinä ei ole mitään pahaa, jargonit ovat aika yleisiä. Mutta kannattaa avata ne, jotta muutkin voivat ymmärtää ja näin voi saada itsensä paremmin ymmärretyksi.
        Onhan jo Raamattua käännettäessä pitänyt tehdä tulkintaa – ei mitään voida sanatarkasti kääntää, koska eri kielet rakentuvat eri tavoin.

        Uskontotieteilijät käyttävät yleisesti tunnistettavaa kieltä – toki myös tieteellistä jargonia. Heidän huomionsa on myös siinä, että havaintoihin perustuvia määritelmiä tehdään, ja ne auttavat asian hahmottamisessa.


      • onko.loppu kirjoitti:

        Miten se voi olla elämän ehto, kun kuolema kerran kukistetaan? Miten elämää voi olla sitten, kun ei ole kuolemaa? Siinäpä dilemma ihmisen ratkaistavaksi.

        Vesa Annala kirjoittaa, että modernin raamattukriittisyyden myötä ruvettiin spekuloimaan ja hajottamaan raamatuntekstejä. Teologit alkoivat "nähdä" kaksi luomiskertomusta Raamatussa niin myöhään kuin 1700- ja 1800-luvulla, tieteellisen ja filosofisen kehityksen seurauksena. Tämän mukaan polveutuu "ensimmäinen luomiskertomus" n.k. pappiskirjoituksesta ("P", oletettu kirjoittaja) ja toinen Jahve-kirjoituksesta "J" (myös joku oletettu kirjoittaja).

        Monenlaista käsitystä on historia tuottanut.

        En ymmärrä kristinuskon ajatusta että kuolema pitää kukistaa, hepr. raamatun mukaan Jumala on kaiken maailmassa olevan luoja, myös kuoleman, ei hänen tarvitse kukistaa mitään itse luomaansa. Jos pysytään luonontieteiden piirissä niin tiedämme että ihminen on nuori laji ja maapallo olisi täyttynyt vanhimmista elämänmuodoista jo ennen ihmislajia jos kuolemaa ei olisi .Meitäkään ei silloin olisi .
        Kuolema on solutason tapahtuma, ei teologinen tai filosofinen, vaikka sitä voidaan filosofoida ja sille rakennella erilaisia teologisia selityksiä kuten uskonnot ovat tehneet.Olemme laji jonka ykisilöt tietävät kuolevansa, joten kuolema on myös pelottava ja koko ajan läsnä. Siksi se on myös uskonnoissa niin voimakkasti esillä ja jotenkin vaatii selityksen .

        Raamatussa on eriaikaisia tekstejä ne voidaan esim.hepr.raamatun (VT) osalta ajoittaa siitä millaista hepreaa kirjoittaja käyttää, mitkä ovat tyylikeinot, mitä lainasanoja tekstissä on -hepreahan kehittyi ja muuttui koko jan kuten kaikki kielet tekevät- ja tekstit voidaan ajoittaa myös monen muun seikan perusteella .
        Hepr. raamatun 1.Mooseksen kirjan luvuissa 1ja 2 on kaksi erilaista luomiskertomusta, ne ovat keskenään ristiriitaiset ja niissä mm käytetään eri nimeä jumalasta. Ne ovat selkeästi eri lähteistä. Ei tutkimus ole noita eroja tuottanut vaan se tutkii asiaa joka tekstissä näkyy.


      • voikapinanmäärää
        dikduk kirjoitti:

        En ymmärrä kristinuskon ajatusta että kuolema pitää kukistaa, hepr. raamatun mukaan Jumala on kaiken maailmassa olevan luoja, myös kuoleman, ei hänen tarvitse kukistaa mitään itse luomaansa. Jos pysytään luonontieteiden piirissä niin tiedämme että ihminen on nuori laji ja maapallo olisi täyttynyt vanhimmista elämänmuodoista jo ennen ihmislajia jos kuolemaa ei olisi .Meitäkään ei silloin olisi .
        Kuolema on solutason tapahtuma, ei teologinen tai filosofinen, vaikka sitä voidaan filosofoida ja sille rakennella erilaisia teologisia selityksiä kuten uskonnot ovat tehneet.Olemme laji jonka ykisilöt tietävät kuolevansa, joten kuolema on myös pelottava ja koko ajan läsnä. Siksi se on myös uskonnoissa niin voimakkasti esillä ja jotenkin vaatii selityksen .

        Raamatussa on eriaikaisia tekstejä ne voidaan esim.hepr.raamatun (VT) osalta ajoittaa siitä millaista hepreaa kirjoittaja käyttää, mitkä ovat tyylikeinot, mitä lainasanoja tekstissä on -hepreahan kehittyi ja muuttui koko jan kuten kaikki kielet tekevät- ja tekstit voidaan ajoittaa myös monen muun seikan perusteella .
        Hepr. raamatun 1.Mooseksen kirjan luvuissa 1ja 2 on kaksi erilaista luomiskertomusta, ne ovat keskenään ristiriitaiset ja niissä mm käytetään eri nimeä jumalasta. Ne ovat selkeästi eri lähteistä. Ei tutkimus ole noita eroja tuottanut vaan se tutkii asiaa joka tekstissä näkyy.

        siinäpä seongelma onkin kun ette ymmärrä mitään aiheeata josta olett e kovasti jotain selittävinänne.

        kokeilkaa ostaa joululahjat ja tehdä joulupaketteja silmät kiinni, se onnistuu paremmin luulen.


      • dikduk kirjoitti:

        En ymmärrä kristinuskon ajatusta että kuolema pitää kukistaa, hepr. raamatun mukaan Jumala on kaiken maailmassa olevan luoja, myös kuoleman, ei hänen tarvitse kukistaa mitään itse luomaansa. Jos pysytään luonontieteiden piirissä niin tiedämme että ihminen on nuori laji ja maapallo olisi täyttynyt vanhimmista elämänmuodoista jo ennen ihmislajia jos kuolemaa ei olisi .Meitäkään ei silloin olisi .
        Kuolema on solutason tapahtuma, ei teologinen tai filosofinen, vaikka sitä voidaan filosofoida ja sille rakennella erilaisia teologisia selityksiä kuten uskonnot ovat tehneet.Olemme laji jonka ykisilöt tietävät kuolevansa, joten kuolema on myös pelottava ja koko ajan läsnä. Siksi se on myös uskonnoissa niin voimakkasti esillä ja jotenkin vaatii selityksen .

        Raamatussa on eriaikaisia tekstejä ne voidaan esim.hepr.raamatun (VT) osalta ajoittaa siitä millaista hepreaa kirjoittaja käyttää, mitkä ovat tyylikeinot, mitä lainasanoja tekstissä on -hepreahan kehittyi ja muuttui koko jan kuten kaikki kielet tekevät- ja tekstit voidaan ajoittaa myös monen muun seikan perusteella .
        Hepr. raamatun 1.Mooseksen kirjan luvuissa 1ja 2 on kaksi erilaista luomiskertomusta, ne ovat keskenään ristiriitaiset ja niissä mm käytetään eri nimeä jumalasta. Ne ovat selkeästi eri lähteistä. Ei tutkimus ole noita eroja tuottanut vaan se tutkii asiaa joka tekstissä näkyy.

        "Olemme laji jonka ykisilöt tietävät kuolevansa, joten kuolema on myös pelottava ja koko ajan läsnä. Siksi se on myös uskonnoissa niin voimakkasti esillä ja jotenkin vaatii selityksen ."

        Aivan. Ja meitä pitää elossa toivo. Meillä kristityillä toivo on ehkä erilainen kuin muilla, mutta ellei toivoa ole, ei ole halua elääkään.


      • onko.loppu
        mummomuori kirjoitti:

        ”Olet rajoittunut näkemään vain tämän nykyisen maailman. Et voi tietää, milaista olisi ilman kuolemaa. Rajoitat Jumalaa.”

        :D En ole rajoittunut. Siispä en ajattele ikuista elämää tällaisessa olomuodossa, missä nyt olemme. Se ei yksikertaisesti ole mahdollista. Se, mitä se voisi olla, voimme vain arvailla.

        ”Sivistykseen ei kuulu jatkuvasti haukkua lohkoista ja lahkoista eikä solvata nimeltä mainiten muita ihmisiä natseiksi tai rasisteiksi perusteetta, vain omien luulojen pohjalta.”

        Olen aivan samaa mieltä. Siksi tulee käyttää asiallisia termejä. Samoin tulee perustella, miksi katsoo jonkun esim. olevan uusnatsi tai rasisti – eli kun henkilö selkeästi tekee tai sanoo jotain, mikä kuuluu tuon tietyn aatteen sisältöön.
        Samoin auttaa paljon kun ymmärtää tietyn kristinuskon suutauksesta jotakin. Näin voi paremmin havaita, mistä mm. joku erimielisyys johtuu – vaikka kasteen, ehtoollisen, pappeuden jne. suhteen. Toki on monia muita opillisia erityispiirteitä.

        ”Jos Raamattu-uskolliset ovat määritelleet fundamentalistisen näkemyksen Raamatun perustalta, sitä ei muut voi määritellä, eikä varsinkaan ympätä muiden uskontojen määritelmiä siihen.”

        Kun tietää fundamentalismin historian 1800 luvun lopusta ja sen, mihin suuntaan se on kehittynyt, voi toki muutkin määritellä asiaa. Koska aivan vastaavia piirteitä löytyy muidenkin uskontojen parista, hyvällä syyllä voi nuo ryhmitellä saman ilmaisun alle.

        ”Se ei ole oman kielen luomista vaan pysyttäytymisessä Jumalan sanaan. Uskontotielteilijä luovat oman kielensä ja määritelmänsä…”

        Kyllä se on kielen luomista. Sinällään siinä ei ole mitään pahaa, jargonit ovat aika yleisiä. Mutta kannattaa avata ne, jotta muutkin voivat ymmärtää ja näin voi saada itsensä paremmin ymmärretyksi.
        Onhan jo Raamattua käännettäessä pitänyt tehdä tulkintaa – ei mitään voida sanatarkasti kääntää, koska eri kielet rakentuvat eri tavoin.

        Uskontotieteilijät käyttävät yleisesti tunnistettavaa kieltä – toki myös tieteellistä jargonia. Heidän huomionsa on myös siinä, että havaintoihin perustuvia määritelmiä tehdään, ja ne auttavat asian hahmottamisessa.

        > En ole rajoittunut. Siispä en ajattele ikuista elämää tällaisessa olomuodossa, missä nyt olemme. Se ei yksikertaisesti ole mahdollista. Se, mitä se voisi olla, voimme vain arvailla.<

        Missä olen puhunut ikuisesta elämästä? Sinä puhut luonnon kiertokulusta aivan kuin se olisi ainut tapa elämälle täällä. Se tuli lankeemuksen seurauksena, kuolema määrittää elämää nyt, mutta et voi tietää, mitä elämä olisi ollut ilman lankeemuksesta aiheutunutta kuolemaa. Jumalalle kaikki on mahdollista, hän voi muuttaa luomansa elämää väliaikaisesti toiseksi kuin se alkuperältään oli. Hän voi siirtää esim. atomiytimessä protonit neutronien tilalle ja neutronit protonien tilalle. Tällaisen mahdollisen selityksen muistan kuulleeni ylösnousseiden ruumiin rakenteeksi. Kun Jeesus nousi kuolleista, hän pystyi menemään seinien läpi, mutta oli edelleen saman näköinen ja söi ruokaa kuin ennen kuolemistaan. Tämä on esimerkki, miten Jumala voi omaa luomistyötään muuttaa. Toivon, ettet typeriä nauru-hymiöitäsi postaile tähän, en minä sano, että näin on vaan näin olen kuullut. Jumala voi luoda ihan uutta materiaa, esim. moni on kokenut ruuan monistumisen tai lisääntymistä tilanteissa, joissa syöjiä on ollut enemmän kuin ruokaa, Raamatun kala-leipä ihmeen tavoin.

        >”Sivistykseen ei kuulu jatkuvasti haukkua lohkoista ja lahkoista eikä solvata nimeltä mainiten muita ihmisiä natseiksi tai rasisteiksi perusteetta, vain omien luulojen pohjalta.”

        – Olen aivan samaa mieltä. Siksi tulee käyttää asiallisia termejä. Samoin tulee perustella, miksi katsoo jonkun esim. olevan uusnatsi tai rasisti – eli kun henkilö selkeästi tekee tai sanoo jotain, mikä kuuluu tuon tietyn aatteen sisältöön. <

        Et ole, et ainakaan kirjoituksillasi sitä osoita. Pyydettäessä etkä muutenkaan et koskaan laita perusteita väitteillesi, katoat vain kuin tuhka tuuleen tai alat vänkäämään savuverhona asian vierestä.

        Sinun suuntausvimmasi on etovaa, se ei ole muuta kuin parjausta, mikä paistaa kirjoituksistasi.

        No ei voi asettaa islamia ja kristinuskoa saman suuntauksen alle!

        Ei ole oman kielen luomista - onpas- eipäs - onpas - eipäs….


      • onko.loppu
        dikduk kirjoitti:

        En ymmärrä kristinuskon ajatusta että kuolema pitää kukistaa, hepr. raamatun mukaan Jumala on kaiken maailmassa olevan luoja, myös kuoleman, ei hänen tarvitse kukistaa mitään itse luomaansa. Jos pysytään luonontieteiden piirissä niin tiedämme että ihminen on nuori laji ja maapallo olisi täyttynyt vanhimmista elämänmuodoista jo ennen ihmislajia jos kuolemaa ei olisi .Meitäkään ei silloin olisi .
        Kuolema on solutason tapahtuma, ei teologinen tai filosofinen, vaikka sitä voidaan filosofoida ja sille rakennella erilaisia teologisia selityksiä kuten uskonnot ovat tehneet.Olemme laji jonka ykisilöt tietävät kuolevansa, joten kuolema on myös pelottava ja koko ajan läsnä. Siksi se on myös uskonnoissa niin voimakkasti esillä ja jotenkin vaatii selityksen .

        Raamatussa on eriaikaisia tekstejä ne voidaan esim.hepr.raamatun (VT) osalta ajoittaa siitä millaista hepreaa kirjoittaja käyttää, mitkä ovat tyylikeinot, mitä lainasanoja tekstissä on -hepreahan kehittyi ja muuttui koko jan kuten kaikki kielet tekevät- ja tekstit voidaan ajoittaa myös monen muun seikan perusteella .
        Hepr. raamatun 1.Mooseksen kirjan luvuissa 1ja 2 on kaksi erilaista luomiskertomusta, ne ovat keskenään ristiriitaiset ja niissä mm käytetään eri nimeä jumalasta. Ne ovat selkeästi eri lähteistä. Ei tutkimus ole noita eroja tuottanut vaan se tutkii asiaa joka tekstissä näkyy.

        Kukistetaan eli hävitetään.

        https://rll.fi/2014/08/kuinka-kuolema-kukistetaan-vihollisista-viimeisena-vaikka-jeesus-voitti-jo-sen/

        Minä taasen en ymmärrä, että juutalaisena asetat Jumalan olemassaolon epäilyksen alaiseksi ja silti olet väíttelemässä kirjoituksista.


      • onko.loppu kirjoitti:

        > En ole rajoittunut. Siispä en ajattele ikuista elämää tällaisessa olomuodossa, missä nyt olemme. Se ei yksikertaisesti ole mahdollista. Se, mitä se voisi olla, voimme vain arvailla.<

        Missä olen puhunut ikuisesta elämästä? Sinä puhut luonnon kiertokulusta aivan kuin se olisi ainut tapa elämälle täällä. Se tuli lankeemuksen seurauksena, kuolema määrittää elämää nyt, mutta et voi tietää, mitä elämä olisi ollut ilman lankeemuksesta aiheutunutta kuolemaa. Jumalalle kaikki on mahdollista, hän voi muuttaa luomansa elämää väliaikaisesti toiseksi kuin se alkuperältään oli. Hän voi siirtää esim. atomiytimessä protonit neutronien tilalle ja neutronit protonien tilalle. Tällaisen mahdollisen selityksen muistan kuulleeni ylösnousseiden ruumiin rakenteeksi. Kun Jeesus nousi kuolleista, hän pystyi menemään seinien läpi, mutta oli edelleen saman näköinen ja söi ruokaa kuin ennen kuolemistaan. Tämä on esimerkki, miten Jumala voi omaa luomistyötään muuttaa. Toivon, ettet typeriä nauru-hymiöitäsi postaile tähän, en minä sano, että näin on vaan näin olen kuullut. Jumala voi luoda ihan uutta materiaa, esim. moni on kokenut ruuan monistumisen tai lisääntymistä tilanteissa, joissa syöjiä on ollut enemmän kuin ruokaa, Raamatun kala-leipä ihmeen tavoin.

        >”Sivistykseen ei kuulu jatkuvasti haukkua lohkoista ja lahkoista eikä solvata nimeltä mainiten muita ihmisiä natseiksi tai rasisteiksi perusteetta, vain omien luulojen pohjalta.”

        – Olen aivan samaa mieltä. Siksi tulee käyttää asiallisia termejä. Samoin tulee perustella, miksi katsoo jonkun esim. olevan uusnatsi tai rasisti – eli kun henkilö selkeästi tekee tai sanoo jotain, mikä kuuluu tuon tietyn aatteen sisältöön. <

        Et ole, et ainakaan kirjoituksillasi sitä osoita. Pyydettäessä etkä muutenkaan et koskaan laita perusteita väitteillesi, katoat vain kuin tuhka tuuleen tai alat vänkäämään savuverhona asian vierestä.

        Sinun suuntausvimmasi on etovaa, se ei ole muuta kuin parjausta, mikä paistaa kirjoituksistasi.

        No ei voi asettaa islamia ja kristinuskoa saman suuntauksen alle!

        Ei ole oman kielen luomista - onpas- eipäs - onpas - eipäs….

        ”Sinä puhut luonnon kiertokulusta aivan kuin se olisi ainut tapa elämälle täällä.”

        ?? Siis se mitä me ymmärrämme elämällä, on tosiaan se ainut muoto.

        ”…et voi tietää, mitä elämä olisi ollut ilman lankeemuksesta aiheutunutta kuolemaa.”

        En, en todellakaan. Se ei olisi tätä elämää, jota nyt elämme. Ehkä sillä olisi aivan toinen termikin? Toki voimme visioida, millaista joku ei materiaalinen olotila voisi olla.

        ”Pyydettäessä etkä muutenkaan et koskaan laita perusteita väitteillesi…”

        Pyrin miltei aina laittamaan. Poikkeuksen tekee pari juttua – jos asiasta on jo monia ketjuja, joissa olen vastannut perusteellisesti, ei aina jaksa toistaa. Toinen on sitten trollaus, joilla ei taida olla edes tarkoitus muuta kuin tehdä kiusaa.


      • onko.loppu
        mummomuori kirjoitti:

        ”Sinä puhut luonnon kiertokulusta aivan kuin se olisi ainut tapa elämälle täällä.”

        ?? Siis se mitä me ymmärrämme elämällä, on tosiaan se ainut muoto.

        ”…et voi tietää, mitä elämä olisi ollut ilman lankeemuksesta aiheutunutta kuolemaa.”

        En, en todellakaan. Se ei olisi tätä elämää, jota nyt elämme. Ehkä sillä olisi aivan toinen termikin? Toki voimme visioida, millaista joku ei materiaalinen olotila voisi olla.

        ”Pyydettäessä etkä muutenkaan et koskaan laita perusteita väitteillesi…”

        Pyrin miltei aina laittamaan. Poikkeuksen tekee pari juttua – jos asiasta on jo monia ketjuja, joissa olen vastannut perusteellisesti, ei aina jaksa toistaa. Toinen on sitten trollaus, joilla ei taida olla edes tarkoitus muuta kuin tehdä kiusaa.

        Ihminne elää vaika olisi kuollut, sanotaan Raamatiussa. Elämä on aina elämää, jonka Jumala on luonut. Vain uusliberaali-uuskommunisti voi aatteensa mukaisesti yrittää keksiä uutta kieltä jopa elämälle.

        Jeesus ei ollut ei-materiaalinen ylösnoustuaan, häntä pystyi koskettamaan, hän söi tämän maailman materiaa.

        No kun et pyri laittamaan. Ja jos olisit vastannut jossain toisessa ketjussa, silloin järkevää on laittaa linkki, missä olet vastannut, koska kaikki ei lue kaikkia ketjuja. Sen sijaan vaikenet tai letkauttelet asiattomia.


    Ketjusta on poistettu 5 sääntöjenvastaista viestiä.

    Luetuimmat keskustelut

    1. Valkeakosken 15-v tapauksessa ihmettelen ??

      On sääli, että pahoja ihmisiä liikkuu aina vapaana eri puolilla Suomea, mutta minkä ihmeen takia 15-vuotiaan nuoren täyt
      Maailman menoa
      618
      19201
    2. Valkeakosken tappo

      "Tyttö löytyi poliisin mukaan kuolleena läheisestä metsästä muutaman sadan metrin päässä kotoaan. Uhrin löysivät hänen k
      Henkirikokset
      70
      16052
    3. Nyt ahdistaa

      Joku nuori tyttö on surmattu Valkeakoskella. En tunne ihmistä, mutta silti se koskettaa. Uutisissa oli hiljattain, että
      Valkeakoski
      394
      9519
    4. Kuka oli tekijä?

      Jos tekijä oli suomalainen, onko hänen vanhempiaan jo tavoitettu? Mitä mieltä ovat aikamiespoikansa teosta? Entä puoliso
      Valkeakoski
      83
      6755
    5. 15-vuotiaan ruumis valkeakoskella

      Nuoria tyttöjä tappavat miessaalistajat ja toiset nuoret. Miessaalistajille ruumiin kätkeminen tai tuhoaminen ei ole on
      Poliisi
      19
      4964
    6. Valkeakosken murhaaja-raiskaaja on kantasuomalainen mies tiedottaa poliisi

      Some- ja palstapersut ehtivät jo moneen kertaan julistaa tekijän maahanmuuttajaksi. Miten meni niin kuin omasta mielestä
      Maailman menoa
      213
      3971
    7. Kantasuomalainen mies pidätetty - ulkomaalaiset syyttömiä tekoon

      Verityöstä on pidätetty vuonna 2005 syntynyt mieshenkilö. Ulkomaalaisilla ei mitään yhteyttä tekoon.
      Valkeakoski
      127
      2222
    8. Mitä hänellä oli päällään kun viimeksi näit hänet?

      Avoimia vastauksia saa kirjoitella... Ehkä joku saattaa tunnistaa itsensä kommenttien joukosta :)
      Ikävä
      62
      1485
    9. Meidän tarinako on ohi?

      Ootko niin päättänyt?
      Ikävä
      111
      1437
    10. Miksi kaupunki toivoo nuorilta malttia?

      https://www.hs.fi/suomi/art-2000010453236.html "Älkää suunnitelko kostoa" Mistä on kyse? ”Toivomme, että nuorten taho
      Valkeakoski
      41
      1287
    Aihe